241
MODUL 2 Special pathomorphology Text tests 1 All of the following statements correctly describe hereditary spherocytosis, EXCEPT: A. *Red blood cells have the membrane-associated protein spectrin B. A mutation in the ankyrin gene is present in most cases C. Hemolytic crisis D. An aplastic crisis E. Splenectomy is invariably therapeutic 2 Features of megaloblastic anemias include all of the following, EXCEPT: A. Hypersegmented neutrophils B. *Lackof the membrane-associated protein spectrin C. Increased intramedullary hemolysis D. Increased extramedullary hemolysis E. Pancytopenia 3 All of the following statements correctly describe aplastic anemia, EXCEPT: A. Production of all hematopoietic bone marrow elements is reduced B. Chemical \ drug exposure is the most common cause C. Fanconi's anemia represents an inherited form of the disease D. No underlying etiology is evident in E0% of cases E. * Splenomegaly is a characteristic clinical finding 4 Multiple myeloma is associated with all of the following features, EXCEPT: A. Hypercalcemia B. Renal failure C. Amyloidosis D. * Increased susceptibility to viral infections E. Rouleau formation on peripheral smear 5 Osmotic fragility is characteristic of the erythrocytes in which of the following diseases: A. Fanconi's anemia B. Sickle cell anemia C. Glucose-6-phosphate dehydrogenase deficiency D. * Hereditary spherocytosis E. Pernicious anemia 6 Iron-deficiency anemia is commonly associated with all of the following factors, EXCEPT:

MODUL 2 Special pathomorphologyintranet.tdmu.edu.ua/data/kafedra/internal/patologanatom... · Web viewDisorders of cardiac conduction Systemic emboli in infective endocarditis may

Embed Size (px)

Citation preview

MODUL 2 Special pathomorphology

Text tests

1 All of the following statements correctly describe hereditary spherocytosis, EXCEPT:A. *Red blood cells have the membrane-associated protein spectrinB. A mutation in the ankyrin gene is present in most casesC. Hemolytic crisisD. An aplastic crisisE. Splenectomy is invariably therapeutic

2 Features of megaloblastic anemias include all of the following, EXCEPT:A. Hypersegmented neutrophilsB. *Lackof the membrane-associated protein spectrinC. Increased intramedullary hemolysisD. Increased extramedullary hemolysisE. Pancytopenia

3 All of the following statements correctly describe aplastic anemia, EXCEPT:A. Production of all hematopoietic bone marrow elements is reducedB. Chemical \ drug exposure is the most common causeC. Fanconi's anemia represents an inherited form of the diseaseD. No underlying etiology is evident in E0% of casesE. * Splenomegaly is a characteristic clinical finding

4 Multiple myeloma is associated with all of the following features, EXCEPT:A. HypercalcemiaB. Renal failureC. AmyloidosisD. * Increased susceptibility to viral infectionsE. Rouleau formation on peripheral smear

5 Osmotic fragility is characteristic of the erythrocytes in which of the following diseases:A. Fanconi's anemiaB. Sickle cell anemiaC. Glucose-6-phosphate dehydrogenase deficiencyD. * Hereditary spherocytosisE. Pernicious anemia

6 Iron-deficiency anemia is commonly associated with all of the following factors, EXCEPT:A. Colon cancerB. *PolycythemiaC. GastrectomyD. Normal mensesE. Pregnancy

7 Polycythemia vera is a proliferative disorder of stem cells that:A. Has an X-linked recessive mode of transmissionB. Is associated with high levels of erythropoietinC. Produces abnormalities in the red cell series onlyD. *Is rapidly fatal if untreatedE. Is compatible with a normal life span if treated

8 Increased blood viscosity (hyperviscosity syndrome) is major complication of all of the following disorders, EXCEPT:A. Polycythemia veraB. Immunoglobulin A myelomaC. *"Hairy-cell" leukemiaD. Waldenstrom's macroglobulinemia

E. Heavy-chain disease9 All of the following statements correctly describe nodular non- Hodgkin's lymphomas,

EXCEPT:A. They are often associated with a predictable chromosomal translocationB. They have a better prognosis than diffuse lymphomasC. They are characterized by cells with irregular nuclear contoursD. *They are commonly composed of neoplastic B cellsE. They are highly responsive to chemotherapy

10 All of the following features can be found in myeloid metaplasia with myelofibrosis, EXCEPT:A. *Clonal proliferation of marrow fibroblastsB. Giant platelets in the peripheral bloodC. Teardrop-shaped red cells in the peripheral bloodD. Elevated leukocyte alkaline phosphatase levels in the serumE. Massive splenomegaly

11 Transformation to acute leukemia occurs as a complication of all of the following disorders, EXCEPT:A. Paroxysmal nocturnal hemoglobinuriaB. Myeloid metaplasia with myelofibrosisC. Aplastic anemiaD. *"Hairy-cell" leukemiaE. Polycythemia vera

12 Intermediate grade lymphomas include all of the following, EXCEPT:A. Follicular, predominantly large-cell lymphomaB. * Follicular, predominantly small cleaved cell lymphomaC. Diffuse, small cleaved cell lymphomaD. Diffuse, mixed small- and large-cell lymphomaE. Diffuse large-cell lymphoma

13 Chronic myelogenous leukemia is characterized by all of the following features, EXCEPT:A. Lack of alkaline phosphatase in circulating neutrophilsB. Extreme splenomegalyC. Transition to acute leukemiaD. Philadelphia chromosomes in stem cellsE. * Transition to non-Hodgkin's lymphoma

14 Factor that provides for a definitive diagnosis of hemolytic anemia is which of the following:A. Red blood cell antibodiesB. *Red blood cell destructionC. Red blood cell enzyme deficiencyD. Bone marrow erythroid hyperplasiaE. Abnormal hemoglobin

15 A deficiency of the red cell membrane component spectrin causes which of the following diseases:A. Pernicious anemiaB. * Hereditary spherocytosisC. Sickle cell anemiaD. Thalassemia major E. Iron — deficiency anemiaE. -

16 Nonimmune hemolytic anemia occurs in patients with which of the following disease:A. Systemic lupus erythematosusB. * MalariaC. Chronic lymphocytic leukemiaD. Hodgkin's disease

E. ALL17 The disorder that most commonly causes iron deficiency anemia is which of the following:

A. Liver cirrhosisB. CardiomyopathyC. * Peptic ulcerD. PancreatitisE. Hemorrhagic stroke

18 The coagulation factor, reduced or absent in classical hemophilia is which of the following:A. IB. IIIC. *VIIID. X E. XIII

19 The similarity between tha-lassemia major and thalassemia minor is which of the following:A. Severity of anemiaB. Incidence of infectionC. Life spanD. * Familial occurrenceE. Homozygous form

20 Myelophthisic anemia can occur in patients with any of the following conditions, EXCEPT:A. Miliary tuberculosisB. CarcinomatosisC. MyelofibrosisD. Multiple myelomaE. * Uremia

21 Patients with polycythemia vera show all of the following characteristic features, EXCEPT:A. High platelet count (thrombocytosis)B. High hematocritC. *High erythropoietin levelD. High mean age at presentationE. Higher than average chance of stroke or myocardial infarction

22 All of the following etiologic associations with human acute leukemia have been proved valid, EXCEPT:A. *SmokingB. IrradiationC. AntibioticsD. Antineoplastic drugsE. Benzene compounds

23 Transformation to acute leukemia may occur during the course of all of the following disorders, EXCEPT:A. Polycythemia veraB. Chronic myelogenous leukemiaC. Chronic lymphocytic leukemiaD. *Hodgkin's diseaseE. Myelofibrosis

24 Thrombocytopenia is commonly caused by all of the following factors, EXCEPT:A. Folate deficiencyB. MyelofibrosisC. * Chronic lymphocytic leukemiaD. Marked splenomegalyE. Platelet sequestration

25 The diagnostic cell for Hodgkin's disease is which of the following:

A. LymphocyteB. "Foam cell"C. *Reed-Sternberg cellD. "Popcorncell"E. Macrophage

26 Subtypes of Hodgkin's disease include all of the following, EXCEPT:A. Nodular sclerosisB. Mixed cellularity subtypeC. Lymphocyte predominanceD. * Nodular amyloidosisE. Lymphocyte depletion

27 The diagnostic cell for multiple myeloma is which of the following:A. T-lymphocyteB. EosinophileC. Reed-Sternberg cellD. * Plasma cellE. "Popcorncell"

28 Multiple myeloma is characterized by the abnormal accumulation in tissues of which of the following substances:A. HyalineB. LipofuscinC. * AmyloidD. HemosiderinE. Melanin

29 Burkitt lymphoma is characterized by which of the following nistologic appearances:A. *"Starrysky" appearanceB. Infiltration of plasma cellsC. Presence of " foam cells "D. Presence of Reed-Sternberg cellsE. "Glassy " appearance of the cells

30 Mixed cellularity type of Hodgkin's disease is characterized by the presence of all of the following cells, EXCEPT:A. Reed-Sternberg cellsB. LymphocytesC. Plasma cellsD. MacrophagesE. *"Heart failure " cells

31 Acute rheumatic carditis is characterized by all of the following, except: A. Unremarkable gross appearance of the heart B. The Aschoff bodies in myocardium C. Diffuse nonspecific myocarditis D. The Aschoff bodies in joints E. *"Tigered effect" gross appearance

32 All of the following are stages of the atherosclerotic plaque development, except: A. *Fibrinous plaque B. Fibrous plaque C. Fibrofatty plaque D. Atheromatous plaque E. Fibrolipid plaque

33 All of the following components can be found in the necrotic center of atheromatous plaque, except: A. Cell debris

B. Cholesterol crystals C. "Foam" cells D. *Glycogen granules E. Calcium crystals

34 All of the following factors may contribute to vasoconstriction, except: A. *Acute coagulation factor B. Behavioral factors C. Neurogenic factors D. Increased release of vasoconstric-tive agents E. Increased sensitivity of vascular smooth muscles to constricting agent

35 Aschoff giant cells are large mesenchymal cells and are believed to be derived from which of the following: A. *Macrophages B. Fibroblasts C. Lymphocytes D. Eosinophils E. Erythrocytes

36 Atherosclerosis commonly affects the arteries supplying all of the following organs, except: A. *Upper extremities B. Lower extremities C. Heart D. Brain E. Kidney

37 Blood vessels affected in atherosclerosis are which of the following: A. Aorta only B. *Aorta and large arteries C. Large arteries and large veins D. Large veins E. Aorta and arterioli

38 Blood vessels affected in systemic hypertension are which of the folowing: A. *Arterioli B. Aorta and large arteries only C. Aorta only D. Large arteries and large veins E. Venules

39 Changes in the aging heart are all of the following, except: A. Decreased left ventricular cavity size B. Epicardial coronary artery tortuosity C. Decreased subepicardial fat D. Brown atrophy of myocardium E. *Increased number of inflammatory cells in myocardium

40 Cholesterol-laden macrophages and smooth muscle cells in atheromatous plaque are also referred to as: A. Giant cells B. "Popcorn" -cells C. Epithelioid cells D. Vacuolated cells E. *"Foam" cells

41 Coronary artery supplying blood to the anterior part of the inter-ventricular septum is which of the following: A. Left main coronary artery B. Left anterior descending artery

C. *Right coronary artery D. Left circumflex coronary artery E. Proximal marginal coronary artery

42 Etiologically and pathogenetically, rheumatic fever and rheumatic heart disease are characterized by all of the following, except: A. Initial attack of illness some weeks after streptococcal infection B. Elevated serum titers of antibodies to streptolysin and hyaluronidase C. Sterile tissue lesions not resulting from direct bacterial invasion D. Recurrent acute illness following the streptococcal infection E. *Decreased serum protein levels

43 Fibrosis bridging across the valvular commissures and calcification in chronic rheumatic heart disease cause which of the following: A. *Fish mouth or buttonhole stenosis B. "Tigered effect" C. Fibrofatty atheromas D. Starry sky appearance

44 Heart lesions in acute rheumatic fever are called as which of the following: A. Tuberculoma B. *Aschoff bodies C. Foreign bodies D. Gumma E. Mitral stenosis

45 Infective endocarditis is characterized by all of the following, except: A. Microbic invasion into the heart valves B. Destructive vegetation formation C. Mural thrombosis D. Destruction of the underlying cardiac tissues E. *Amyloid mass deposition

46 Coronary artery suppying blood to the posterior portion of the interventricular septum is which of the following: A. Left main coronary artery B. Left anterior descending artery C. *Right coronary artery D. Left circumflex coronary artery E. Proximal marginal coronary artery

47 Material accumulated in macrophages and smooth muscle cells in atherosclerotic plaque is which of the following: A. Glycogen B. Lipohyaline C. Triglycerides D. *Cholesterol or cholesterol esters E. Lipoproteids

48 Material deposited in arteriolar walls in systemic hypertension is which of the following: A. Amyloid B. Glycogen C. Cholesterol or cholesterol esters D. Lipoprotein E. *Hyaline

49 Microscopically, chronic rheumatic heart disease is characterized by all of the following, except: A. Diffuse fibrosis of leaflets B. Neovascularization of leaflets

C. *Acute purulent inflammation D. Destruction of original leaflet architecture E. Aschoff bodies in perivascular connective tissue

50 Microscopically, most cases of acute myocarditis are characterized by all of th following, except: A. Nonspecific inflammatory infiltrate B. Variable amount of interstitial edema C. Variable degree of myofiber degeneration D. *Marked fibrosis of interstitium E. Variable degree of myofiber necrosis

51 Microscopycally the vegetations of subacute infective endocarditis are characterized by all of the following, except: A. Fibrosis B. Microbic body calcifications C. Chronic inflammatory infiltrate D. *Granuloma formation E. Granulation tissue at vegetation's bases

52 Nonembolic complications of infective endocarditis include all of the following, except; A. Valvular insufficiency or stenosis with cardiac failure B. Myocardial ring abscess C. Suppurative pericarditis D. Endocarditis of artificial valves E. *Acute myocardial infarction

53 Nonbacterial thrombotic endocarditis is characterized by all of the following, except: A. Mitral and/or aortic valve involvement B. *Tricuspid valve involvement C. Small (A to E mm), sterile vegetations D. Vegetations along the line of valve closure E. Systemic emboli development

54 On gross examination, the kidneys in systemic chronic hypertension are characterized by all of the following features, except: A. Reduced in size and weight B. Fine granularity on cortical surface C. Grain leather-like cortical surface D. *Hemorrhages on cortical surface E. Narrowed cortical surface on section

55 Resulting from left-sided heart failure are all of these pathologic changes, except: A. Brown induration of the lung B. Hypoxic encephalopathy C. Pulmonary edema D. Prerenal azotemia E. *Secondary amyloidosis

56 Secondary hypertension may be caused by all of the following diseases, except: A. Acute glomerulonephritis B. *Lobar pneumonia C. Gushing's syndrome D. Pheochromocytoma E. Coarctation of aorta

57 Symptoms of cardiac dysfunction include all of the following, except: A. Failure of the pump itself B. *Disorders of blood coagulation C. Obstruction of blood flow

D. Regurgitant blood flow E. Disorders of cardiac conduction

58 Systemic emboli in infective endocarditis may occur because of all of the following, except: A. Involvement of aortic and mitral valves B. Severe valvular damage C. Numerous amount of vegetations D. Friable nature of vegetations E. *Chronic heart insufficiency

59 Systemic embolism in infective endocarditis may cause infarcts in all of the following organs, except: A. Brain B. Kidneys C. Heart D. *Lungs E. Spleen

60 The Anitschkow cells are characterized by all of the following, except: A. Abundant amorphophilic cytoplasm B. Central round-to-ovoid nucleus C. Central disposed chromatin (caterpillar cells) D. Monocytic cytogenesis E. *B-lymphocytic cytogenesis

61 The antihypertensive substances produced in the kidney are all of the following, except: A. Prostaglandines B. Urinary kallikrein-kinin system C. Platelet-activating factor D. Nitric oxide E. *Thromboxane

62 The Aschoff bodies consist of all of the following, except: A. Lymphocytes (primarily T cells) B. Anitschkow cells C. *"Foam cells" D. Fibrinoid degeneration E. Plasma cells

63 The cardinal anatomic changes of the mitral (or tricuspid) valve in chronic rheumatic heart disease are all of the following, except: A. *Leaflet softening B. Leaflet thickening C. Commisural fusion D. Shortening and thickening of the tendinous cords E. Fusion of the tendinous cords

64 The cells accumulating cholesterol or cholesterol esters in atherosclerotic plaque are which of the following: A. Neutrophils B. Lymphocytes and macrophages C. *Macrophages and smooth muscle cells D. Mesenchymal cells E. Giant cells.

65 The chronic endothelial injury in atherosclerosis may be caused by all of the following factors, except: A. Hyperlipidemia B. Hypertension C. Smoking

D. *Genetic defects E. Immune reactions

66 The chronic endothelial injury in atherosclerosis includes all pathologic processes, except: A. Increased endothelial permeability B. Adhesion of blood monocytes to the endothelium C. Adhesion of platelets to the intima D. Insudation of lipoproteins into the vessel wall E. *Regeneration of endothelial cells

67 The clinical consequences of valvular dysfunction depend on all of the following, except; A. Valve involved B. Sex of the patient C. Degree of impairment D. Rate of dysfunction development E. Rate and quality of compensatory mechanisms

68 The clinical syndrome of malignant hypertension includes all ofthe following symptoms, except: A. *Systolic pressure >120 mmHg B. Diastolic pressure >120 mmHg C. Renal failure D. Retinal hemorrhages E. Papilledema

69 The complications of atheromatous plague are all of the following, except: A. Thrombus formation B. Plaque rupture C. *Plague malformation D. Calcification E. Hemorrhage

70 The consequences of atherosclerosis are all of the following, except: A. Myocardial infarction B. *Cardiomyopathy C. Cerebral infarct D. Gangrene of lower extremities E. Abdominal aortic aneurism

71 The correct order of histologic phases in the development of Aschoff bodies is which of the following: A. *Exudative — proliferative — fibrous B. Granulomatous — late — exudative C. Acute — chronic — healed D. Nonspecific — specific E. Early — pathognomonic — intermediate

72 The disease that is characterized by the systolic murmur without signs of mitral valve failure is called as: A. Rheumatic heart disease B. *Mitral valve prolapse C. Mitral annular calcification D. Degenerative calcific aortic valve stenosis E. Congenitally bicuspid aortic valve

73 The fibrous cap of atheromatous plaque consists of all of the following components, except: A. *Glycogen B. Smooth muscle cells C. Macrophages D. Collagen and elastan

E. Proteoglycans 74 The gene defects in enzymes involved in aldosterone metabolism in systemic hypertension

lead to all of the following effects, except: A. Adaptive increase in aldosterone secretion B. Increased salt resorption C. Increased water resorption D. *Increased lipoprotein resorption E. Plasma volume expansion

75 The humoral constriction factors that lead to the peripheral resistence in hypertension are all of the following, except: A. Angiotensin II B. Catecholamines C. *Prostaglandines D. Thromboxanes E. Leukotriens

76 The lowered risk of ischemic heart disease in atherosclerosis depends on which of the following: A. High serum level of low-density lipoproteins B. *High serum level of high-density lipoproteins C. High serum level of very low density lipoproteins D. High serum-level of intermediate-density lipoproteins E. High serum level of triglycerides

77 The major nonmodifiable risk factors for atherosclerosis are all of the following, except: A. Increasing age B. *Increased water resorption C. Male gender D. Family history E. Genetic abnormalities

78 The malignant hypertension can be identified microscopically by all of the following features, except: A. Onion-skin concentric laminated thickening of the arteriolar walls B. Progressive narrowing of the arteriolar lumina C. *Cholesterol-laden "foam cells" in the arteriolar walls D. Fibrinoid necrosis of the arteriolar walls E. Perivascular fibrosis

79 The mitral annular calcification is characterized by all of the following, except: A. *Severely impaired valvular function B. Irregular, stony hard nodule appearance C. Possible thromboembolic complications D. Infective endocarditis E. Absence of inflammatory changes

80 The morphological and clinical effects of pure (isolated) right-sided heart failure are all of following, except: A. *Cerebral hematoma B. Congestive hepatomegaly C. Ascites D. Pleural and pericardial effusions E. Anasarca

81 The nonrheumatic degenerative calcific aortic stenosis is characterized by all of the following, except: A. Heaped-up calcified masses within aortic cusps B. *Lipid deposition and cellular proliferation

C. Distorted cuspal architecture D. Not involved free cuspal edges E. Primary involvement of the valvular fibrosa

82 The pathologic process in arterioles in malignant hypertension is referred to as: A. Endothrombovasculitis B. Thromboangitis obliterans C. Necrotizing atherosclerosis D. Productive arteriolitis E. *Necrotizing arteriolitis

83 The potentially controllable risk factors for atherosclerosis are all of the following, except: A. Hyperlipidemia B. Hypertension C. *Genetic abnormalities D. Cigarette smoking E. Diabetes mellitus

84 The renin-angiotensin system consists of all of the following components, except: A. Renin B. Angiotensin I C. Plasma angiotensinogen D. *Tissue angiotensinogen E. Angiotensin II

85 The special stain used to identify "foam cells" in atherosclerotic plaque is which of the following: A. *SudanIII B. Picrofuchsin C. Hematoxylin and eosin D. Congo red stain E. PAS-reaction

86 The typical pathologic process in arterioles and small arteries in systemic hypertension is referred to as: A. Hyaline atrophy B. Hyaline hyperplasia C. Fibroelastic hyalinosis D. Hyaline atherosclerosis E. *Hyaline arteriolosclerosis

87 The valvular effects of chronic rheumatic heart disease are all of the following, EXCEPT: A. Thickened and deformed valve leaflets B. Valvular calcification C. Thickened and shortened chordae tendineae D. "Fish-mouth" appearance of mitral valve E. *Valvular abscess formation

88 Valvular insufficiency without primary changes in the cusps may result from distortion of all of the following structures, except: A. *Parietal endocardium B. Tendinous cords C. Aorta or mitral annulus D. Papillary muscles E. Ventricular free wall

89 Rheumatic diseases - a group of chronic diseases characterized by systemic lesion: A. *connective tissue and blood vessels B. joints C. the circulatory system

D. cellE. Kidney

90 What is rheumatic disease - (a group of chronic diseases characterized by systemic lesion): A. *connective tissue and blood vessels B. joints C. the circulatory system D. cell E. kidney

91 In the etiology of rheumatic diseases is essential clinically expressed or latent: A. *streptococcal infection B. Meningococcal infection C. diplokoccal infection D. fungal infections E. all infections

92 Pathogenic mechanisms of rheumatic diseases: A. *allergic reactions of immediate and delayed-type B. streptococcal septicemia C. shockD. staphylococcus septicopyemiaE. gipertermical reactions

93 To belong to pathogenetic mechanisms of rheumatic diseases: A. *allergic reactions of immediate and delayed-type B. streptococcal septicemia C. shockD. staphylococcus septicopyemia E. gipertermical reactions

94 In rheumatic diseases the main pathogenetic plot are: A. *progressive disorganization of the connective tissue B. stagnant around the plethora of small joints C. ischemia D. Hypertrophy of left heart E. atrophy

95 In the pathogenesis of rheumatic diseases is: A. atrophyB. *progressive disorganization of the connective tissue C. stagnant around the plethora of small joints D. ischemia E. Hypertrophy of left heart

96 In the pathogenesis of rheumatic diseases is: A. atrophyB. *progressive disorganization of the connective tissue C. stagnant around the plethora of small joints D. ischemia E. hypertrophy of left heart

97 Disturbance of connective tissue in rheumatic diseases includes: A. atrophyB. hyaline droplets dystrophy C. *mucoid swelling D. amyloidosis E. hydropic dystrophy

98 What includes the disturbance of connective tissue in rheumatic diseases: A. atrophy

B. hyaline droplets dystrophy C. *mucoid swellingD. amyloidosis E. hydropic dystrophy

99 Disturbance of connective tissue in rheumatic diseases includes: A. *fibrinoid swelling and necrosis B. hyaline droplets dystrophy C. amyloidosis D. hydropic dystrophy E. atrophy

100 What type of dystrophy involves the disruption of connective tissue in rheumatic diseases: A. necrosisB. *fibrinoid edema and necrosis C. hyaline droplets dystrophy D. amyloidosis E. hydropic dystrophy

101 Disturbance of connective tissue in rheumatic diseases includes: A. *Hyalinosis B. cell reaction (granulematosis) and sclerosis C. hyaline droplets dystrophy D. amyloidosis E. hydropic dystrophy

102 In rheumatism sensitizing factor antibodies:A. *B-hemolytic streptococcus typeАB. B-hemolytic streptococcus, type BC. B-hemolytic streptococcus type ABD. B-hemolytic staphylococcus type DE. B-hemolytic staphylococcus type C

103 What antibodies are sensitizing factor in rheumatic conditions: A. B-hemolytic streptococcus, type BB. B-hemolytic streptococcus type AB C. *B-hemolytic staphylococcus type A D. B-hemolytic streptococcus type D E. B-hemolytic staphylococcus type C

104 B-hemolytic streptococci type A has affinity with antigens аntibodies againstA. *connective tissue of heart B. liverC. kidney D. tonsils E. connective tissue of mediastinum

105 What kind of relationship with the antigens are B-hemolytic streptococci type A: A. kidney B. tonsils C. connective tissue of mediastinum D. *connective tissue of heart E. liver

106 Kinship antibodies against B-hemolytic streptococci type A antigens from connective tissue of the heart explains the predominant lesion in patients with rheumatism:A. *heart B. liverC. only epikard

D. only endocard E. more pericardiocentesis

107 What explains the predominant lesion in patients with rheumatism affinity antibodies against B-hemolytic streptococci type A antigens from connective tissue of the heart: A. liver B. *heart C. only epikard D. only endocardE. more pericardiocentesis

108 In rheumatoid arthritis in patients with damage mainly to the connective tissue: A. *capsule of the joints B. heart C. kidney D. renal capsule E. liver

109 Connective tissue structures are damaged in rheumatoid arthritis: A. heart B. kidney C. renal capsule D. liverE. *capsule of the joints

110 In the pathogenesis of rheumatoid arthritis are important: A. *immune complexes, where the antibodies are different classes of immunoglobulins (Ig

M, Ig G, Ig A) B. -- C. immune complexes, where there are different classes of immunoglobulins (Ig N, Ig Q, Ig

B) D. complexes, where there are different classes of immunoglobulins (Ig Y, Ig H, Ig L) E. immune complexes, where the antigen is a globulin of different classes (Ig M, Ig G, Ig A)

111 What have an important role in the pathogenesis of rheumatoid arthritis: A. immune complexes, where the antigen is a globulin of different classes (Ig M, Ig G, Ig A)B. *immune complexes, where the antibodies are different classes of immunoglobulin (Ig

M, Ig G, Ig A) C. immune complexes, where the antigen is a globule of different classes (Ig M, Ig G, Ig A)D. immune complexes, where there are different classes of immunoglobulins (Ig N, Ig Q, Ig

B) E. complexes, where there are different classes of immunoglobulins (Ig Y, Ig H, Ig L)

112 In systemic lupus erythematosus is violated:A. exchange of DNA and antigens to components formed the nucleus and cytoplasm B. *exchange of DNA and formation of antibodies to components of the nucleus and

cytoplasm C. exchange of RNA and formation of antibodies to components of the nucleus and

cytoplasm D. exchange of RNA and formation of antibodies E. exchange of DNA and antigens to components formed the nucleus and cytoplasm

113 Violations in systemic lupus erythematosus: A. exchange of DNA and antigens to components formed the nucleus and cytoplasm B. *exchange of DNA and formation of antibodies to components of the nucleus and

cytoplasm C. exchange of RNA and formation of antibodies to components of the nucleus and

cytoplasm D. exchange of RNA and formation of antibodies

E. exchange of DNA and antigens to components formed the nucleus and cytoplasm 114 Systemic scleroderma is characterized in the skin:

A. violation of vascularization B. *sclerotic and atrophic changes C. inflammation D. calcification E. violation of vascularization

115 What changes in the skin are identified with systemic scleroderma: A. violation of vascularization B. *sclerotic and atrophic changes C. inflammation D. calcificationE. violation of vascularization

116 The decisive factor for the development of scleroderma is a violation of the synthesis: A. keloid B. *collagen C. fibrin D. fibrinogen E. keloid

117 What is the decisive factor for the development of scleroderma - a violation of the synthesis: A. keloid B. *collagen C. fibrin D. fibrinogen E. keloid

118 For nodular periarteritis characterized mechanism immunokomplex vascular lesions such as: A. arterioles- capillaries anastomosis B. * small artery C. aorta D. large artery E. arterioles-capillary anastomosis

119 What affects the blood vessels in the nodular periarteritis: A. large-caliber artery B. arterioles-capillaries anastomosis C. *artery and small-medium caliber D. aorta E. large-caliber artery

120 It is believed that with nodular periarteritis due to fibrinous necrosis secondary hull vessels develop proliferative response of cells in the outer shell, followed by transfer to: A. granulomaB. *sclerosis and the formation of knots C. hyalinosis D. amyloidosis E. inflammation

121 In nodular periarteritis due to fibrinous necrosis secondary hull vessels develop proliferative response of cells in the outer shell, followed by transfer to: A. granulomaB. *sclerosis and the formation of knots C. hyalinosis D. amyloidosis

E. inflammation122 Bekhterev disease - a chronic rheumatic disease with the damage:

A. *glenoid spine-connected device B. articular apparatus C. large joints D. small and large joints E. articular apparatus

123 What kind of surprised when the disease Bekhterev -- A. *glenoid spine-connected device B. articular apparatus C. large joints D. small and large joints E. articular apparatus

124 As a result, the disease develops Bekhterev:A. Osteoporosis B. *bone ankylosis C. bone amyloidosis D. osteopetrosis E. Osteoporosis

125 Complications of the disease Bekhterev -- A. *bone ankylosis B. bone amyloidosis C. osteonecrosis D. osteoporosis E. necrosis

126 Dermatomyositis - rheumatic diseases, leading to the clinical and morphological expression of which is a system failure: A. *skeletal muscles B. skin C. dermis D. only the smooth muscle E. kidney

127 What is leading to the clinical and morphological expression, with dermatomyositis: A. System failure only of smooth muscle B. *System failure skeletal muscles C. Systemic dermal D. System failure dermis E. System failure only of smooth muscle

128 What kind of surprised when dermatomyositis: A. *skeletal musculature B. skin C. derm D. only the smooth muscle E. kidney

129 Indicate which affects the structure with dermatomyositis: A. *skeletal musculature B. skin C. dermis D. only the smooth muscle E. kindey

130 Rheumatism - a chronic disease with predominant lesion: A. *heart and vascular

B. kidney and liver C. arteries of medium caliber D. muscular arteries, elastic-type E. liver

131 In rheumatism predominantly affects: A. *heart and blood vessels B. kidney and liver C. medium-sized artery D. artery muscle-elastic type of E. liver

132 Determination of rheumatism - a chronic disease with predominant lesion: A. *heart and vascular B. kidney and liver C. arteries of medium caliber D. muscular arteries, elastic-type E. kidney and liver

133 In rheumatism periods alternate with periods of exacerbation of extinction (remission), so during rheumatism: A. remitting B. nonhomogeneous C. homogeneousD. *wave E. repeatedly

134 In rheumatism streptococci penetrate the body through the tonsils, release toxins and cause destruction in the invasion of cells and inflammation, which usually manifests itself: A. *angina B. tonzilyar abscesses C. retropharyngeal abscesses D. Qatar nasopharyngeal E. abscesses

135 Because rheumatic conditions streptococci penetrate the body through the tonsils, release toxins and cause destruction in the invasion of cells and inflammation, which usually manifests itself: A. *angina B. tonzilyar abscesses C. retropharyngeal abscesses D. Qatar nasopharyngeal E. abscesses

136 In rheumatism streptococci penetrate the body through the tonsils, release toxins, which are products of the destruction of cells is: A. *activated antibodies B. immune complexes C. antibodies, which produce antigens D. аntigens, that produce antibodies E. leukocytes

137 What structures are formed by the interaction of streptococcus toxins and products of destruction of cells in the amygdala rheumatic conditions: A. activated antibodies B. immune complexes C. antibodies, which produce antigens D. *antigens, that produce antibodies E. activated antibodies

138 In the development of rheumatism occurs mukoid edema, which is: A. *Surface and reverse disruption of connective tissue B. irreversible disruption of connective tissue C. Surface restructuring of the connective tissue D. reverse proliferation of connective tissue E. Surface restructuring of the connective tissue

139 Which involves mukoid swelling that develops rheumatic conditions: A. *surface reverse disruption of connective tissue B. irreversible disruption of connective tissue C. surface reorganization of connective tissue D. reverse the proliferation of connective tissue E. irreversible disruption of connective tissue

140 Early diagnosis and early treatment of rheumatism: A. *can lead to complete recovery B. can never lead to a cure C. can not lead to a reorganization of the connective tissue D. can not lead to the restitution of connective tissue E. can not lead to a reorganization of the connective tissue

141 As a result of early diagnosis and early treatment of rheumatism are: A. *Full recovery B. can never lead to recovery C. can not lead to a reorganization of the connective tissue D. can not lead to the restitution of connective tissue E. can never lead to a cure

142 .In the development of rheumatism among the stages of disorganization of connective tissue are fibrinoid changes as swelling and necrosis, refer them to: A. *irreversible B. reversible C. partially reversible D. border E. reversible

143 Phase of disorganization of connective tissue such as fibrinoid changes, swelling and necrosis, refer them to: A. *irreversible B. reversible C. partially reversible D. border E. reversible

144 Fibrinoidnye changes (swelling and necrosis) in the rheumatic conditions are characterized by: A. *homogenization of collagen fibers with infiltration of plasma proteins B. homogenization of nerve fibers with infiltration their proteins C. homogenization keloidnyh fibers with infiltration their proteins D. granulyatsionnoy homogenization of tissue with infiltration plasma proteins E. homogenization keloid fibers with infiltration their proteins

145 What are fibrinoid changes (swelling and necrosis), rheumatic conditions: A. *homogenization of collagen fibers with prosyakannyam of plasma proteins B. homogenization of nerve fibers with prosyakannyam their proteins C. homogenization keloidnyh fibers with prosyakannyam their proteins D. granulyatsionnoy homogenization of tissue with prosyakannyam plasma proteins E. homogenization keloidnyh fibers with prosyakannyam their proteins

146 . Stage granulematous rheumatic conditions morphologically evident:

A. *Cell incendiary reaction B. Monokin reaction C. cytokine response D. polymorphonuclear cell inflammation of the reorganization E. --

147 What stage is morphologically manifested granulematous rheumatic conditions: A. *Cell incendiary reaction B. Monokin reaction C. cytokine response D. polymorphonuclear cell inflammation of the reorganization E. --

148 The first stage granulematous in the form of nodular formations in the stroma of heart described: A. *Ashof B. Bischoff C. Amosov D. Langerhans E. Amosov

149 Who first described the stage granulematous in the form of nodular formations in the stroma of heart: A. *Ashof B. Bischoff C. Amosov D. Langerhans E. Amosov

150 Three phases of development of rheumatic granulomas – alteration , exudation proliferations and sclerotic identified: A. *Talalaev B. Bischoff C. Amosov D. Ashof E. Amosov

151 Who first identified three phases of development of rheumatic granulomas – alteration? Exudation and proliferation and sclerotic: A. Amosov B. *Talalaev C. Bischoff D. Amosov E. Ashtof

152 The entire development cycle granuloma rheumatic conditions are: A. *D-6 months B. A year C. A-B years D. 9-AB months E. B years

153 The entire development cycle granuloma rheumatic conditions are: A. 9-AB Months B. C years C. *D-6 months D. A year E. A-B years

154 Specify how much of the entire development cycle granuloma rheumatic conditions:

A. 9-AB Months B. E years C. *D-6 months D. A year E. A-B years

155 Alterative and exudative phase of development characterized by the accumulation of rheumatic granuloma: A. * macrophages around the focus fibrinoid necrosis B. lymphocytes and the formation of follicles C. lymphocytes and the formation of follicles in breeding centers D. neutrophils, lymphocytes, histiocytes E. neutrophils, lymphocytes, histiocytes

156 What is alteration, axudative phase development of rheumatoid granuloma: A. accumulation of neutrophils, lymphocytes, histiocytes B. accumulation of lymphocytes and the formation of follicles in breeding centers C. *a macrophages ccumulation around the focus fibrinoid necrosis D. accumulation of lymphocytes and the formation of follicles E. accumulation of lymphocytes and the formation of follicles in breeding centers

157 Indicate what is alteration and exudation phase development of rheumatoid granulomy: A. accumulation of neutrophils, lymphocytes, histiocytes B. accumulation of neutrophils,C. *accumulation around the campfire fibrinoidnogo necrosis of macrophages D. accumulation of lymphocytes and the formation of follicles E. accumulation of lymphocytes and the formation of follicles in breeding centers

158 The presence of «sclerosing granuloma» refers to: A. *fading process B. exacerbation of the process after remission C. active attack of rheumatism D. recovery process after an active attack of rheumatism E. slight slowdown in the pace of development

159 At that indicates the existence of «sclerosing granuloma»: A. active attack of rheumatism B. *fading process C. exacerbation of the process after remission D. recovery process after an active attack of rheumatism E. slight slowdown in the pace of development

160 With the development of rheumatoid granulomy phase sclerosis fibroblasts replace the necrotic zone fibrinoid are synthesized: A. * argyrophil and collagen fibers B. mukoid swellingC. fibrinoid swelling D. hyalinosis E. amyloidosis

161 With the development of rheumatoid granuloma phase sclerosis fibroblasts replace the necrotic zone fibrinoid, are synthesized: A. * argyrophil and collagen fibers B. Mukoid swelling C. Fibrinoid swelling D. Hyalinosis E. amyloidosis

162 With the development of rheumatoid granuloma phase sclerosis granuloma recruits properties scar, a sign indicates:

A. *remission of disease B. Acute periodC. escalation process after remission D. rebuilding process after the strong attack of rheumatism E. slight slowdown in the pace of development

163 What period of the disease shows the development of rheumatoid granuloma in the phase of MS: A. acute periodB. *remission of disease C. escalation process after remission D. rebuilding process after the strong attack of rheumatism E. slight slowdown in the pace of development

164 In a typical course of rheumatism in the first place is damaged: A. *heart B. only the large joints C. only small joints D. Only kidney E. liver

165 Which body is damaged in the first place, with a typical current of rheumatism: A. *heart B. only the large joints C. only small joints D. Only kidney E. liver

166 In a typical course of rheumatism in the first damaged heart, which develops a pathological process, such as: A. *endocarditis and myocarditis B. endovaskulitis coronary arteries C. pericarditis and pankarditis D. perivaskulitis coronary arteries E. cardiosclerosis

167 . What pathologic process develops in the heart, with a typical current of rheumatism: A. *endocarditis and myocarditis B. endovaskulitis coronary arteries C. pericarditis and pankarditis D. perivasculitis of coronary arteries E. cardiosclerosis

168 Endocarditis rheumatic conditions on the localization can be: A. *valvular B. mural C. transmural D. epicardial E. pericardial

169 What endocarditis rheumatic conditions may be to localize: A. *valvular B. mural C. transmural D. epicardialE. pericardial

170 Endocarditis (inflammation endocardium) rheumatic conditions on the localization can be:A. *chordal

B. mural C. transmural D. epicardial E. pericardial

171 How to localize endocarditis (inflammation of endocardium) rheumatic conditions: A. *chordal B. mural C. transmural D. epicardial E. pericardial

172 . Endocarditis (inflammation of endocardium) rheumatic conditions on the localization can be:A. *subendocardialB. mural C. transmural D. epicardial E. pericardial

173 How to localize endocarditis (inflammation of endocardium) may be rheumatic conditions: A. *subendocardial B. mural C. transmural D. epicardialE. pericardial

174 Often the rheumatoid process affects: A. *mitral and aortic valves B. tricuspid valve C. valve pulmonary artery D. valve and the pulmonary artery and tricuspid valve E. --

175 What most amazes rheumatic process: A. *mitral and aortic valves B. tricuspid valve C. valve pulmonary artery D. valve and the pulmonary artery valve tricuspid E. --

176 What kind-hearted often surprised when rheumatic process: A. *mitral and aortic valve B. tricuspid valve C. valve pulmonary artery D. valve and the pulmonary artery tricuspid valveE. --

177 Depending on the prevalence alterative or regenerative processes distinguish the type of rheumatic valve endocarditis: A. *diffuse endocarditis B. mural endocarditis C. transmural endocarditis D. epicardial endocarditis E. pericarditis

178 Depending on the prevalence alterative or regenerative processes distinguish the type of rheumatic valve endocarditis: A. *diffuse endocarditis

B. mural endocarditis C. transmural endocarditis D. epicardial endocarditis E. pericarditis

179 Diffuse endocarditis as a kind of rheumatic valve endocarditis is characterized by diffuse: A. *mucoid swelling of connective tissue without damaging the endothelium B. fibrinoid necrosis of connective tissue and endothelial C. fibrinoid edema of connective tissue and endothelial damage D. hyalinosis of connective tissue and endothelial E. amyloidosis

180 What is diffuse endocarditis as a kind of rheumatic valve endocarditis: A. -- B. *mukoid edema of connective tissue without damaging the endothelium C. fibrinoid necrosis of connective tissue and endothelial D. fibrinoid edema of connective tissue and endothelial damage E. hyalinosis of connective tissue and endothelial

181 Depending on the prevalence alterative or regenerative processes distinguish the type of rheumatic valve endocarditis: A. *warty acute endocarditis B. Acute mural endocarditis C. transmural subacute endocarditis D. Acute endocarditis E. --

182 What types of rheumatic valve endocarditis differ depending on the prevalence alterative or regenerative processes: A. *warty acute endocarditis B. Acute endocarditisC. Acute mural endocarditis D. transmural subacute endocarditis E. Acute endocarditis

183 Acute papillose endocarditis rheumatic conditions is accompanied by: A. *fibrinoid changes in connective tissue and endothelial desquamation with delay in the

sites of injury trombotic masses in the form of warts B. mucoid changes in connective tissue with a delay trombotic mass in the form of warts C. -- D. fibrinotic changes in connective tissue and endothelium of the postponement of the

masses of warts E. fibrinoid changes in cardiomyocytes and desquamation epicardium with postponement of

warts184 What is accompanied by a sharp papillose endocarditis rheumatic conditions:

A. *fibrinoid changes in connective tissue and endothelial desquamation with delay in the sites of injury trombotic masses in the form of warts

B. mukoid changes in connective tissue with a delay trombotic mass in the form of warts C. -- D. fibrinoid changes in connective tissue and endothelium of the masses in the form of

postponement of warts E. fibrinoid changes in cardiomyocytes and desquamation epicardium with postponement of

warts185 Depending on the prevalence alterative or regenerative processes distinguish the type of

rheumatic valve endocarditis: A. *fibroplastic endocarditis

B. intramural endocarditis C. -- D. transmural endocarditis E. epicardial endocarditis

186 Specify the type of rheumatic valve endocarditis depending on the prevalence alterative or regenerative processes: A. --B. *fibroplastic endocarditis C. mural endocarditis D. transmural endocarditis E. epicardial endocarditis

187 Depending on the prevalence alterative or regenerative processes distinguish the type of rheumatic valve endocarditis - fibroplastichny endocarditis, characterized by: A. *growth of new tissue, scar revisions and valve insufficiency (acquired heart disease) B. -- C. mucoid changes in connective tissue with a delay trombotic mass in the form of warts on

the valves D. fibrinogen changes in connective tissue valves E. fibrinoid changes in cardiomyocytes and endothelial

188 Depending on the prevalence alterative or regenerative processes distinguish the type of rheumatic valve endocarditis: A. *rotary-papillose endocarditis B. re endocarditis C. re-trombotic endocarditis D. diffuse-papillose endocarditis E. --

189 Rotary-papillose endocarditis is characterized by repeated disruptions: A. *newly formed connective tissue, endothelial injury and fibrin accumulation on a

background of sclerosis and hyalinosis valve B. collagen tissue in the middle of the valve with the proliferation of endothelium C. -- D. fibrous tissue valve with diffuse deposition of fibrin E. intact endothelium and the postponement of fibrinogen

190 Indicate the degree of disorganization of connective tissue in rotary-papillose endocarditis: A. -- B. *neoformation of connective tissue, endothelial damage and fibrin deposition on the

background of sclerosis and hyalinosis valve C. collagen tissue in the middle of the valve with the proliferation of endothelial D. fibrous tissue valve with diffuse deposition of fibrin E. intact endothelium and delays fibrinogen

191 Myocarditis (inflammation of the myocardium) - permanent manifestation of rheumatic disease, a form of it are: A. *granulematozny B. -- C. focal parenchymatous D. diffuse ekssudative parenchymatous E. focal interstitial

192 Specify the form myocarditis (inflammation of the myocardium) - permanent manifestation of rheumatism: A. -- B. *granulematozny

C. focal parenchymatous D. diffuse exudative parenchymatous E. focal interstitial

193 . Granulematous myocarditis rheumatic conditions characterized by the presence of "blooming", and sclerotic rheumatic granulomas: A. *in the perivascular connective tissue of the myocardium B. in cardiomyocytes C. vascular endothelium in myocardial D. in epycardium E. --

194 . Specify the localization of "blooming", sclerotic granulomas rheumatic conditions: A. -- B. *in the perivascular connective tissue of the myocardium C. in cardiomyocytes D. vascular endothelium in myocardial E. in epicardium

195 Exudative diffuse interstitial myocarditis is characterized by: A. *interstitial significant infiltration of lymphocytes, gistiocyts, neutrophils and eosinophils

in the presence of single granulomas Ashof-Talalaev B. -- C. slight infiltration of lymphocytes intersticial, gistiocyts, neutrophils and eosinophils in

the presence of single granulomas Ashof-Talalaev D. interstitsiya significant infiltration of lymphocytes in the presence of single granulomas

Ashof-Talalaev E. slight infiltration of neutrophils in the presence of interstitial isolated granulomas Ashof-

Talalaev 196 What is exudative diffuse interstitial myocarditis:

A. *interstitial significant infiltration of lymphocytes, gistiocites, neutrophils and eosinophils in the presence of single granulomas Ashof-Talalaev

B. slight infiltration of lymphocytes interstitial, gistiocyts, neutrophils and eosinophils in the presence of single granulomas Ashof-Talalaev

C. interstitsiey significant infiltration of lymphocytes in the presence of single granulomas Ashof-Talalaev

D. -- E. slight infiltration of neutrophils in the presence of interstitial isolated granulomas Ashof-

Talalaev 197 Development of myocarditis cardiosclerosis ends with:

A. *favorable course of rheumatism B. -- C. disadvantaged during rheumatism D. in the frequent recurrence of rheumatism E. in rheumatism with high activity indices of blood

198 Cardiosclerosis is the result of miocarditis with: A. -- B. *favorable course of rheumatism C. disadvantaged during rheumatism D. During the frequent recurrence of rheumatism E. During rheumatism with high activity indices of blood

199 Microscopical features of acute appendicitis are all of the following, EXCEPT:A. *Epithelioid cell infiltrationB. Diffuse neutrophil infiltrationC. Absesses formation within the appendicular wall

D. Ulceration of the mucosaE. Foci of hemorrhages

200 The cell types in the corpus gastric glands are all of the following, EXCEPT:A. Mucous cellsB. Parietal cellsC. Chief cellsD. * Hurtle cellsE. Endocrine cells

201 The gastric mucosal protection from autodigestion is possible due to all of the following factors, EXCEPT:A. Mucus secretionB. Bicarbonate secretionC. *Gastrin receptor activationD. Epithelial barrierE. Mucosal blood flow

202 Chronic infection of the gastric mucosa is associated with which of the following bactertia:A. Escherichia coliB. *Helicobacter pyloriC. Campilobacter jejuniD. Enterococcus falciumE. Escherichia chaffeensis

203 Chronic gastritis may be characterized by all of the following features, EXCEPT:A. *Suppurative inflammationB. Lymphocyte infiltrationC. Plasma cell infiltrationD. Intestinal metaplasiaE. Atrophy

204 Acute gastritis is commonly caused by all of the following factors, EXCEPT:A. Excessive use of aspirinB. Heavy smokingC. Excessive alcohol consumptionD. Treatment with cancer chemotherapeutic drugsE. * Genetic abnormalities

205 Acute gastritis is commonly associated with all of the following factors, EXCEPT:A. UremiaB. Severe stressC. Systemic infectionsD. * Delayed gastric emptyingE. Ischemia and shock

206 Acute catarrhal gastritis is morphologically characterized by aU of the following features, EXCEPT:A. Moderate edema of lamina propriaB. Vascular congestion of lamina propriaC. * Atrophy of the mucosaD. Neutrophil infiltrationE. Abundant mucus amount on the gastric epithelium

207 The major etiologic associations of chronic gastritis are all of the following, EXCEPT:A. Chronic Helicobacter pylori infectionB. * Ischemia and shockC. Autommune factorsD. Toxic factors

E. Bile reflux208 The major etiologic associations of chronic gastritis are all of the following, EXCEPT:

A. *Salmonella infectionsB. Helicobacter pylori infectionC. Radiation injuryD. Granulomatous diseasesE. Autoimmune reactions

209 Special forms of gastritis are all of the following, EXCEPT:A. Eosinophilic gastritisB. Allergic gastroenteropathyC. Lymphocytic gastritisD. Granulomatous gastritisE. * Interstitial gastritis

210 The clinical syndrome associated with gastric, duodenal and jejunal peptic gastrin-induced ulcers development is which of the following:A. Horner syndromeB. Nephrotic syndromeC. DIG syndromeD. *Zollinger- Ellison syndromeE. Malabsorption syndrome

211 Morphologically, all of the following zones can be found in the active ulcer base, EXCEPT:A. Necrotic debrisB. Nonspecific acute inflammationC. * Granulomatous tissueD. Granulation tissueE. Fibrosis

212 The appendix in acute appendicitis is grossly characterized by all of the following features, EXCEPT:A. Enlarged with thickened wallsB. *Firm and induratedC. SwollenD. Hyperemic with engorged vesselsE. With fibrin covered serosa

213 Catarrhal gastritis is grossly characterized by all of the following features, EXCEPT:A. Thickened gastric wallsB. Swollen gastric mucosaC. * Multiple erosions on the mucosaD. Pinpoint hemorrhages on the mucosaE. Abundant viscuos mucus on the mucosa

214 All of the following complications may be found in chronic peptic ulcer, EXCEPT:A. *CaseationB. PerforationC. BleedingD. MalformationE. Penetration

215 The causes of malabsorption syndrome are all of the following, EXCEPT:A. Defective intraluminal digestionB. Primary mucosal cell abnormalitiesC. Reduced surface of small intestineD. Lymphatic obstructionE. *Small intestine diverticula

216 Pathologic changes of the intestinal wall in Crohn's disease include all of the following, EXCEPT:A. Granuloma formationB. Areas of chronic inflammation with vasculitisC. *Pseudomembranous colitisD. Segmental fibrosisE. Slit ulcers

217 The predisposing conditions for ischemic bowel disease are all of the following, EXCEPT:A. Arteriolar thrombosisB. Arteriolar embolismC. Venous thrombosisD. *Helicobacter pylori infectionE. Portal hypertension syndrome

218 Giant cerebriform enlargement of the gastric mucosa in Menetrier disease is caused by which of the following pathologic processes:A. InflammationB. *Hyperplasia of the mucosal epithelial cellsC. Atrophy of the mucosaD. Interstitial metaplasiaE. Fibrosis

219 Complications of the duodenal peptic ulcer disease include all of the following, EXCEPT:A. *MalabsorptionB. BleedingC. PerforationD. PenetrationE. Obstruction by edema or scarring

220 Cancerous ulcer margins are grossly characterized by all of the following features, EXCEPT:A. Greyish-whiteB. ElevatedC. FirmD. Irregular and thickened .E. * Relatively straight walls

221 The most common localization of peptic ulcer is which of the following:A. Greater curvatureB. Lesser curvatureC. * Duodenal pyloric ringD. Anterior wall of the gastric corpusE. Posterior wall of the gastric corpus

222 The classic peptic ulcer is grossly characterized by all of the following features. EXCEPT:A. Round-to-ovalB. Sharply definedC. Punchet-out defectD. Elevated marginsE. *Superficial defect

223 The inflammatory infiltrate in the remission stage of chronic gastritis may consist of all of the following cells, EXCEPT:A. LymphocytesB. *Neutrophils

C. Plasma cellsD. FibroblastsE. Macrophages

224 The granulation tissue in the active ulcer base is infiltrated predominantly by all of the following cells, EXCEPT:A. NeutrophilsB. LymphocytesC. Plasma cellsD. * Giant cellsE. Macrophages

225 .The most common pathologic change of uninvolved gastrointestinal tract in peptic ulcer patients is which of the following:A. Cancerous ulcerB. Hypertrophic gastropathyC. Gastric dilatationD. Menetrier diseaseE. * Chronic gastritis

226 Bleeding from peptic ulcer is characterized by all of the following, EXCEPT:A. Occurs in 15-20% of patientsB. Is most frequent complication of peptic ulcerC. May be life-threateningD. * Causes chronic renal failureE. Causes 25% of ulcer deaths

227 Gastrointestinal tract obstruction from edema or scarring in peptic ulcer disease is characterized by all of the following, EXCEPT:A. Occurs in 2% of patientsB. Often occurs in pyloric channel ulcerC. May occur in duodenal ulcerD. *May occur in the stomach corpusE. Causes incapacitating, crampy abdominal pain

228 Multiple superficial lesions located in the gastric mucosa are called:A. Peptic ulcersB. * ErosionsC. Acute stress ulcersD. Gastric ulcerationsE. Active ulcers

229 Point out the hepatic cells that participate in the storage and metabolism of vitamin A:A. HepatocytesB. *Stellate cells (Ito cells)C. Kupfer cellsD. Endothelial cellsE. Hepatic fibroblasts

230 Microfilaments surrounding the canaliculi and propelling secreted biliary fluid along the canaliculi are which of the following:A. Fibronectin and lamininB. Vinculin and actinC. * Actin and myosinD. Talin and vinculinE. Tensin and myosin

231 A single large lipid droplet, macro vesicular steatosis, that displaces the nucleus may be seen in all of the following, EXCEPT:A. Hepatitis virus C

B. Alcohol abuseC. ObesityD. ""Hepatitis virus BE. Diabetis mellitus

232 Injury to the liver associated with an influx of acute or chronic inflammatory cells is termed as which of the following:A. Liver regenerationB. Liver cirrhosisC. HepatomaD. Liver degeneationE. * Hepatitis

233 Yellow discoloration of the skin and sclerae, internal organs and mucus membranes is referred to as:A. MelanosisB. VitiligoC. NeviD. CyanosisE. * Jaundice

234 Unconjugated bilirubin is characterized by all of the following, EXCEPT:A. Insoluble in waterB. Tightly complexed to serum albuminC. Cannot be excreted in the urineD. May diffuse into tissue and produce toxic injuryE. * Increased only in impaired bile flow

235 Conjugated bilirubin is characterized by all of the following, EXCEPT:A. Water solubleB. NontoxicC. Loosely bound to albuminD. Can be excreted in the urineE. *May induce kernicterus

236 Jaundice occurs when the equilibrium between bilirubin production and clearance is disturbed by all of the following factors, EXCEPT:A. Excessive production of bilirubinB. * Increased hepatocellular excretionC. Impaired bile flowD. Reduced hepatocyte uptakeE. Impaired conjugation

237 All of the following syndromes are characterized by hereditary hyperbilirubinemias, EXCEPT:A. Crigler-Najjar syndrome type IIB. Gilbert syndromeC. *Crigler-Najjar syndrome type IIID. Dubin-Johnson syndromeE. Rotor syndrome

238 The morphologic alterations that cause liver failure are all of the following, EXCEPT:A. *Small fatty droplets in hepatocytesB. Massive hepatic necrosisC. Acute inflammationD. CirrhosisE. Chronic inflammation

239 Liver chirrhosis is characterized by all of the following, EXCEPT:A. Bridging fibrous septa

B. *Hemosiderin granules in liver cellsC. Parenchymal nodular architectureD. Reorganized vascular architectureE. Disrupted liver architecture

240 Collagen synthesis by hepatic stellate cells (Ito cells) and its deposition may be caused by all of the following, EXCEPT:A. Disruption of the extracellular matrixB. Direct stimulation of stellate cells by toxinsC. *Apoptosis of Ito cellsD. Cytokine production by stimulated liver cellsE. Inflammatory cytokine production (TNF-a, TGF-b)

241 The possible machanisms of most cirrhotic deaths are all of the following, EXCEPT:A. Progressive liver failureB. * Pulmonary thromboembolismC. Complication related to portal hypertensionD. Development of hepatocellular carcinomaE. Systemic infection

242 The major consequences in portal hypertension in cirrhosis are all of the following, EXCEPT:A. Formation of portosystemic venous shuntsB. Congestive splenomegalyC. Hepatic encephalopathyD. *Intracerebral hematomaE. Ascites

243 The pathogenesis of ascites involves all of the following mechanisms, EXCEPT:A. Sinusoidal hypertensionB. Percolation of the hepatic lymph into the peritoneal cavityC. * Increased oncotic plasma pressureD. Intestinal fluid leakageE. Renal retention of sodium and water

244 Principal sites of portosystemic shunts opening with the rise of portal systemic pressure are all of the following, EXCEPT:A. Veins within the rectum (hemorrhoids)B. Veins of cardioesophageal junctionC. Retroperitoneal veinD. Veins of falciform ligament of the liverE. * Hepatic veins

245 Hepatitis A is characterized by all of the following, EXCEPT:A. Benign, self-limited infectious courseB. Low death rateC. Fecal-oral route of transmissionD. Protective immunity against reinfectionE. * Fulminant course

246 Hepatitis B may be characterized by all of the following, EXCEPT:A. Prolonged persistence of HBV in patient's bloodB. Presence of HBV in all pathologic and physiologic body fluidsC. * Small size virus with defective RNAD. Vertical transmissionE. Contamination of blood and blood products

247 Hepatitis B virus may produce all of the following morphologic variants of liver disease, EXCEPT:A. * Liver steatosis

B. Acute hepatitisC. Nonprogressive chronic hepatitisD. Progressive chronic hepatitisE. Fulminant hepatitis

248 Hepatitis B virus may produce all of the following variants of liver disease, EXCEPT:A. Progressive chronic hepatitisB. Asymptomatic carrier stateC. Indolent chronic hepatitisD. Acute hepatitisE. * Congestive cirrhosis

249 The carrier state for hepatitis virus B is defined by the presence of which of the following:A. HBeAg in the urine for 12 monthsB. HBcAg in the cerebrospinal fluid for at least 6 monthsC. *HBsAg in the serum for 6 monthsD. HBcAg in the serum for 12 monthsE. HBsAg in the urine for 8 months

250 The possible outcomes of hepatitis virus B acute infection include all of the following, EXCEPT:A. Subclinical diseaseB. *"Nutmeg" liverC. Acute hepatitisD. "Healthy" carrierE. Persistent infection

251 Hepatitis virus C is characterized by all of the following, EXCEPT:A. Parenteral route of transmissionB. Possible sexual and vertical transmissionC. High rate of progression to chronic diseaseD. *Low rate of progression to cirrhosisE. Absence of effective immunity

252 Hepatitis virus D is characterized by all of the following, EXCEPT:A. High risk for HDV infection to male homosexualsB. Rare development of chronicityC. Conversion of mild chronic HBV infection to fulminant diseaseD. High risk for HDV infection to drug addicts and hemophiliacsE. * Common development of hepato-cellular carcinoma

253 Hepatitis virus E is characterized by all of the following, EXCEPT:A. Enterically transmitted, water-borne infectionB. Common in young to middle aged adultsC. High mortality rate among pregnant womenD. Commonly self-limited diseaseE. * Inducing massive liver necrosis

254 Morphologically, acute viral B hepatitis is characterized by all of the following, EXCEPT:A. Diffuse swelling (ballooning degeneration) of hepatocytesB. Inflammatory cells in the sinusoids and portal tractsC. * Portal tract expansion with fibrous tissueD. Hepatocellular apoptosisE. Confluent and bridging necrosis of hepatocytes

255 Morphologically, chronic indolent viral B hepatitis is characterized by all of the following, EXCEPT:A. Well preserved liver architecture

B. Periportal and bridging fibrosisC. Limited to portal tract inflammationD. * Severe cholestasisE. Smoldering hepatocyte necrosis throughout the lobule

256 Microscopically, fulminant hepatitis virus B is characterized by all of the following, EXCEPT:A. Complete destruction of hepatocytes in damaged lobulesB. Collapsed reticulin frameworkC. Preserved portal tractsD. Inflammatory cell reactionE. * Hepatocyte regenerative nodules

257 Macroscopically, the liver in chronic alcoholism is characterized by all of the following, EXCEPT:A. Enlargement (up to 4 to 6 kg)B. *"Nutmeg" liverC. Soft consistenceD. Yellow and greasy appearanceE. Smooth surface

258 Alcoholic hepatitis is characterized by all of the following, EXCEPT:A. Swelling, steatosis and necrosis of hepatocytesB. Mallory bodies formationC. * Councilman bodies formationD. Neutrophilic reactionE. Sinusoidal and perivenular fibrosis

259 Pneumonia (pneumonia) is the disease, that includes large group of varied etiology, pathogenic and morphological description of inflammations of: A. respirator department of lung B. pleura C. media sternum D. * pericardium E. overhead respiratory tracts

260 What can be in lungs by corking of bronchial tubes by mucus, inflammatory exsudate? A. * atelectatic pneumonia B. stagnant pneumonia C. paravertebral pneumonia D. hypostatic pneumonia E. croupous pneumonia

261 What can be in lungs by the hit of extraneous bodies in respiratory tracts? A. * aspiration pneumonia B. stagnant pneumonia C. hypostatic pneumonia D. atelectatic pneumonia E. croupous pneumonia

262 What is the most prevalent way of penetration of bronchopneumonia exciter? A. * bronchogenic B. haematogenic C. lymphogenic D. mixed E. enterogenic

263 Atelectasis is: A. * active сollaps of pulmonary fabric, which can arise up at the lack of surfactant B. multiplying the amount of teethridge

C. in diminishing of amount of teethridge D. passive сollaps force-feed exsudate or air E. passive сollaps of pulmonary fabric force-feed tumour

264 Collapse is: A. * passive сollaps force-feed exsudate, air or tumour B. active сollaps of pulmonary fabric, which can arise up at the lack of surfactant C. multiplying the amount of teethridge D. diminishing of amount of teethridge E. active сollaps of pulmonary fabric with diminishing of amount of teethridge

265 What can be after the exception of part of teethridge from a respiratory function? A. * vicarian (scray) emphysema B. collapse C. atelectasis D. syndrome of Hummen-Rich E. collapse and atelectasis

266 What is the type of pneumonia depending on character of exsudate? A. * serosal B. primary C. bronchopneumonia D. interstitial E. second

267 What is the possible complication of bronchopneumonia? A. * gangrene of lungs B. infarction of myocardium C. infarction of lungs D. syndrome of Hummen-Rich E. infarction of kidney

268 What belongs to the atypical forms of pneumonia? A. * intermediate B. viral C. croupous D. bronchopneumonia E. bacterial

269 Intermediate pneumonia is: A. * peribronchial B. lobular C. segmental D. acinusal E. croupous

270 The way of penetration of exciter of pneumonia can be: A. * bronchogenic B. contageous C. primary D. second E. enteral

271 What is the possible complication of interstitial pneumonia? A. * pneumosclerosis B. infarction of myocardium C. bronchial asthma D. acute bronchitis E. infarction of kidney

272 A chronic bronchitis with the protracted motion is accompanied:

A. * by dystrophy of elastic, muscular and cartilaginous frameworks B. by hypertrophy of elastic and cartilaginous frameworks. C. by atrophy elastic to framework D. by hyperplasia of muscular type E. by dystrophy cartilaginous to framework

273 Emphysema of lungs is: A. * pathological state of pulmonary fabric, which is characterized by enhanceable

maintenance in her air B. active collapse of pulmonary fabric, which can arise up at the lack of surfactant C. multiplying the amount of teethridge D. passive collapse force-feed exsudate, air or tumor E. pathological state of pulmonary fabric, which is characterized by the lowered

maintenance in her air 274 Emphysema of lungs is:

A. * vesicular B. acute C. chronic D. croupous E. subacute

275 Development of vesicular emphysema is connected with: A. * chronic bronchitis, bronchiolitis B. croupous pneumonia C. delete of part of lungs D. age-old involution of lungs E. lobar pneumonia

276 Chronic bronchitis is associated with all of the following pathologic changes, EXCEPT: A. * Inflammatory infiltration of alveolar walls B. Goblet cell hyperplasia in bronchi C. Goblet cell metaplasia in bronchioli D. Hypertrophy of bronchial submucosal glands E. Inflammatory infiltration of bronchioli

277 An autopsy of a DE-year-old male patient, who had double bronchopneumonia and died under the phenomena of intoxication, revealed in the lower lobe of the right lung some thick-walled cavity, D cm in diameter, filled with liquid yellowish masses. Choose the pathological process complicated by the course of pneumonia: A. * Tuberculoma B. Gangrene C. Abscess D. Sequester E. Empyema

278 An autopsy of 78-year-old male patient, who died from cardiopulmonary insufficiency, revealed an enlarged right lung with massive fibrinous superpositions on the pleura. Histologically, the alveolar lumen had accumulations of fibrin and neutrophils. The lymph nodes in the lung roots were pale pink and somewhat enlarged. Name the stage of croupous pneumonia: A. Red hepatization B. Influx C. * Grey hepatization D. Resolution E. Empyema

279 On autopsy, C80 ml of some yellow fluid with an unpleasant odour were found in the right pleural cavity. Microscopically, the liquid contained a lot of neutrophilic granulocytes. Choose the diagnosis: A. Pulmonary abscess B. Phlegmon C. * Pleural empyema D. Pulmonary gangrene E. Pulmonary infarction

280 An autopsy of a DC-year-old male, who died from cardiopulmonary insufficiency, revealed an enlarged grey dense air-free lower lobe of the left lung, the vincernal pleura of this lobe had superpositions of fibrin. Microscopically, the lumens of alveoli revealed some fibrinous-leukocytic exudate. Name the stage of croupous pneumonia: A. Red hepatization B. Influx C. * Grey hepatization D. Resolution E. Empyema

281 In 9E % cases croupous pneumonia is caused: A. * by Franklyn’s pneumococcus B. streptococcus C. staphylococcus D. by chlamydeous E. by a coli bacillus

282 An autopsy of a EE-year-old male, who died from cardiopulmonary insufficiency, revealed thickened deformed bronchi filled with some mucopurulent exudate. Microscopically, an inflammatory infiltration, foci of squamous metaplasia of the epithelium and an increased number of goblet cells were found in the bronchi. Choose the diagnosis: A. Chronic bronchitis B. Bronchopneumonia C. * Bronchiectatic disease D. Acute bronchitis E. Fibrinous bronchitis

283 Chronic obstructive bronchitis is associated with all of the following pathologic changes, EXCEPT: A. Goblet cell hyperplasia in bronchi B. Goblet cell metaplasia in bronchioli C. Hypertrophy of bronchial submucosal glands D. Inflammatory infiltration of bronchioli E. * Inflammatory infiltration of alveolar walls

284 Complications of bronchopneumonia include all of the following, EXCEPT: A. * Lung histiocytosis B. Bronchiectasis C. Pleural fibrosis D. Metastatic abscess formation E. Permanent lobar solidification

285 All of the following factors commonly predispose to bacterial pneumonias, EXCEPT: A. Viral respiratory tract infections B. Cigarette smoking C. Congestive heart failure D. * Bacterial urinary tract infection E. Anesthesia

286 Aspiration of gastric contents produces all of the following types of pulmonary injury, EXCEPT: A. Adult respiratory distress syndrome B. Lipoid pneumonia C. Lung abscess D. Empyema E. * Pulmonary alveolar proteinosis

287 Diseases with diffuse interstitial lung fibrosis development include all of the following, EXCEPT: A. Sarcoidosis B. Asbestos C. Rheumatoid arthritis D. * Measles pneumonia E. Bleomycin lung

288 Eosinophilic infiltrates characterize all of the following disorders, EXCEPT: A. * Pneumocystic infection B. Loeffler's syndrome C. Allergic bronchopulmonary aspergillosis D. Bronchial asthma E. Pigeon/bird breeder's lung

289 Cigarette smoke contributes to the pathogenesis of the chronic obstructive emphysema by all of the following mechanisms, EXCEPT: A. Attracting neutrophils into the lung B. Stimulating release of neutrophil elastase C. * Inhibiting the ability of pulmonary leukocytes to clear bacteria D. Inhibiting alpha A-antitrypsin E. Stimulating macrophage elastase activity

290 Noncaseating pulmonary granulomas can be found in all of the following diseases, EXCEPT: A. Chronic berylliosis B. Silicosis C. Sarcoidosis D. Alveococcosis E. * Tuberculosis

291 Primary pulmonary hypertension is characterized by which of the following: A. Strong association with cigarette smoking B. Association with pulmonary embolism C. Common association with chronic obstructive lung disease D. * Atherosclerosis of the pulmonary arteries E. "Bush tea " drinking lung

292 What is the color of sputum of croupous pneumonia in the stage red slivering? A. * ferruginous B. black C. red D. yellow E. grey

293 Cigarette smoking is causally related to all of the following pulmonary diseases, EXCEPT: A. Chronic bronchitis B. Centrilobular emphysema C. * Idiopathic pulmonary fibrosis (fibrosing alveolitis) D. Small airway disease (bronchiolitis)

E. Bronchogenic carcinoma 294 Types of emphysema, according to the anatomic nature of the lesion include all of the

following, EXCEPT: A. Centriacinar B. Panacinar C. Paraceptal D. Irregular E. * Interstitial

295 The abnormal fenestrations of the walls of the alveoli, destruction of septal walls, abnormal airspaces and possibly blebs or bullae are common histological features of which of the following diseases: A. * Chronic obstructive emphysema B. Chronic bronchitis C. Bronchial asthma D. Pheumonia E. Lung abscess

296 The abnormal dilation of the bronchi and bronchioli is the main characteristic feature of which of the following diseases: A. Emphysema B. Lung abscess C. Bronchiectasis D. Bronchial asthma E. Pneumonia

297 Patchy consolidation of the lung is the dominant characteristic of which of the following: A. * Bronchopneumonia B. Lobar pneumonia C. Bronchiectasis D. Emphysema E. Bronchial asthma

298 Acute bacterial infection of an entire lobe is often referred to as which of the following: A. Bronchopneumonia B. * Lobar pneumonia C. Emphysema D. Tuberculosis E. Bronchial asthma

299 The stages of lobar pneumonia include all of the following, EXCEPT: A. Congestion B. Red hepatization C. * Yellow hepatization D. Grey hepatization E. Resolution

300 Exudate in lobar pneumonia may consist of all of the following, EXCEPT: A. Fibrin B. Neutrophils C. Erythrocytes D. Macrophages E. * Plasma cells

301 Complications of pneumonia include all of the following, EXCEPT: A. Abscess formation B. Empyema C. Organization of the exudate D. * Thromboembolism of the pulmonary artery

E. Bacteremic dissemination 302 The most characteristic feature of viral pneumonia is which of the following:

A. * The interstitial nature of the inflammatory reaction B. Presence of acute suppurative inflammation C. Emphysema formation D. Coalescent granulomas formation E. Suppurative destruction of the lung parenchyma

303 Pulmonary complications of croupous pneumonia can be: A. * carnification B. pneumofibrosis C. acute bronchitis D. alveolitis fibrosis E. meningitis

304 The cardinal histologic change in the acute abscess of the lung is which of the following: A. * Suppurative destruction of the lung parenchyma B. Emphysema formation C. The interstitial nature of the inflammatory reaction D. Fibrinous exudate E. Ghon complex formation

305 All of the following diseases are pneumoconioses, EXCEPT: A. Anthracosis B. Silicosis C. Tuberculosis D. Asbestosis E. Berylliosis

306 The stain used to identify fibrin is which of the following: A. * Weigert's stain B. Congo red stain C. PAS-reaction D. Hematoxylin and eosin stain E. Toluidin blue stain

307 The term given to the group of lung diseases that are caused by the chronic inhalation of particulate or gaseous agents as a result of occupational exposure is which of the following: A. Granulomatous disease B. * Pneumoconiosis C. Mycobacteriosis D. Pseudolymphoma E. Bronchiectasis

308 Conditions that predispose to lung abscess formation are all of the following, EXCEPT: A. Aspiration of infective material B. Antecedent primary bacterial infection C. Septic embolism D. Neoplasia E. * Injury of the mucociliary apparatus

309 Factors that predispose to bacterial pneumonia are all of the following, EXCEPT: A. Loss or supression of the cough reflex B. Injury of the mucociliary apparatus C. Pulmonary congestion and edema D. Accumulation of secretions E. * Septic embolism

310 Type A hypersensitivity reaction is the most common cause of which of the following diseases:

A. * Atopic asthma B. Nonatopic asthma C. Drug-induced asthma D. Bacterial pneumonia E. Bronchogenic carcinoma

311 The histological features in hypersensitivity pneumonitis include all of the following, EXCEPT: A. Interstitial pneumonitis B. Interstitial fibrosis C. Obliterative bronchiolitis D. * Hemosiderosis E. Granuloma formation

312 The infection causing interstitial pneumonia is which of the following: A. Gram-positive bacterial B. Gram-negative bacterial C. * Viral D. Fungal E. Parasitic

313 The form of chronic interstitial pneumonia characterized by marked proliferation and desquamation of alveolar lining cells is which of the following: A. Usual interstitial pneumonitis B. Idiopathic pulmonary fibrosis C. * Desquamative interstitial pneumonitis D. Lymphoid interstitial pneumonitis E. Hamman-Rich syndrome

314 Out of lung complications of croupous pneumonia can be: A. * meningitis B. abscess of the lung C. empyema of pleura D. gangrene of the lung E. bronchitis

315 Tumor associated mainly with occupational exposure to asbestos is which of the following: A. Bronchioloalveolar carcinoma B. Oat cell carcinoma C. * Mesothelioma D. Squamous cell carcinoma E. Adenocarcinoma

316 Out of lung complications of croupous pneumonia can be: A. * peritonitis B. abscess of the lung C. empyema of pleura D. gangrene of the lung E. carnification

317 In case of insufficiency of blood circulation patients can have: A. * stagnant pneumonia B. hypostatic pneumonia C. paravertebral pneumonia D. atelectatic pneumonia E. croupous pneumonia

318 What patients can have in lungs after staying in bed during a long period of time? A. * hypostatic pneumonia

B. stagnant pneumonia C. aspiration pneumonia D. atelectatic pneumonia E. croupous pneumonia

319 "Black measles" is characterized by which of the following: A. * Hemorrhages B. Hypermelanosis C. Icterus D. Hyperchromatosis E. Multiple nevi

320 A calcified focus (fibrocalcific scar) forming in the lung parenchyma and in the hilar lymph node after the primary tuberculosis infection is also referred to as: A. Keloid B. Granuloma C. * Ghon focus D. Aschoff-Pule focus E. Simon focus

321 Acute adrenal insufficiency syndrome is also referred to as: A. * Waterhouse — Friderichsen syndrome B. Kimmelstill-Wilson syndrome C. Hamman-Rich syndrome D. Zollinger-Ellison syndrome E. Budd-Chiari syndrome

322 All of the following pathologic processes can be found in the second period of scarlet fever, EXCEPT: A. Vasculitis B. Acute glomerulonephritis C. Arthritis D. Endocarditis E. * Amyloidosis

323 All of these cells participate in immune response in primary lung tuberculosis, EXCEPT: A. Type I pneumocytes B. * Alveolar macrophages C. CDD+ helper T-cells D. CD8+ suppressor T-cells E. Double negative T-cells

324 An infectious agent causing diphtheria is which of the following: A. * Corynebacterium B. Staphylococcus C. Treponema D. Bordetella E. Esherichia

325 Caseating destructive secondary tuberculosis includes all of the following lesions, EXCEPT: A. Caseation in the lung B. Cavities in the lung C. Caseation in the lung lymph nodes D. * Miliary extrapulmonary lesions E. Extrapulmonary caseation

326 Complications of pyogenic meningitis include all of the following, EXCEPT: A. Leptomeningeal fibrosis B. Hydrocephalus

C. Adhesive arachnoiditis D. * Intracerebral hemorrhage E. Focal encephalitis

327 Complications of tuberculosis osteomyelitis include all of the following, EXCEPT: A. Tuberculosis arthritis B. Sinus tract formation C. Cold abscess formation D. * Caseation in the lung E. Amyloidosis

328 Consequences of diphtheria include all of the following, EXCEPT: A. Hyperplasia of the spleen B. Fatty myocardial changes C. * Systemic hemosiderosis D. Polyneuritis E. Focal necroses of the parenchyma of the organs

329 Diphtheria is characterized by the derangement of all of the following organs, EXCEPT: A. Nasopharynx B. Oropharynx C. Larynx D. Trachea E. * Esophagus

330 Erythematous rash in scarlet fever can be found on which of the following parts of the body: A. The trunk and inner aspects of the arms and legs B. * The upper aspects of the arms and legs C. The area about the mouth D. The knees E. The head

331 Exotoxin in diphtheria damages all of the following organs, EXCEPT: A. * Heart B. Nervous system C. Adrenals D. Kidneys E. Thyroid gland

332 Favored targets for miliary extrapulmonary seeding are all of the following organs, EXCEPT: A. Bone marrow B. Kidneys C. Liver D. * Uterus E. Spleen

333 Figuratively the influensa pneumonia is often referred to as: A. Hemosiderosis of the lung B. * Multicolored lung C. Silicosis D. Lung infarct E. Farmer's lung

334 Growth and multiplication of the tubercle bacilli in cavitary fibrocaseous tuberculosis are favored by which of the following pathologic conditions: A. Lymphatic drainage B. Progressive hypoxia C. Increased perfusion

D. * Increased oxygen tension E. Sludging of blood in alveolar capillaries

335 Healed lesions in primary tuberculosis include all of the following, EXCEPT: A. Fibrous incapsulation B. * Caseous pneumonia C. Fibrocalcific scar D. Foci of ossification E. Focal pleural adhesions

336 Impairment of bronchociliary function in influenza results in which of the following: A. Fungal superinfection B. * Bacterial superinfection C. Protozoal superinfection D. Mycoplasmal superinfection E. Chlamydial superinfection

337 in epidemic typhus fever is which of the following: A. Abscess B. Hyaline thrombus C. Caseous necrosis D. * VascuIitis E. Hemangioma

338 Influenza tracheitis is characterized by all of the following, EXCEPT: A. Hyperemic and swollen mucosa B. * Purulent exudate C. Narrowing of trachea lumen D. Pinpoint hemorrhages E. Grey-yellow mucosa covering films within trachea

339 Influenza virus of type A infects all of the following, EXCEPT: A. Horses B. Humans C. Pigs D. * Dogs E. Birds

340 Macrophages in tuberculous granulomatous inflammation can transform into which of the following cells: A. Monocytes B. Epithelial cells C. * Epithelioid cells D. Plasma cells E. Lymphocytes

341 Measles virus is transmitted by which of the following: A. Milk B. Food C. * Respiratory droplets D. Feces E. Blood

342 Measles virus multiplies inside all of the following cells, EXCEPT: A. Upper respiratory epithelial cells B. B lymphocytes C. Macrophages D. T lymphocytes E. * Hepatocytes

343 Meningococcal meningitis is most commonly characterized by which of the following types of inflammation: A. Serous B. * Purulent C. Fibrinous D. Granulomatous E. Catarrhal

344 Meningococcal nasopharyngitis is characterized by which of the following types of inflammation: A. * Catarrhal B. Serous C. Fibrinous D. Purulent E. Diphtherial

345 Microscopic features in diphtheria include all of the following, EXCEPT: A. Neutrophilic infiltration B. Vascular congestion C. Interstitial edema D. * Hyaline deposition E. Fibrin exudation

346 Miliary tuberculosis is associated with which of the following pathologic conditions: A. Reinfection B. Localized caseation in the lungs C. Localized caseation in the lymph node D. Primary infection E. * Hematogenous tuberculosis

347 Morphologically, measles pneumonia is characterized by all of the following, EXCEPT: A. Diffuse neutrophil infiltration of the bronchiolar walls B. Destruction of some fragments of bronchiolar walls and squamous cell metaplasia C. Exudate with giant cells in the adjacent alveoli D. Interstitial lung inflammation E. * Large areas of hemorrhage in lung tissue

348 Morphology of influenza pneumonia is characterized by all of the following features, EXCEPT: A. Diffuse neutrophil infiltration of the bronchiolar walls B. Dense fibrin-rich neutrophil exudate into adjacent alveoli C. Foci of necrosis and hemorrhages in lung tissue D. * Granuloma formation in bronchiolar walls E. Foci of necrosis and ulceration in bronchiolar mucosa

349 ?Mycobacterium tuberculosis is characterized by all of the following features, EXCEPT: A. * Pili-forming B. Aerobic C. Non-spore-forming D. Nonmotile E. Red colored in acid-fast staining

350 Nervous system pathology in diphtheria includes which of the following: A. Intracerebral hemorrhage B. Ischemic necrosis C. Hydrocephalus D. * Polyneuritis E. Encephalitis

351 On gross examination, measles pneumonia is characterized by all of the following, EXCEPT: A. Small grey-white foci in the lung B. Peribronchial localization of foci C. * Subpleural localization of foci D. Firm consistence of inflammatory foci E. Pus containing foci

352 On gross examination, measles pneumonia is characterized by which of the following: A. Cavities B. Foci of necrosis C. Microbe colonies D. * Leukocyte infiltration E. Lymphocyte infiltration

353 On intracutaneous injection of tuberculin in an individual previously exposed to tubercle bacilli arise all immunopathologic processes, EXCEPT: A. * Release of CCb, the opsonin that promotes phagocytosis of microorganisms B. Interaction of memory THA cells with the antigen on the surface antigen- presenting cells C. Activation of THA cells D. Blast transformation of THA cells E. Proliferation of THA cells

354 Pathogenetic mechanisms of cell injury in infectious diseases are all of the following, EXCEPT: A. Direct cell damage B. Indirect cell damage by release of toxins C. Indirect cell damage by release of enzymes D. Indirect cell damage by induction of killer effect E. * Direct cell damage by reduction of host responses

355 Pathological feature in the skin found in meningococcemia is which of the following: A. Erythematous rash B. * Hemorrhagic rash C. Furuncle D. Impetigo E. Koplic spots

356 Pathological feature that can be found in the heart in diphtheria is which of the following: A. Fibrinous pericarditis B. * Toxic myocarditis C. Bacterial endocarditis D. Myocardial infarction E. Heart aneurysm

357 Peribronchial pneumonia in measles often results in which of the following complications: A. Peripheral lung tumor B. Central lung tumor C. Hemosiderosis of the lung D. * Bronchiectasis E. Lung gangrene

358 Primary complex in tuberculosis may directly transform in all of the following pathologic conditions, EXCEPT: A. Fibrocalcific scars B. Latent pulmonary lesions C. * Miliary tuberculosis D. Latent extrapulmonary lesions

E. Progressive primary tuberculosis 359 Prolonged bronchiectasis may result in which of the following derangements of protein

metabolism: A. Primary amyloidosis B. Systemic hyalinosis C. * Secondary amyloidosis D. Mucoid changes E. Fibrinoid changes

360 Secondary tuberculosis is characterized by all of the following features, EXCEPT: A. Caseous necrosis and cavities in the lung B. * Primary focus in the lung C. Reinfection with Mycobacteria D. Reactivation of dormant disease E. Progression directly into the disseminated disease

361 Severe destruction of vertebrae in spine tuberculosis may result in all of the following pathologic conditions, EXCEPT: A. Permanent compression fractures B. Scoliotic deformities C. * Drainage tract (sequestrum) forming D. Kyphotic deformities E. Neurologic deficits

362 Substances that prevent complete phagocytosis of Mycobacterium tuberculosis by macrophages and induce delayed type hypersensitiv-ity are all of the following, EXCEPT: A. Cord factor B. * Interleukin C. Lipoarabinomanan (LAM) D. Heart-shock protein E. Activated complement

363 The cause of measles development is which of the following: A. RNA virus of the retrovirus family B. * RNA virus of the paramyxovirus family C. DNA virus of the picornavirus family D. DNA virus of the adenovirus family E. RNA virus of the togavirus family

364 The cause of poliomyelitis development is which of the following: A. RNA virus of the retrovirus family B. RNA virus of the paramyxovirus family C. * DNA virus of the picornavirus family D. DNA virus of the adenovirus family E. RNA virus of the togavirus family

365 The causes of death in meningococcemia include all of the following, EXCEPT: A. Acute adrenal insufficiency B. Bacterial shock C. Acute tubular necrosis D. Acute heart failure E. * Cerebral cachexia

366 The cavity in cavitary fibrocaseous tuberculosis is characterized by all of the following features, EXCEPT: A. Localized in the apex of the lung B. Lined by yellow-grey caseous material C. Walled by fibrous tissue D. Drained by bronchus

E. * Filled with suppurative exudate 367 The cells playing the most important role in chronic tuberculous inflammation are which

of the following: A. * Macrophages B. Leukocytes C. Eosinophils D. Erythrocytes E. Plasma cells

368 The complications of scarlet fever include all of the following, EXCEPT: A. Poststreptococcal glomeru-lonephritis B. Retropharyngeal abscess C. Purulent mastoiditis D. * Acute poststreptococcal hepatitis E. Phlegmon of the neck

369 The disease causing predominant derangement of the upper airways is which of the following: A. Bacterial pneumonia B. * Diphtheria C. Meningococcal nasopharyngitis D. Cholera E. Tuberculosis

370 The disease characterized by the loss of motor neurons and flaccid paralysis with muscle wasting and hyporeflexia is which of the following: A. Intracerebral hemorrhage B. Graves disease C. * Poliomyelitis D. Meningoencephalitis E. Diabetes mellitus

371 The electron micrograph of epithelial cells infected by influenza virus is characterized by all of the following, EXCEPT: A. Presense of influenza virus particles B. Hyperchromatic and enlarged nuclei C. * Displasia of epithelial cells D. Dilation of cysterns of endoplasmic reticulum E. Large vacuoles

372 The granuloma in tuberculosis is composed predominantly of which of the following cells: A. Fibroblasts B. * Epithelioid cells C. Eosinophils D. Plasma cells E. Neutrophils

373 The infectious agent causing meningococcal infection is which of the following: A. Streptococcus pyogenes B. Staphylococcus aureus C. Corynebacterium D. Viruses E. * Dyplococcus

374 The intestine may be affected in which of the following forms of tuberculosis: A. Secondary tuberculosis B. Cavitary fibrocaseous tuberculosis C. Dormant tuberculosis

D. * Primary tuberculosis complex E. Miliary tuberculosis

375 The measles rash results from which of the following: A. Nonimmune inflammation B. Delayed — type hypersensitivity C. Systemic immune complex reaction D. * T-cell — mediated immunity E. Antibody-mediated cellular dysfunction

376 The meningococcal infection includes all of the following forms, EXCEPT: A. Nasopharyngitis B. Pyogenic meningitis C. Meningococcemia D. * Pyogenic encephalitis E. Chronic meningitis

377 The miliary lung tuberculosis is characterized by which type of inflammation: A. * Granulomatous B. Serous C. Fibrinous D. Suppurative E. Hemorrhagic

378 The most common cause of death in patients with diphtheria is which of the following: A. * Acute heart failure B. Chronic heart failure C. Chronic lung failure D. Acute tubular necrosis E. Intracerebral hemorrhage

379 The most common sites of skeletal tuberculosis involvement are all of the following, EXCEPT: A. * Skull bones B. Thoracic vertebrae C. Lumbar vertebrae D. Knees E. Hips

380 The pathologic change seen in the spinal cord in poliomyelitis is which of the following: A. * Neuronophagia of the anterior horns motor neurons B. Large hematoma C. Hemosiderosis of the anterior horns motor neurons D. Lipofuscinosis of the posterior horns motor neurons E. Amyloidosis of the posterior horns motor neurons

381 The pathologic changes seen in the spinal cord in poliomyelitis are all of the following, EXCEPT: A. Mononuclear cell perivascular cuffs B. Neuronophagia of the anterior horns motor neurons C. Inflammation extension into the posterior horns D. * Abscess formation E. Cavitation

382 The possible causes of chronicity of inflammation in tuberculosis are all of the following, EXCEPT: A. Persistence of certain microorganisms B. Prolonged exposure to toxic agents C. Incomplete phagocytosis D. * Complete phagocytosis

E. Resistance of etiologic agent 383 The severe form of measles pneumonia is characterized by all of the following, EXCEPT:

A. Destructive panbronchitis B. Interstitial mononuclear infiltration C. Foci of suppurative inflammation D. Measles giant cells in the alveoli E. * Catarrhal bronchitis

384 The severe form of measles pneumonia is characterized by which of the following: A. Foci of hemorrhagic inflammation B. Catarrhal bronchitis C. * Destructive panbronchitis D. Caseous necrosis E. Granulomatous inflammation

385 The spine tuberculosis breaking through intervertebral discs and extending into the soft tissue with cold abscess forming is also referred to as: A. Paget's disease B. * Pott's disease C. Gohn complex D. Reinfection focus E. Dormant disease

386 The stain used to identify the causative organism in pyogenic meningitis is which of the following: A. Hematoxylin and eosin B. PAS-reaction C. * Gram D. Congo red E. Weigert's

387 The term "Koplik spots" referes to which of the following: A. Ulcerated gastric mucosal lesions B. Foci of granulomatous inflammation of the oral cavity C. Foci of purulent inflammation in the skin D. Ulcerated esophageal mucosal lesions E. * Ulcerated mucosal lesions in the oral cavity

388 The type of immune response initiated in tuberculosis is which of the following: A. Direct cell cytotoxicity reaction mediated by CDD T-cells B. * Delayed-type hypersensitivity reaction mediated by CDD T- cells C. Complement-dependent reaction D. Antibody-mediated cellular dysfunction E. Antibody-dependent cell mediated cytotoxicity

389 Tuberculoma is macroscopically characterized by all of the following features, EXCEPT: A. Intraparenchymal single mass B. Greyish-white appearance C. Well-circumscribed mass D. Several centimeters in diameter E. * Several millimeters in diameter

390 Tuberculous salpingitis can be found in which of the following forms of tuberculosis: A. Secondary tuberculosis B. Cavitary fibrocaseous tuberculosis C. Dormant lesions D. * Miliary tuberculosis E. Primary tuberculosis

391 Typical tuberculous granuloma is characterized by all of the following, EXCEPT:

A. * Plasma cells B. Area of central necrosis C. Epithelioid cells D. Langhans-type giant cells E. Lymphocytes

392 Ulcerated mucosal lesions in the oral cavity near the opening of Stensen ducts are referred to as: A. * Koplik spots B. Luschka spots C. Burkitt spots D. Paget spots E. Vegetations

393 Viruses can penetrate into the cell cytoplasm through all of the following mechanisms, EXCEPT: A. Translocation of the virus across the plasma membrane B. Fusion of the viral envelope with the cell membrane C. Receptor-mediated endocytosis of the virus D. Fusion with endosomal membranes E. * Virus apoptosis

394 A DC years old man is diagnosed clinically with the subacute rapidly progressive| glomerulonephritis. Its|its| morphological equivalent is|appears|: A. Intracapillar| proliferative | glomerulonephritis; B. Glomerulosclerosis |; C. Membranous glomerulonephritis; D. * Extracapillary proliferative glomerulonephritis | | E. Mesangial proliferative | glomerulonephritis.

395 A clinical syndrome is characteristic|character,typical| for the renal amiloidisis|kidneys| A. Hepatonephric B. Lowe's syndrome | C. Alport's syndrome | D. * Nephrotic syndrome E. Nephritic syndrome|

396 A gouty kidney|kidney| develops as a result of abnormality of metabolism of A. Chromoproteins | B. Lipids| C. * Purines D. Albumins E. Carbonhydrates

397 Advanced cervical carcinoma can extend by direct continuity to all of the following, EXCEPT: A. Urinary bladder B. Ureters C. * Colon transversum D. Rectum E. Peritoneum

398 All of the following clinical features are likely to be found in nephrotic syndrome, EXCEPT: A. Proteinuria B. Hypoalbuminemia C. * Hematuria D. Hyperlipidemia E. Edema

399 All of the following conditions predispose to urolithiasis, EXCEPT: A. * Sickle cell nephropathy B. Hyperparathyroidism C. Gout D. Proteus pyelonephritis E. Enteric hyperoxaluria

400 All of the following morphologic features characterize the ovary adenocarcinoma cells, EXCEPT: A. Enlarged nucleoli B. Atypical mitoses C. Variation in size and shape D. Hyperchromatic nuclei E. * Nucleus inclusions

401 All of the following statements correctly describe chronic pyelonephritis, EXCEPT: A. It causes asymmetrically scarred kidneys B. It is associated with vesicoureteral reflux in most cases C. * It spares the calyces and pelvis D. It may produce thyroidization of tubules E. It is an important cause of secondary nephrosclerosis

402 All of the following statements correctly describe analgesic abuse nephropathy, EXCEPT: A. It is characterized by tubulo-interstitial component B. It is often caused by phenacetin C. It causes inability to concentrate urine D. It often improves with drug withdrawal E. * It predisposes to the development of renal cell carcinoma

403 All of the following statements correctly describe renal artery stenosis, EXCEPT: A. It is an uncommon form of hypertension B. It is the most common curable form of hypertension C. It is usually caused by atherosclerotic plaque D. It produces high renin levels in the venous blood of the ischemic kidney E. * It is treated by hemodialysis

404 All of the following statements correctly describe chronic pyelonephritis, EXCEPT: A. It causes asymmetrically scarred kidneys B. It is associated with vesicoureteral reflux in most cases C. * It spares the calyces and pelvis D. It may produce thyroidization of tubules E. It is an important cause of secondary nephrosclerosis

405 All of the following statements correctly describe analgesic abuse nephropathy, EXCEPT: A. It is characterized by tubulo-interstitial component B. It is often caused by phenacetin C. It causes inability to concentrate urine D. It often improves with drug withdrawal E. * It predisposes to the development of renal cell carcinoma

406 All of the following statements correctly describe renal artery stenosis, EXCEPT: A. It is an uncommon form of hypertension B. It is the most common curable form of hypertension C. It is usually caused by atherosclerotic plaque D. It produces high renin levels in the venous blood of the ischemic kidney E. * It is treated by hemodialysis

407 All of the following statements regarding Goodpasture's syndrome are true, EXCEPT:

A. Patients present with hemoptysis and hematuria B. Death occurs due to uremia and pulmonary hemorrhage C. Electron microscopy shows the absence of electron-dense deposits D. * Immunofluorescence reveals granular deposits of IgG in the glomeruli E. Immunofluorescence reveals linear deposits of IgG in the glomeruli

408 All of the following statements regarding postinfectious glomerulonephritis are true, EXCEPT: A. The disease follows streptococcus infection B. Electron microscopy shows large subendothelial immune-type deposits C. The histologic picture is that of diffuse proliferative glomerulonephritis D. The clinical picture is characteristic of acute nephritis E. * Most affected children develop chronic renal failure

409 All statements concerning chorion-carcinoma are true, EXCEPT: A. The tumor has a dimorphic pattern B. Hemorrhages and necrosis are present C. The tumor is malignant D. The tumor is composed of cytotrophoblast and syncytiotrophoblast cells E. * The tumor produces chorionic villi

410 All statements concerning leiomyomas are true, EXCEPT: A. They regress or calcify after castration or menopause B. * They do not respond to estrogens C. They may undergo rapid increase in size during pregnancy D. Their cause is unknown E. They are found in BE% of reproductive women

411 An immunofluorescence-stained kidney specimen from a patient with poststreptococcal glomerulonephritis is likely to show which of the following: A. * Granular deposits of Ig G B. Linear deposits of Ig G C. Granular deposits of IgA D. Linear deposits of streptococcal antigen E. Granular deposits of streptococcal antigen

412 An important factor in cervical oncogenesis is which of the following: A. * HPV-infection B. Herpes simplex C. Herpes zoster D. Human immunodeficiency virus E. Respiratory syncytial virus

413 At extracapillar productive glomerulonephritis half moons are formed by A. Red corpuscles B. Neutrophilic leucocytes C. Red corpuscles and neutrophilic leucocytes D. Exfoliated endothelium and fibrin E. * Exfoliated renal epithelium| and fibrin

414 Benign nephrosclerosis is characterized by all of the following, EXCEPT: A. Narrowing of the lumen of the arterioles and small arteries B. Thickening and hyalinization of the vessels' walls C. * Deposition of amyloid within the Bowman space D. Foci of tubular atrophy E. Deposition of collagen within the Bowman space

415 Changes in a colon at uremia are A. Granulomatous inflammation B. Stricture

C. Polyposis D. Ulcerous colitis E. * Fibrinous [diphtheritic] colitis

416 Chorioncarcinoma is commonly associated with which of the following: A. Systemic hypertension B. * Pregnancy C. Obesity D. Oral contraceptive steroid use E. Diabetes mellitus

417 Conditions leading to endometrial hyperplasia include all of the following, EXCEPT: A. Polycystic ovarian disease B. Functioning granulosa cell tumors of the ovary C. Excessive cortical function (cortical stroma hyperplasia) D. * Endometriosis E. Estrogen replacement therapy

418 Criteria to differentiate between benign and malignant tumors are all of the following, EXCEPT: A. Maturity B. Rate and character of growth C. * Edema D. Metastases E. Local invasion

419 Diabetes mellitus is associated with all of the following renal disorders, EXCEPT: A. Diffuse glomerulosclerosis B. Nodular glomerulosclerosis C. Benigh nephrosclerosis D. * Urate nephropathy E. Acute pyelonephritis

420 Distant hematogenic metastases of invasive cervical carcinoma occur in all of the following, EXCEPT: A. Liver B. Lungs C. Bone marrow D. * Lymph nodes E. Kidney

421 Endometrial hyperplasia is associated with which of the following: A. High estrogenic stimulation with normal progestational activity B. Normal estrogenic stimulation with increased progestational activity C. * High estrogenic stimulation with diminished progestational activity D. High estrogenic stimulation and progestational activity E. Normal estrogenic stimulation and progestational activity

422 First lymphogenous metastases of the papillary mucinous cystadnocarcinoma of the ovary can be found in which of the following organs: A. * Pelvic lymph nodes B. Aortic lymph nodes C. Liver D. Lungs E. Bones

423 First metastases of the Chorioncarcinoma can be found in which of the following organs: A. Liver B. * Lung C. Kidney

D. Bones E. Brain

424 Give the determination|definition| of gynecomastia A. Non-cancerous cystous dysplasia |of mammary gland B. Non-cancerous hyperplasia| of mammary gland C. Malignant dysplasia| of mammary gland D. Non-cancerous scleroyic| dysplasia| of mammary gland E. * Non-cancerous dysplasia| of breast in men

425 Glomerular injury caused by circulating complexes occurs in all of the following disorders, EXCEPT: A. Syphilis B. Goodpasture's syndrome C. Hepatitis B D. Systemic lupus erythematosus E. * Lung cancer

426 Hematuria is a characteristic clinical feature of all of the following diseases, EXCEPT: A. Glomerulonephritis B. * Malakoplakia C. Nephrolithiasis D. Renal cell carcinoma E. Bladder papilloma

427 Higher frequency of carcinoma of the endometrium is commonly associated with all of the following, EXCEPT: A. * Young age B. Obesity C. Diabetes mellitus D. Hypertension E. Infertility (anovulatory cycles)

428 Histologic features of malignant nephrosclerosis include all of the following, EXCEPT: A. Fibrinoid necrosis of arterioles B. Medial thickening of arterioles C. * Fibromuscular dysplasia of the renal artery D. Renal artery thrombosis E. Focal renal parenchymal infarction

429 Histological type of of high quality hyperplasia| of prostatic| gland A. Muscular type| B. Stromal vascular type C. Amyloid type D. Fibroid cystous type E. * Mixed type

430 Hydronephrosis is caused by all of the following, EXCEPT: A. * Chronic renal vein thrombosis B. Large uterine leiomyoma C. Renal calculi D. Benign prostatic hypertrophy E. Papillary transitional cell carcinoma of the ureter

431 Hydronephrosis is characterized by all of the following, EXCEPT: A. * Kidney infarct B. Thinning of the renal parenchyma C. Dilatation of the renal pelvis D. Dilatation of the renal calyces E. Progressive atrophy of the kidney

432 Immunologicaly mediated glomerulonephritis all of the following cells contribute to the glomerular injury, EXCEPT: A. * Mast cells B. Macrophages C. Platelets D. Neutrophils E. Mesangial cells

433 In case of poisoning by mercuric chloride there is a necrosis of | A. Cardiac hystiocytes | B. * Epithelium of renal tubules C. Glomerular system of kidneys|kidneys| D. Renal stroma||kidneys| E. Wall of ureter

434 In nonobstructive chronic pyelonephritis the most common way for bacteria to gain entrance into the kidney is which of the following: A. Arterial bloodstream B. The lymphatics C. Venous bloodstream D. * Vesicoureteral reflux E. Aberrant arteriovenous shunts

435 In pathogeny of glomerulonephritis a considerable role plays borrows: A. * Sensitizing of an organism B. Reinfection C. Toxicness of microbes germs D. A presence of the inflammatory diseases of kidneys in anamnesis kidneys E. Superinfection

436 In the urinary tract obstruction all pathologic processes can be found. EXCEPT: A. Dilatation of the pelvis and calyces B. * Ischemic tubular necrosis C. Interstitial inflammation D. Interstitial fibrosis E. Glomerular and tubular atrophy

437 In the urinary tract obstruction all pathologic processes can be found. EXCEPT: A. Dilatation of the pelvis and calyces B. * Ischemic tubular necrosis C. Interstitial inflammation D. Interstitial fibrosis E. Glomerular and tubular atrophy

438 Invasive cervical carcinoma can manifest in which of the following patterns: A. * Ulcerating tumor B. Endocervical polyp C. Grapelike projections D. Paget's cancer E. Flattened plaque cancer

439 Mesangial cells can be characterized by all of the following properties, EXCEPT: A. Ingestion of macromolecules B. Connection with Lacis cells C. Ability to contract D. * Production of renin E. Production of basement membrane proteins

440 Microscopically, simple hyperplasia of endometrium is characterized by all of the following, EXCEPT:

A. * Pressed endometrial glands B. Dilated endometrial glands C. Increased gland-to-stroma ratio D. Abundant stroma E. Complex ramified endometrial glands

441 Morphological form|shape| of subacute glomerulonephritis is:|stream,current| A. Mesangial membranous glomerulonephritis | B. * Extracapillar proliferative C. Mesangial proliferative| D. Minimum changes E. Intracapillar| proliferative|

442 Most forms of chronic renal failure produce increased serum levels of all of the following substances, EXCEPT: A. * Calcium B. Aldosterone C. Phosphate D. Parathormone E. Renin

443 Most frequent complication of glomerulonephritis is A. Amiloidosis B. Increase|rise| in arterial pressure C. * Arteriolosclerotic kidney D. Renal abscesses E. Hematuria|

444 Name the|calls| complication of dishormonal hypertrophic prostatopathy|? A. Formation of the prostatic cysts. B. Purulent melting of prostate C. Sclerosis of prostate D. Appearance of polypous| excrescences E. * Difficulty in the urine outflow

445 Name|calls| a process, which is regarded as an inflammatory disease of mucous membrane of uterus|matrix| A. Cystophorous hyperplasia| B. Pseudo erosion C. Polypous| excrescence D. Endometriosis| E. * Endometritis

446 Obstetrically related renal disease includes all of the following disorders, EXCEPT: A. * Nephrocalcinosis B. Diffuse cortical renal necrosis C. Acute ischemic tubular necrosis D. Acute glomerulonephritis E. Hydronephrosis

447 of the following features of the tumor: A. Lack of the capsule B. Absence of estrogen receptors C. Presence of secondary necrosis and hemorrhages D. Extensive angiogenesis E. * Early spread of hematogenous metastases

448 On gross examination, leiomyoma of the uterus is characterized by all of the following, EXCEPT: A. Intramural, submucosal or subserosal localization

B. Discrete multiple nodules C. Firm consistence D. Fibrous capsule E. * Dark brown color

449 On gross examination, leiomyoma of the uterus may be characterized by which of the following: A. Indistinct margins B. Brown color C. Soft consistence D. * Fibrillated cut surface E. Multicolored

450 On gross examination, the hyperplasia of endometrium is characterized by all of the following, EXCEPT: A. Enlarged uterus B. Thickened uterus wall C. Foci of hemorrhages into endometrium D. * Suppurative endometritis E. Irregular thickened endometrium

451 Patogenetic basis of glomerulonephritis is formed by: A. * Immunoreactions on the basic membranes of glomerular capillaries B. Exsudates reactions on the basic membranes of glomerular capillaries C. Necrotic reactions on the basic membranes of glomerular capillaries D. Exsudative necrotic reactions on the basic membranes of glomerular capillaries E. Proliferative| reactions on the basic membranes of glomerular capillaries

452 Picrofuchsin by van Gieson staining of leiomyoma of the uterus results in which of the following: A. Red color of tumor parenchyma B. Blue color of tumor parenchyma C. Orange color of tumor parenchyma D. Yellow color of the tumor stroma E. * Red color of the tumor stroma

453 Proteinuria| at glomerulonephritis is caused by A. Purulent exsudate permeating into the urine B. Hyperproteinemia| C. Purulent infiltration of stroma| D. * Damage of wall of glomerular capillares E. Damage of tubular wall

454 Renal diseases producing systemic hypertension include all of the following, EXCEPT: A. Acute glomerulonephritis B. * Renal amyloidosis C. Chronic glomerulonephritis D. Chronic pyelonephritis E. Renal vasculitis

455 Renal diseases producing systemic hypertension include all of the following, EXCEPT: A. Acute glomerulonephritis B. * Renal amyloidosis C. Chronic glomerulonephritis D. Chronic pyelonephritis E. Renal vasculitis

456 Risk factors for cervical cancer development include all of the following, EXCEPT: A. Early age at first intercourse B. Multiple sexual partners

C. Oral contraceptive use D. * Alcohol abuse E. HPV- infection

457 Structurally-functional unit of kidney|kidney| is|appears|: A. Kidney pyramid; B. Kidney little body; C. Kidney lobule|clove,section|; D. Kidney bowl; E. * Nephron

458 Subendothelial granular electron-dense deposits can be found in which of the following diseases: A. Rapidly progressive glomerulonephritis B. Side cell nephropathy C. Membranous glomerulonephritis D. * Systemic lupus erythematosus E. Gouty nephropathy

459 Symptomatic hypertension evolves|secondary|| most frequently from: A. * Glomerulonephritis B. Atherosclerosis of renal arteries C. Thyrotoxicosis D. Aldosteronism | E. Pheochromocytoma |

460 Systemic lupus erythematosus gives rise to all of the following patterns of glomerular injury, EXCEPT: A. Focal proliferative glomerulonephritis B. Diffuse membranous glomerulonephritis C. Diffuse proliferative glomerulonephritis D. * Lipoid nephrosis E. Mesangial proliferative glomerulonephritis

461 The benign tumor characterized by the development of cysts lined by tall columnar epithelial cells and formation of regular papillary structures is referred to as: A. Chorioncarcinoma B. Adenoma C. Adenocarcinoma D. * Cystadenoma E. Fibroadenoma

462 The changes of epithelium of tubulis are in the shock stage of acute kidney insufficiency A. Metaplasia of epithelium B. * Dystrophy of epithelium C. Necrosis of epithelium D. The changes are absent|absents| E. Removing a layer by the layer of epithelium

463 The changes of tubular epithelium in the oliguric| stage of acute renal insufficiency A. Metaplasia of epithelium B. Dystrophy of C. * Necrosis of epithelium D. Changes are absent|absents| E. Exfoliation of epithelium

464 The characteristic|character,typical| feature of chorioepithelioma| is|appears| A. Sclerosis of stroma| B. * Absence of stroma C. Atrophy of chorionic| epithelium

D. Presence of lymphatic vessels E. Petrification|

465 The dishormonal disease of the sexual system of men is A. * Gynecomastia B. Prostatitis| C. Orchitis| D. Epididymitis E. Cancer of testicles

466 The dishormonal disease of the sexual system of men is A. Prostatitis | B. * Adenoma of prostatic| gland C. Orchitis | D. Cancer of prostatic| gland E. Urethritis

467 The dishormonal disease of the sexual system of women is | A. Cervicitis | B. Endometritis | C. * Mastopathy D. Salpingitis E. Cancer of the body of uterus

468 The dishormonal disease of the sexual system of women is A. * Endocervicosis B. Oophoritis | C. Cystitis D. Sarcoma of uterus|matrix| E. Endometritis |

469 The essence of pyelonephritis is A. Autoimmune aggression ||by| of renal tissue B. Non suppurative inflammation of glomerules C. Necrosis of renal cortical layer|kidney| D. * Suppurative inflammation of kidney E. Hyalinosis of glomerular vessels

470 The factor least likely to cause acute pyelonephritis is which of the following: A. Pregnancy B. Nephrolithiasis C. Catheterization of the bladder D. Prostatic hypertrophy E. * Septicemia

471 The factor least likely to cause acute pyelonephritis is which of the following: A. Pregnancy B. Nephrolithiasis C. Catheterization of the bladder D. Prostatic hypertrophy E. * Septicemia

472 The leiomyoma parenchyma is characterized by all of the following, EXCEPT: A. Haphazardly situated cells B. Uniformed in size and shape cells C. * Small vessels D. Oval nuclei of smooth muscle tumor cells E. Long cytoplasmic processus of tumor cells

473 The main complication of nephrolithiasis |is A. Hypertrophy of the left ventricle

B. Hypertension C. * Hydronephrosis D. Contracted [granular] kidney |kidney| E. Cancer of kidney

474 The malignant tumor characterized by easily recognizable glandular tubular pattern is referred to as: A. Chorioncarcinoma B. Adenoma C. * Adenocarcinoma D. Cystadenoma E. Fibroadenoma

475 The most common benign tumor of the ovary is which of the following: A. * Cystadenoma B. Adenocarcinoma C. Cystadenocarcinoma D. Papilloma E. Fibroadenoma

476 The most frequent clinical display of fibroid hyperplasia| of mucous membrane of uterus is|matrix|: A. Pain|anguish| syndrome B. Masculinization C. * Metrorrhagia D. Enlargement of uterus|matrix| determined by ultrasonography E. The characteristic|characters,typical| clinical displays are absent|absents|

477 The Russian scientist who was the first to describe the histogenesis of chorioncarcinoma was: A. R.Virchov B. * M.N.Nikiforov C. N.I.Pirogov D. I.M.Setchenov E. A.I.Abrikosov

478 The stain that helps to detect precancer and cancer of the cervix in smears is which of the following: A. Weigert's B. * Papanicolaou C. Sudan III D. Toluidin blue E. Hematoxylin and eosin

479 The stain used to differentiate leiomyoma and fibroma is which of the following: A. * Picrofuchsin B. Congo red C. Sudan III D. Toluidin blue E. Fuchselin

480 The tubular epithelial cells in acute tubular necrosis are characterized by all of the following pathologic features, EXCEPT: A. Karyolysis B. Plasmolysis C. Plasmorrhexis D. Plasmocoagulation E. * Tubulorrhexis

481 The tubular epithelial cells in acute tubular necrosis are characterized by all of the following pathologic features, EXCEPT: A. Karyolysis B. Plasmolysis C. Plasmorrhexis D. Plasmocoagulation E. * Tubulorrhexis

482 The tumor composed of clusters of cuboid cytotrophoblast cells separated by streaming masses of syncytiotrophoblast giant cells with marked atypia is referred to as: A. * Chorioncarcinoma B. Adenoma C. Adenocarcinoma D. Cystadenoma E. Fibroadenoma

483 Ultrastructural changes in children primary nephrotie syndrome involve which of the following glomerular elements: A. Endothelium B. * Podocytes C. Mesangium D. Bloodvessels E. Basement membrane

484 Ultrastructural changes in children primary nephrotie syndrome involve which of the following glomerular elements: A. Endothelium B. * Podocytes C. Mesangium D. Bloodvessels E. Basement membrane

485 Uremia is associated with all of the following abnormalities, EXCEPT: A. Peripheral neuropathy B. Gastritis C. * Polycythemia D. Pericarditis E. Diffuse alveolar damage

486 Uremia is associated with all of the following abnormalities, EXCEPT: A. Peripheral neuropathy B. Gastritis C. * Polycythemia D. Pericarditis E. Diffuse alveolar damage

487 What |enumerated| illness from the listed below relates to|by| the large mottled kidney|kidney| A. Amiloidosis of kidney|kidneys| B. Chronic pyelonephritis C. Nephrolithiasis| D. * Subacute glomerulonephritis E. Chronic glomerulonephritis

488 What abnormality develops at shock A. * Acute kidney insufficiency B. Chronic kidney insufficiency C. Glomerulonephritis D. Renal amiloidisis|kidneys|

E. Renal cirrhosis| 489 What belongs to|by| the inherited diseases from these tubular patologies|

A. Pyelonephritis B. Necrotic nephrosis C. * Tubular enzimatic| pathology D. “Myelome” kidney|kidney| E. “Shield-shaped|” kidney|kidney|

490 What develops in case of prolong obstruction of ureters by stone|rocks| in a kidney|kidney| A. Heart attack B. Gangrene C. Glomerulonephritis D. Amiloidosis E. * Hydronephrosis

491 What develops in case of prolong obstruction of ureters by stone|rocks| in a kidney|kidney| A. Heart attack B. Gangrene C. Glomerulonephritis D. Amiloidosis E. * Hydronephrosis

492 What disease|kidneys| can be complicated by amyloidosis: A. Atherosclerosis B. Heart ischemic disease C. * Fibrous cavernous | tuberculosis of lungs D. Croupous pneumonia E. Hypertension

493 What disease|kidneys| can be complicated by amyloidosis: A. Atherosclerosis B. Heart ischemic disease C. * Fibrous cavernous | tuberculosis of lungs D. Croupous pneumonia E. Hypertension

494 What pathologic condition of the kidneys is caused by mercury poisoning? A. * Acute tubular necrosis B. Renal papillary necrosis C. Crescentic glomerulonephritis D. Acute interstitial nephritis E. Renal cell carcinoma

495 What pathologic condition of the kidneys is caused by mercury poisoning? A. * Acute tubular necrosis B. Renal papillary necrosis C. Crescentic glomerulonephritis D. Acute interstitial nephritis E. Renal cell carcinoma

496 What pathological process the hypertrophy of prostatic| gland is related to? A. Chronic prostatitis| B. Acute prostatitis| C. Hyperfunction| of sexual glands D. Disorder of the urine outflow| E. * Hypofunction of sexual glands

497 What process occurs|exists| in the wall of urinary bladder at prostatopathy |?

A. Sclerosis B. Fibrosis| C. Atrophy D. The changes are absent|absents| E. * Compensatory hypertrophy

498 What type of epithelium covers the mucous membrane of urinoexcretory ways? A. Monolayer | pavement epithelium; B. * Multi-layered transitional C. Multi-layered | ciliated| D. Monolayer | cylindrical E. Multi-layered pavement unonkeratinizing |.

499 What type of epithelium covers the mucous membrane of urinoexcretory ways? A. Monolayer | pavement epithelium; B. * Multi-layered transitional C. Multi-layered | ciliated| D. Monolayer | cylindrical E. Multi-layered pavement unonkeratinizing |.

500 Which statement correctly characterizes membranous glomerulopathy? A. It is the most common cause of nephrotie syndrome in children B. Patients usually present with acute renal failure C. It is characterized by diffuse proliferative glomerulonephritis D. * Electron microscopy demonstrates numerous subepithelial immunetype deposits E. It is characterized by mesangial interposition phenomenon

501 Which statement correctly characterizes membranous glomerulopathy? A. It is the most common cause of nephrotie syndrome in children B. Patients usually present with acute renal failure C. It is characterized by diffuse proliferative glomerulonephritis D. * Electron microscopy demonstrates numerous subepithelial immunetype deposits E. It is characterized by mesangial interposition phenomenon

502 During an ectopic pregnancy in the uterine wall of the pipe through a small hole, which is overweighted with packages tamponirovalos blood. What is the name of such a complication? A. Blood drop B. *Covered the gap C. Cystic drop D. Incomplete tubal abortion E. Full Pipe abortion

503 What are the main morphological structures are signs of pregnancy? A. glandular hyperplasia edometriya B. placental polyp C. * chorionic villi and decidual tissueD. Iron-cystic hyperplasia E. Endotservikoz

504 Termination of pregnancy and the allocation of the uterus before the fetus from 14 weeks of conception denote as: A. Miscarriage B. Late abortion C. *Early abortion D. Premature birth E. Criminal abortion

505 Termination of pregnancy and the allocation of the fetus from the uterus of 14 weeks to 20 weeks from the moment of conception denote as:

A. Miscarriage B. *Late abortion C. Early abortion D. Premature birth E. Criminal abortion

506 Allocation of the fetus from the uterus of 28 to 37 weeks is called: A. Miscarriage B. Late abortion C. Early abortion D. *Premature birth E. Criminal abortion

507 Abortion was carried out outside the boundaries of the medical institution is called: A. Septic abortion B. *Criminal abortion C. Artificial board D. Cystic mole E. Vesiculare cystic mole

508 It is called an abortion outside a medical institution: A. Septic abortion B. *Criminal abortion C. Artificial board D. Puzirny mole E. Vesiculare cystic mole

509 Villi cystic placenta reborn, their number increased dramatically, becoming a conglomerate of the vine with: A. *Cyst change of placentaB. Premature birth C. Late abortion D. Miscarriage E. Trubnaya pregnancy

510 .What is the name of the process, when the placenta villi cystic reborn, their number increased dramatically, becoming a conglomerate of vines: A. *Vesiculare mole B. Premature birth C. Late abortion D. Miscarriage E. Tubular Pregnancy

511 In microscopic examination revealed endometrial villi chorion, the convolution of blood decidual tissue. What is the process of being developed? A. Glandular hyperplasia of endometrial B. Iron-cystic hyperplasia of endometrial C. *Placental polyp D. Fibrous polyp E. Adenomatous polyp

512 In the microscopic study revealed endometrial villi of chorion, the convolution of blood decidual tissue. What is the process of being developed? A. Glandular hyperplasia of endometrial tissue B. Iron-cystic hyperplasia endometrial tissue C. *Placental polyp D. Fibrous polyp E. Adenomatous polyp

513 . In the microscopic examination of endometrial tissue revealed diffuse leukocyte infiltration. What is the process of being developed? A. Placental polyp B. Horionepithelioma C. *Endometritis D. Septic endometritis E. Glandular hyperplasia of endometrium

514 . Name the process in endometrium which appears diffuse leukocyte infiltration. A. Placental polyp B. Horionepithelioma C. *Endometritis D. Septic endometritis E. Glandular hyperplasia of endometrium

515 . Embriopathias - A. *Pathology embryonic period, with 16 days of pregnancy B. Pathology embryonic period, with 20 days of pregnancy C. Pathology embryonic period, with 40 days of pregnancy D. Pathology embryonic period, with 52 days of pregnancy E. Pathology embryonic period, with 58 days of pregnancy

516 . Which includes embriopatias - A. *Pathology embryonic period, with 16 days of pregnancy B. Pathology embryonic period, with 20 days of pregnancy C. Pathology embryonic period, with 40 days of pregnancy D. Pathology embryonic period, with 52 days of pregnancy E. Pathology embryonic period, with 58 days of pregnancy

517 . What is embriopathias - A. *Pathology embryonic period, with 16 days of pregnancy B. Pathology embryonic period, with 20 days of pregnancy C. Pathology embryonic period, with 40 days of pregnancy D. Pathology embryonic period, with 52 days of pregnancy E. Pathology embryonic period, with 58 days of pregnancy

518 The most difficult gaps arise in the development of the child injury in the early neural tube bookmark: A. *3-4 weeks of intrauterine development B. 5-6 week of intrauterine development C. 7-8 week of intrauterine development D. 9-10 weeks of intrauterine development E. 11-12 weeks of intrauterine development

519 What are the disadvantages of the most difficult child development occur when damage to the neural tube early favorite: A. 3-4 weeks of intrauterine development B. 5-6 week of intrauterine development C. 7-8 week of intrauterine development D. 9-10 weeks of intrauterine development E. 11-12 weeks of intrauterine development

520 When the most difficult developmental damage in children with neural tube early favorite: A. *3-4 weeks of intrauterine development B. 5-6 week of intrauterine development C. 7-8 week of intrauterine development D. 9-10 weeks of intrauterine development E. 11-12 weeks of intrauterine development

521 Microcephaly, primary marrow hypoplasia relate to: A. *Embriopathias B. Fetopathias C. Pathology consequences D. Secular change E. Perinatal Pathology

522 To which group pathology include microcephaly-(hypoplasia main brain): A. *Embriopathias B. Fetopathias C. Pathology consequences D. Secular change E. Perinatal Pathology

523 Indicate which group pathology include microcephaly-(hypoplasia main brain): A. *Embriopathias B. Fetopathias C. Pathology consequences D. Secular change E. Perinatal Pathology

524 . Born hydrocephalus, the excess accumulation of liquor in the cerebral ventricle or subarachnoid space, refers to: A. *Embriopathias B. FetopathiasC. Pathology consequences D. Secular change E. Perinatal Pathology

525 In which group of diseases include hydrocephalus nee-(excessive accumulation of liquor in the cerebral ventricle or subarachnoid space): A. *EmbriopathiasB. Fetopathias C. Pathology consequences D. Secular change E. Perinatal Pathology

526 Specify which group of diseases include hydrocephalus nee-(excessive accumulation of liquor in the cerebral ventricle or subarachnoid space): A. *Embriopathias B. FetopathiasC. Pathology consequences D. Secular change E. Perinatal Pathology

527 . Spinal cord herniation associated with splitting dorsal vertebrae divisions, are: A. *Embriopathias B. Fetopathias C. Pathology consequences D. Secular change E. Perinatal Pathology

528 To which group of diseases included spinal cord herniation associated with splitting dorsal vertebrae divisions: A. *Embriopathias B. Fetopathias C. Pathology consequences D. Secular change E. Perinatal Pathology

529 . Specify which group of diseases included spinal cord herniation associated with splitting dorsal vertebrae divisions: A. Embriopathias B. Fetopattias C. Pathology consequences D. Secular change E. Perinatal Pathology

530 Defect mezhzheludochnoy partition belongs to: A. Fetopathias B. Pathology consequences C. *Embriopathias D. Birth trauma E. Perinatal Pathology

531 . To which group of diseases include defect ventricular septa: A. FetopathiasB. Pathology consequences C. *Embriopathias D. Birth trauma E. Perinatal Pathology

532 Specify which group of diseases include defect ventricular septa: A. Fetopathias B. Pathology consequences C. *Embriopathias D. Birth trauma E. Perinatal Pathology

533 . Complete transposition pulmonary artery and aorta refers to: A. Fetopathias B. Pathology consequences C. Birth trauma D. *Embriopathias E. Perinatal Pathology

534 To which group of diseases include full transposition pulmonary artery and aorta: A. Fetopathias B. Pathology consequences C. Birth trauma D. *Embriopathias E. Perinatal Pathology

535 Specify which group of diseases include full transposition pulmonary artery and aorta: A. Fetopathias B. Pathology consequences C. Birth trauma D. *EmbriopathiasE. Perinatal Pathology

536 Stenosis and atresia occurs when the pulmonary artery walls of blood replacement barrel to the right, often with a defect ventricular septa, and is a manifestation of: A. Fetopathias B. Pathology consequences C. Birth trauma D. *EmbriopathiasE. Perinatal Pathology

537 . Triad Fallouh (defect ventricular septa, pulmonary artery stenosis, right ventricular hypertrophy) is to:

A. Fetopathias B. Pathology consequences C. Birth trauma D. Perinatal Pathology E. *Embriopathias

538 To which the pathology refers tetrad Fallouh (defect ventricular septa, narrowing of the pulmonary artery, dekstrapozition of aorta, right ventricular hypertrophy): A. Fetopathias B. *Embriopathias C. Pathology consequences D. Birth trauma E. Perinatal Pathology

539 Polycystic of liver - multiple cysts of various sizes, found polycystic of the kidneys and pancreas, refers to: A. *Embriopathias B. Fetopathias C. Pathology consequences D. Birth trauma E. Perinatal Pathology

540 . To which the pathology of the liver include polycystic - (multiple cysts of various sizes, found out polycystic kidney and pancreas): A. *Embriopathias B. FetopathiasC. Pathology consequences D. Birth trauma E. Perinatal Pathology

541 . Ageneziya of kidney congenital absence of one or both kidneys is related to A. *EmbriopathiasB. FetopathiasC. Pathology consequences D. Birth trauma E. Perinatal Pathology

542 What pathology refers ageneziya kidney - the absence of one or both kidneys: A. *EmbriopathiasB. Fetopathias C. Pathology consequences D. Birth trauma E. Perinatal Pathology

543 . Renal hypoplasia - reducing weight and volume of the kidney may be one-and two-way: A. Fetopathias B. *Embriopathias C. Pathology consequences D. Birth trauma E. Perinatal Pathology

544 Which renal pathology include renal hypoplasia - reducing weight and volume of the kidney may be one-and two-way: A. Fetopathias B. *Embriopathias C. Pathology consequences D. Birth trauma E. Perinatal Pathology

545 . Displasia renal hypoplasia, with the simultaneous presence of embryonic kidney tissue, refers to: A. Fetopathias B. Pathology consequences C. Birth trauma D. Perinatal Pathology E. *Embriopathias

546 . Congenital emphysema - a dramatic swelling are the top percentage of the left lung with hypoplasia cartilage, elastic and muscular bronchial tissue related to: A. *Fetopathias B. Pathology consequences C. Birth trauma D. Perinatal Pathology E. Embriopathias

547 . Dislocations and dysplasia of the hip joint is related to: A. Pathology consequences B. Birth trauma C. Perinatal Pathology D. EmbriopathiasE. *Fetopathias

548 . Polidaktiliya - Increase the number of fingers is related to: A. *FetopathiasB. Pathology consequences C. Birth trauma D. Perinatal Pathology E. Embriopathias

549 . What pathology refers polidaktiliya - Increase the number of fingers: A. *FetopathiasB. Pathology consequences C. Birth trauma D. Perinatal Pathology E. Embriopathias

550 Perinatal period starts from: A. *196 days fetal life and lasts for up to 1 week of life outside the mother's body B. 197 days fetal life and lasts for up to 1 week of life outside the mother's body C. 198 days fetal life and lasts for up to 1 week of life outside the mother's body D. 199 days fetal life and lasts for up to 1 week of life outside the mother's body E. 200 days fetal life and lasts for up to 1 week of life outside the mother's body

551 When starting the perinatal period: A. *196 days fetal life and lasts for up to 1 week of life outside the mother's body B. 197 days dnya fetal life and lasts for up to 1 week of life outside the mother's body C. 198 days fetal life and lasts for up to 1 week of life outside the mother's body D. 199 days fetal life and lasts for up to 1 week of life outside the mother's body E. 200 days fetal life and lasts for up to 1 week of life outside the mother's body

552 . Infant mortality and childbirth are called: A. Perinatal mortality B. Intranatal C. *Antenatal D. Neonatal E. --

553 Hyaline membrane lung disease refers to: A. Antenatal pathology

B. Intranatal pathology C. Postnatal pathology D. *Pneumopathy E. --

554 What pathology include hyaline membrane lung disease: A. Antenatal pathology B. Intranatal pathology C. Postnatal pathology D. *Pneumopathy E. --

555 Birth injury (damage to tissues and organs of the fetus mechanical forces) occurs: A. Before birth B. *At the time of childbirth C. After childbirth D. As a consequence of childbirth E. --

556 To which the pathology include birth injury (damage to tissues and organs of the fetus mechanical forces): A. Before birth B. *At the time of childbirth C. After childbirth D. As a consequence of childbirth E. --

557 Indicate how pathology relates to birth injury (damage to tissues and organs of the fetus mechanical forces): A. Before birth B. *At the time of childbirth C. After childbirth D. As a consequence of childbirth E. --

558 Causes of fetal birth trauma, set in a position of the fetus during labor is all listed, except: A. High moving vessels B. Lack of blood prothrombin C. The shortage of vitamins C and P D. Softness bone E. *The shortage of vitamin C

559 What pathology include birth injury (damage to tissues and organs of the fetus mechanical forces): A. Before birth B. *At the time of childbirth C. After childbirth D. As a consequence of childbirth E. –

Situational tasks

An autopsy of a 76-year-old male, who smoked for a long period of time, lived sedentary life and had redundant weight, revealed in the intima of the aorta some grey-yellow spots and stripes, fibrous plaques, calcified areas with haemorrhages and calcinosis. What disease do these changes indicate?A. Atherosclerosis B. Hypertensive disease C. Systemic lupus erythematosus D. Visceral syphilisE. Nonspecific aortoarteritisANSWER: A

An autopsy of a 27-year-old male, who died suddenly, revealed in the intima of the abdominal aorta some yellow foci in the form of spots and stripes, which did not rise above the surface of the intima but after staining with sudan III became orange. What stage in the morphogenesis of atherosclerosis was revealed?A. Atherocalcinosis B. Liposclerosis C. Atheroma D. Lipoidosis E. E. PrelipidANSWER: D

A microscopic examination of the wall of an aorta revealed a focal infiltration of the intima by lipids and proteins. The lipids impregnated the intima and accumulated in the muscle cells and macrophages. Determine the stage of atherosclerosis.A. Prelipid B. Liposclerosis C. AtheromatosisD. Atherocalcinosis E. LipoidosisANSWER: E

?The patient died at signs of azotaemic uremia. At the autopsy: enlarged kidney; swollen, red-blooded, easily removable capsule. Cavities of kidney pelvises and cups are enhanced, filled with cloudy urine pus, their membranes are dull, with focuses hemorrhage. At section kidney tissue is mottled, yellow-gray areas are surrounded by a zone of hyperemia and hemorrhage. What disease does correspond to the macroscopic view of the kidneys? A. Acute pyelonephritis B. Acute glomerulonephritis C. Renal amyloidosis D. Chronic pyelonephritis E. Chronic glomerulonephritis ANSWER: A

45 years old man who received treatment for cold with large doses of paracetamol, symptoms of oliguria, azotaemia appeared. After 5 days death occurred at signs of acute renal failure. At histological examination diffuse swelling of the kidney tissue of intermediate renal medulla and its infiltration with lymphocytes, eosinophils, some neutrophils, destruction of tubular epithelium, glomerulars changed insignificantly. What is the most likely disease?

A. Tubule interstitial nephritis B. Acute glomerulonephritis C. Nephrotic syndrome D. Acute pyelonephritis E. NecronephrosisANSWER: A

42 year old man who suffered severe typhoid fever, ARF (acute renal failure) developed, from which he died. At autopsy the kidneys are increased in size, swollen, fibrous capsule is removed easily, at section - cortical substance is pale gray, dark red pyramids. At histological examination in the majority of tubules the lumen is narrowed, epithelial cells increased in size without nuclei; glomerulars are collapsed; in stroma - edema, small leukocyte infiltration, minor hemorrhage. Determine the renal pathology that caused the death of the patient. A. Necronephrosis B. Acute pyelonephritis C. Acute glomerulonephritis D. Pionephros E. Nephrotic syndrome ANSWER: A

In man of 60 years old, who had hypertension for a long time and died of chronic renal failure at autopsy was revealed: both kidneys are greatly reduced in size, the surface is fine-grained; histologically - the majority of glomerulars is hialinised and sclerosed, some glomerulars are - hyperplastic; areas of sclerosis are detected in stroma, arteriolosclerosis and atherosclerosis, elastofibrosis of large branches of renal artery. Name identified changes of kidneys. A. Arteriolosclerotic nephrosclerosis B. Atherosclerotic nephrosclerosis C. Chronic glomerulonephritis D. Chronic pyelonephritis E. Arteriosclerotic kidney ANSWER: A

At the autopsy of 56 years old woman who died of kidney failure, kidney are non-uniformly reduced in size, with macrotuberous surface, at the section pelvises are extended, their walls thickened, scar tissue areas alternating with unmodified parenchyma. Microscopically in the walls of pelvises, cups and interstition – phenomenas of sclerosis and lymphoplasmocytic infiltration. Which of the following diagnoses is the most likely? A. Chronic pyelonephritis B. Acute pyelonephritis C. Acute glomerulonephritis D. Tubulointerstitial nephritis E. Chronic glomerulonephritis ANSWER: A

In a patient who suffered a streptococcal tonsillitis month ago, high blood pressure is identified along with oliguria and oedema of the face. At microscopic examination of renal biopsy material increased in volume, hyperaemic glomerulars are identified, their capillaries and mesangium are diffusely infiltrated by neutrophils. Which is the most likely diagnosis? A. Acute glomerulonephritis, exudative phase B. Acute glomerulonephritis, exudative-proliferative phase C. Acute glomerulonephritis, proliferative phase D. Subacute glomerulonephritis

E. Chronic glomerulonephritis ANSWER: A

At autopsy of person deceased from kidney colic the enlarged flabby kidneys are seen, cortical layer is yellow-gray with the red specks. At microscopic study of biopsy material was revealed that the epithelium of glomerular capsule is proliferating with the formation of "half-moon" capillary loops are necrotised with the fibrin thrombosis in the lumens. Which of the following diagnosis is the most likely? A. Subacute glomerulonephritis B. Acute glomerulonephritis C. Lipoid nephrosis D. Chronic glomerulonephritis E. Renal amyloidosis ANSWER: A

At autopsy a man 45 years old, died of kidney failure were found reduced in size, dense, waxy kidney, with lots of drawings in on their surface. Microscopically: many glomerulars are replaced with positive Congo-rot substance, in other cases - the substance is present in the basal membranes of capillaries, in mesangium and, in the walls of arteries and stroma. Which of the following diagnoses is the most likely? A. Renal amyloidosis B. Acute glomerulonephritis C. Chronic glomerulonephritis D. Subacute glomerulonephritis E. Lipoid nephrosis ANSWER: A

In 68 years old patient bleeding from varicose veins led to significant blood loss. Despite the compensation of hemorrhage the postoperative anuria remained stable and five-day death occurred. At the autopsy kidneys looked like "shocked". Specify the morphological equivalent of acute renal failure. A. Necrotizing nephrosis B. Glomerulonecrosis C. Paraproteinaemic necrosis D. Glomerulosclerosis E. Fibrinoid arteriolonecrosis ANSWER: A

60 years old man for 10 years suffered from chronic osteomyelitis of tibia. Three years ago, appeared nephrotic syndrome. Death occurred from uraemia. At autopsy: moderately reduced kidneys, dense, white with scars in the cortical layer. Name renal pathology that developed. A. periodic disease B. Primary amyloidosis C. Senile amyloidosis D. Secondary amyloidosis E. Idiopathic amyloidosis ANSWER: D

Patient of 49 years hospitalised because pain in the waist. At ultrasound examination in dramatically enhanced renal pelvis and cups of right kidney stones were found. Nephrectomy was performed. At morphological study: kidney is sharply increased, atrophied parenchyma, cups and pelvises are distended, filled with clear yellowish liquid. Microscopically revealed

diffuse sclerosis, tubular and glomerular atrophy saved tubules are distended with cysts. What complications of renal stone disease emerged in the patient? A. Hydronephrosis B. Pionefrosis C. Pyelonephritis D. Glomerulonephritis E. Chronic paranephritisANSWER: A

In the 52-year-old woman with hypertension in medical record, signs of uremia and aedema developed during the last year. At renal biopsy was revealed diffuse glomerular lesions , most of them - with signs and total focal sclerosis and hyalinosis, only in single preserved glomerulas observed expansion mesangium spaces with proliferation of mesangium cells. Diagnose the type of glomerulonephritis. A. Chronic glomerulonephritis fibroplasticB. Chronic mesangiocapillar glomerulonephritisC. Chronic mesangioproliferative glomerulonephritis D. Subacute extracapillar glomerulonephritis E. Acute intracapillar glomerulonephritis ANSWER: A

At autopsy of man 62 years old was revealed: gray color of skin with punctulated hemorrhages, skin as if covered with thin layer of whitish powder, fibrinous-hemorrhagic laryngitis, tracheitis, fibrinous pericarditis, gastritis, enterocolitis. What syndrome is characterized by such complex morphological changes? A. Syndrome of chronic renal failure B. Pituitary Cushing syndrome C. Acute renal failure D. Syndrome of chronic heart failure E. Syndrome of chronic liver failure ANSWER: A

In a patient who suffered from hypertension and died of hemorrhage in the brain, at autopsy small, dense kidneys with fine surface were detected, cortical layer and parenchyma are atrophied. Specify the type of changes of the kidney. A. Contracted kidney B. Amyloid contracted kidneyC. Arteriolosclerotic kidneyD. Piyelonephritic contracted kidneys E. Nephrotic contracted kidneys ANSWER: A

The man of 23 years old fell ill after acute hypothermia. There was high blood pressure, hematuria, swelling of the face. The treatment was ineffective. After 6 months he died of uremia. At the section of kidneys they are increased, flabby, cortical layer of broad, swollen, yellow-gray, dull, with a red specs, well bordereded from crimson brain substance. Microscopically detected proliferation of capsule glomerular epithelium, podocytes and macrophages with formation of a half-moons. Choose a diagnosis. A. Subacute glomerulonephritis B. Acute glomerulonephritis C. Chronic glomerulonephritis D. Acute pyelonephritis

E. Renal amyloidosis ANSWER: A

Patient of 30 years old was admitted to the hospital with complains of headache, fainting. Over the last year high rates of blood pressure are found. In kidney biopsy materials revealed extracapillar epithelial proliferation of capsule glomerulas with formation of "half-moons". For which disease those changes are characteristic? A. Chronic glomerulonephritis B. Acute glomerulonephritis C. Lupoid glomerulonephritis D. Membranous nephropathy E. Subacute glomerulonephritis ANSWER: E

63 years old patient suffered from infectious polyarthritis for a long time, died at signs of kidney failure. Kidneys significantly increased in size, dense, mammiform. At histological examination revealed renal sclerosis intermediate zone and pyramids, a large number of lipids and konhophilic substance in the stroma of tubules. What pathological process complicated of infectious polyarthritis? A. Nephritic amyloidosis B. Nephrotic syndrome C. Nephrosclerosis D. Chronic glomerulonephritis E. Chronic pyelonephritis ANSWER: A

At microscopic study of kidney biopsy material pronounced proliferation of mesangial cells was found with expression of their sprouts at the periphery of capillary loops (interposition of mesangium), diffuse thickening and doubling of the basal membrane of capillaries and accumulation of mesangial matrix. Choose which diagnosis is the most likely? A. Mesangial glomerulonephritis B. Mesangioproliferative glomerulonephritis C. Membranous glomerulonephritis D. Mezanhgiocapillar glomerulonephritis E. Acute intracapillar glomerulonephritis ANSWER: D

At the autopsy of the body of patient with the crush-syndrome enlarged swollen kidneys were found, fibrous capsule is easily removed, discovering pale gray cortical layer, sharply boardered from dark red pyramids. Necrosis of epithelial tubules with signs of tubulorhexis were found, tubular obstruction by detritus and myoglobin crystals, interstitial swelling and its infiltration by neutrophilic leukocytes. Which diagnosis is the most probable? A. Acute glomerulonephritis B. Acute pyelonephritis C. Acute renal failure D. Tubular interstitial nephritis E. Hemolytic kidney ANSWER: C

In a 44 year old patient who is suffering from a chronic kidney disease, in urine stable proteinuria is determined in urine. At microscopic study of kidney biopsy material revealed diffuse, uniform thickening of glomerular capillary basal membranes with formation of

outgrowths towards podocytes (“thorns"), absent or insignificantly expressed proliferation of mesangiocytes, small focuses of sclerosis in medullar layer and fatty dystrophy of epithelium of proximal tubules. Make a diagnosis A. Mesangioproliferative glomerulonephritis B. Mesangiocapillar glomerulonephritis C. Membranous glomerulonephritis D. Lipoid nephrosis E. Focal segmental glomerular sclerosis ANSWER: C

38 years old patient complained of breathlessness, cough with lots of sputum, increasing t ° to 38 ° C, reduced urine output. On the X-ray - focal dark spots of lungs, blood urea and blood uric acid are increased, in urine - low specific gravity, proteinuria, cylinders. Patient died two weeks ago. The fibrinous-necrotic lyarynhotraheobronhitis, fibrinous-haemorrhagic pneumonia were found at autopsy, along with contracted kidney. Indicate which of the following is most likely diagnosis? A. Chronic bronchitis destructive B. Fibrinous-haemorrhagic pneumonia C. Pulmonary complications of influenza D. Chronic glomerulonephritis E. Chronic renal failure ANSWER: E

At electron microscopic examination of kidney biopsy material of 10-year-old boy, who was observed with pronounced edema and proteinuria, was found that in no glomerulas small sprouts of podocytes were absent, thickened capillary basal membranes, mesangium is slightly broadened. In lumens of extended proximal tubules hyaline and granular cylinders were found, tubular epithelium with hyaline-drop phenomenas, hydropic and adipose dystrophy. In the interstitium are many lipids, lipophages Which of the following diagnoses is the most likely? A. Membranous glomerulonephritis B. Subacute glomerulonephritis C. Acute glomerulonephritis D. Adipoid nephrosis E. Mesangiocapillar glomerulonephritis ANSWER: D

At the autopsy of the woman of 60 years old, 15 years suffered from bronchiectasis and died of kidney failure, revealed reduced dense kidney with the "sebaceous appearance” and micronodular surface. At microscopic examination glomerulars are atrophied, replaced by connective tissue. Under adventitious membrane an intima of vessels, in basal membranes of tubules and stroma of kidneys - the accumulation of kongophilic masses. Indicate the most likely diagnosis A. Chronic pyelonephritis B. Adipoid nephrosis C. Renal amyloidosis D. Chronic tubulo-interstitial nephritis E. Healed nephrosclerosis ANSWER: C

During the autopsy woman died of uremia, kidneys of different sizes were found with macrotubercular surface, between the renal capsule and renal surface there are dense adhesions. At microscopic examination the following focal renal changes were identified: Multiple

encapsulated abscesses, expressed limfohistiocytic interstitial infiltration and widening of the connective tissue. Mucous membrane of kidneys with focuses of metaplasia of transitional epithelium into multilayerd. Tubules are partially atrophied, partially distended and filled with colloid-like content Which of the following diagnoses is the most likely? A. Chronic pyelonephritis B. Chronic glomerulonephritis C. Acute pyelonephritis D. Acute glomerulonephritis E. Tubuloyinterstitial nephritis ANSWER: A

After 3 weeks of severe hypothermia in a young man fever began, backpain appeared, oedema, proteinuria, hematuria. Renal biopsy showed increased glomerular size, plethora of capillaries, in the cavity of Shumlyansky-Bowman capsule - eosinophil fluid, fibrin and erythrocytes, mesangium is infiltrated with polymorphonuclear leukocytes. What is the most likely diagnosis? A. Acute pyelonephritis B. Subacute glomerulonephritis C. Acute renal failure D. Acute productive glomerulonephritis E. Acute exudative glomerulonephritis ANSWER: E

A histological examination of a skin biopsy revealed an insignificantly expressed atrophy of the epidermis, hyalinosis of collagenous fibres and scanty perivascular lymphocytic infiltrates in the derma, an expressed interstitial oedema, a loss of transversal striation and microfocal necroses with petrification in the underlying skeletal muscles. Make a diagnosis of the disease.A. Systemic lupus erythematosus B. Scleroderma C. DermatomyositisD. Polyarteritis nodosa E. RheumatismANSWER: C

An autopsy of a 9-year-old child, who suffered from rheumatism and died of heart failure, revealed dilatation of cavities in the ventricles of his heart. Microscopically, the myocardial stroma was characterized by a plethora, oedema, diffuse infiltrations of histiocytes, lymphocytes, neutrophils and eosinophils. What diagnosis was the most probable one?A. Diffuse interstitial exudative myocarditis B. Focal interstitial exudative myocarditis C. Granulomatous productive myocarditis D. Interstitial productive myocarditisE. Alterative myocarditisANSWER: A

A male patient developed substernal pains at 8 a.m., and at 9 a.m. myocardial infarction was diagnosed by ECG data at the admission department. Ten minutes later the patient died. What most reliable sign of myocardial infarction will be found on histological examination?A. Vacuole dystrophy of cardiomyocytes B. Fat infiltration of cardiomyocytesC. Disappearance of glycogen in cardiomyocytes D. Reduced activity of dehydrogenases in fibroblasts E. Relaxation of myofibrils in cardiomyocytes

ANSWER: C

A 75-year-old male was hospitalized complaining of a sharp pain in the abdominal cavity, weakness, filiform pulse. During an operation it was found that the paraaortic fat was imbibed with blood. The abdominal aorta had a sac-like protrusion, its wall was thinned and had an area of rupture. What disease caused the complication?A. Coronary disease B. Atherosclerosis C. Hypertensive disease D. Cardiomyopathy E. Systemic vasculitisANSWER: B

A 67-year-old male patient was suffering from hypertensive disease during 20 years. He died from chronic renal insufficiency. What appearance did his kidneys have on autopsy?A. Large, motley B. Large, red C. Large, white, dense D. Small, dense, macrotuberous E. Small, dense, with a fine-grained surfaceANSWER: E

An autopsy of a male patient, who died from heart failure, revealed an enlarged heart weighing 550 g, fibrinous pericarditis, as well as contracted dense kidneys weighing 50 g each and having a fine-grained surface. Microscopically, the kidneys were characterized by an expressed hyalinosis of arteioles and glomeruli. Name the basic disease.A. Atherosclerosis B. Rheumatism C. Hypertensive disease D. Pericarditis E. CardiomyopathyANSWER: C

An autopsy of a female revealed morphological manifestations of chronic heart failure in the right ventricle, stenosis of the left atrioventricular aperture, insufficiency of the mitral valve. Histologically, a connective-tissue disorganization in the form of some mucoid and fibrinoid swelling was found with presence of blooming Aschoff s bodies against a background of focal cardiosclerosis in the myocardium. Which of the diagnoses listed below was the most probable?A. Scleroderma B. Dermatomyositis C. Polyarteritis nodosa D. Rheumatism E. Systemic lupus erythematosusANSWER: D

Against a background of hypertensive crisis, a male patient with hypertensive disease developed acute renal insufficiency which caused his death. What morphological changes in the renal arteioles were the most probable?A. Stenosing atherosclerosis B. B. Hyperelastosis C. Fibrinoid necrosis D. Hyalinosis

E. SclerosisANSWER: C

A 45-year-old female, who complained of progressing muscular weakness, underwent a biopsy of soft tissues on her shin. A histological examination of the biopsy revealed some microfocal petrification of the derma and skeletal muscles, a reduced amount of glycogen and transversal striation in the muscular fibres, some fibres were necrotized, the stroma was infiltrated by lymphocytes, macrophages and plasma cells. Make a diagnosis of the disease.A. Dermatomyositis B. Systemic scleroderma C. Systemic lupus erythematosus D. Rheumatism E. Polyarteritis nodosaANSWER: A

A histological examination of the cusps of the mitral valve in the heart of a female, who died from cardiac decompensation, revealed a focal desquamation of endotheliocytes with superimposed thrombotic masses in these areas. The connective tissue of the valve cusp had signs of disorganization, areas of sclerosis and angiomatosis. Diagnose the kind of valvular endocarditis.A. Recurrent vegetative B. Diffuse C. Acute vegetative D. Loffler's E. Polypous-ulcerousANSWER: A

A M-yenr-old female suffers from an expressed deformity of joints of her lingers und Iocs. Histologically, the periarticular connective tissue reveals some mucoid swelling, foci of fibrinoid necrosis, clusters of macrophages and areas of sclerosis, the synovial membrane has an oedema of villi, as well as their mucoid and fibrinoid swelling, the synovial cavity contains "rice bodies". Make a diagnosis of the disease.A. Rheumatoid arthritis B. Rheumatism C. C.Bekhterev's disease D. Infectious polyarthritisE. Polyarteritis nodosaANSWER: A

On autopsy of a 68-year-old male, who died from cardiac decompensation, the myocardium of the anterior wall in left ventricle of his heart contained an irregular grey focus, 5 x 4 cm in size, with a dense consistency, fibrous structure and clear borders. What pathological process in the myocardium did the pathologist reveal?A. Myocarditis B. Microfocal cardiosclerosis C. InfarctionD. Postinfarction cardiosclerosis E. RheumatismANSWER: D

An examination of the female, who died from renal insufficiency, revealed that her skin in the regions of the bridge of the nose and lateral surfaces of the face was brown-reddish and

desquamative. On autopsy, the heart was enlarged, the cusps of the aortic valve were thickened, dense and had thrombotic superpositions. The aortic wall had elastolysis and small scars in the middle coat. The kidneys were motley, enlarged, with foci of haemorrhages in their cortical layer. Microscopically, there were haematoxylin bodies in the epithelial nuclei, the basal membranes of capillaries of the glomeruli were thickened and gave an appearance of wire loops, somewhere the capillaries contained hyaline thrombi and foci of fibrinoid necrosis. Make a diagnosis.A. Rheumatism B. Arteriolar nephrosclerosis C. Systemic lupus erythematosus D. Nephropathy amyloidosis E. Arterial nephrosclerosisANSWER: C

An autopsy of a 7-year-old child, who died from progressing heart failure, revealed 200 ml of some semitransparent fluid in the pericardial cavity, the surface of the epicardium had greyish filamentous superpositions. Name the kind of pericarditis.A. Serous B. Haemorrhagic C. Purulent D. Fibroplastic E. FibrinousF. WER:DG. autopsy of a 38-year-old male, who died in a car accident, revealed in his aorta some yellow-grey spots and stripes which did not rise above the surface of the intima. Microscopically, the intima had an abundant deposition of proteins, plasma, fibrin, GAG, cholesterol, low-density lipoproteins; the endothelium had foci of affection. Name the stage of morphogenesis of atherosclerosis. H. PrelipidI. Lipoidosis J. Liposclerosis K. Atheromatosis L. AtherocalcinosisANSWER: B

An autopsy of a 70-year-old male patient, who died from cardiac failure, revealed deformed and narrowed coronary arteries. On section, the inner surface of the arteries was tuberous, the wall was whitish, fragile and stony in consistency. Which of the diagnoses listed below was the most probable?A. Atherocalcinosis B. Liposclerosis C. Atheromatosis D. Lipoidosis E. UlcerationANSWER: A

A clinical examination of a 41-year-old male patient revealed some deformity in small joints of his extremities. A microscopic examination of a biopsy of the synovial coat revealed foci of mucoid and fibrinoid swelling and fibrinoid necrosis in the stroma of villi and vascular walls, proliferation of synoviocytes, a perivascular infiltration by lymphocytes, plasmacytes and neutrophils; there was IgG in the plasmacytes. Which of the diagnoses listed below was the most probable?

A. Polyarthritic form of rheumatism B. Rheumatoid arthritis (stage I) C. Rheumatoid arthritis (stage II) D. Rheumatoid arthritis (stage III) E. Arthritis in systemic lupus erythematosusANSWER: B

Two weeks after angina, a 14-year-old child developed pains in the heart, cyanosis, dyspnoea, oedemata of his lower extremities. The death was caused by cardiac arrest. On autopsy, the heart was enlarged, its cavities were dilated, the myocardium was flaccid. A microscopic examination revealed an acute oedema and plethora of the interstice, an expressed infiltration by lymphocytes, histiocytes, neutrophils and eosinophils, foci of dystrophy of cardiomyocytes. Which of the diagnoses listed below was the most probable?A. Ischaemic dystrophy of myocardium B. Nodular productive myocarditis C. Diffuse interstitial exudative myocarditis D. Focal interstitial exudative myocarditis E. Idiopathic myocarditisANSWER: C

An autopsy of a 70-year-old male, who died of cardiovascular insufficiency mill during his life-time suffered from angina pectoris, hypercholesterolaemia and obesity, revealed a chronic venous plethora of the organs, hypertrophy of the left ventricle of the heart with microfocal cardiosclerosis, voluminous yellow plaques in the intima of the aorta with fine-grained masses in their centre and these masses went deep into the thickness of the wall. Which of the stages of atherosclerosis listed below was the most probable?A. Prelipid B. Lipoidosis C. Liposclerosis D. AtheromatosisE. AtherocalcinosisANSWER: D

A 56-year-old male patient with elevated blood pressure (250/120 mm Hg) died from an impairment of his cerebral circulation. An autopsy of the brain revealed a red focus in the thalamus, 2.5 cm in diameter, which sank on section. Microscopically, there was fibrinoid necrosis of the vascular walls and impregnation of the necrotized brain tissue with blood. Which of the diagnoses listed below was the most probable?A. Cerebral haematoma B. Haemorrhagic infarct of brain C. Anaemic infarct of brain D. Mixed infarctE. AtheromatosisANSWER: B

An autopsy of a male, who suffered from arterial hypertension during his lifetime, revealed oedema of the brain substance, arterial walls at the base were thickened, with white-yellowish plaques in the intima, the left hemisphere had a focus, 5 x 4 x 3 cm in size, representing a cavity filled with liquid blood and its clots. Define the pathological process in the brain.A. Transitory ischaemia B. Haemorrhagic infarct C. Anaemic infarct

D. HaematomaE. Mixed infarctANSWER: D

An autopsy of a 45-year-old female, who died from cardiac failure, revealed on the lateral surfaces of her both cheeks some reddish-brownish spots which fused on the bridge of the nose. The heart was enlarged, the myocardium was flaccid in consistency, the cusps of the aortic valve were thickened and had thrombotic superpositions. The kidneys were motley and had focal haemorrhages. The pyramids of the medullary layer were dark red, the cortical layer was greyish and had red specks. A microscopic examination of the kidneys revealed haematoxylin bodies in the tubular epithelial nuclei, the basal membranes of capillaries of the glomeruli were thickened and gave an appearance of wire loops, somewhere the walls of the capillaries contained hyaline thrombi and foci of fibrinoid necrosis. Which diagnosis was the most probable?A. Rheumatic defect of heart B. Systemic lupus erythematosus C. Septic endocarditisD. Subacute glomerulonephritis E. Atherosclerotic defect of heartANSWER: B

A room for dissections received the body of a 56-year-old male who was ill with rheumatism during 8 years and died from cardiovascular insufficiency. An autopsy revealed the rheumatic defect of the heart -mitral incompetence. A histological examination revealed oedema of the interstice, its diffuse infiltration by lymphocytes, histiocytes, neutrophilic and eosinophilic leukocytes, as well as parenchymatous protein and fatty degeneration of the cardiomyocytes. The left atrial auricle had foci of fibrinoid necrosis surrounded by large macrophages which were located in the form of a veil. Which of the diagnoses listed below was the most probable?A. Diffuse isolated myocarditis B. Focal isolated myocarditis C. Acute serous myocarditisD. Productive granulomatous myocarditis E. Acute purulent myocarditisANSWER: D

On autopsy of a 66-year-old male, who died from acute cardiac failure, an acute venous plethora of the internal organs was found. The cardiac cavities were dilated; a myocardial section revealed some dim yellowish focus, 3.5 x 4 cm in size, in the anterior wall of the left ventricle. The coronary arteries had stenosing atheromatous plaques. Which of the diagnoses listed below was the most probable?A. Myocardial infarction B. Microfocal cardiosclerosis C. Macrofocal cardiosclerosisD. Fatty degeneration of myocardium E. MyocarditisANSWER: A

?A 40-year-old female patient underwent an operation of thyroidectomy. A histological examination of the thyroid tissue revealed that its follicles differed in size, contained some foamy colloid, the follicular epithelium was high and in some places it formed papillae. The stroma of the gland had clusters of lymphocytes which formed follicles with light centres. Make a diagnosis of the disease of the thyroid gland.A. A. Toxic goiter

B. B. Hashimoto's disease C. C. Ligneous thyroiditis D. D. Acute nonsuppurative thyroiditis E. E. Nodular goiterANSWER: A

A histological examination of the thyroid gland revealed a significant infiltration of its tissue by lymphocytes, formation of lymphoid follicles, an atrophy of parenchymatous elements and a significant vegetation of the connective tissue. What disease is characterized by this picture?A. A. Colloid goiter B. B. Endemic goiter C. C. Hashimoto's disease D. D. Diffuse toxic goiter E. E. Parenchymatous goiterANSWER: C

In a young male, an abundant quantity of the somatotropic hormone and enlargement of the nose, lips, ears, lower jaw, hands and feet were revealed. What is your diagnosis?A. A. Pituitary dwarfismB. B. Cushing's diseaseC. C. Addison's diseaseD. D. Adiposogenital dystrophy E. E. AcromegalyANSWER: E

An autopsy of a young female, who died from adrenal insufficiency, revealed diffuse hypermelanosis of the skin, hyperplasia of the cells in islets of Langerhans in the pancreas, the adrenal glands were sharply reduced in size and their thinned cortical substance had foci of necrosis, haemorrhages and sclerosis. What is your diagnosis?A. A. Waterhouse-Friderichsen syndrome B. B. Primary aldosteronism C. C. Cushing's disease D. D. Addison's disease E. E. PheochromocytomaANSWER: D

In a male patient with an increased level of the parathormone, a histological examination in the area of a pathological fracture of his femur revealed foci of a lacunar resolution of the osteoid beams and new formation of a fibrous tissue. What is your diagnosis?A. Multiple myeloma B. Parathyroid osteodystrophy C. OsteoblastoclastomaD. Paget's disease E. OsteopetrosisANSWER: B

A male patient with phenomena of hypothyroidism died from heart failure. On histological examination, his thyroid gland revealed a diffuse infiltration of the gland by lymphocytes and plasmacytes, an atrophy of the parenchyma and a vegetation of the connective tissue. What disease was it?A. Acute nonsuppurative thyroiditis B. Ligneous thyroiditis

C. Hashimoto's diseaseD. Nodular goiterE. Toxic goiterANSWER: C

An autopsy of a 45-year-old female, who was suffering from arterial hypertension, diabetes mellitus and ovarian dysfunction during past 15 years, revealed obesity by the upper type, a pituitary basophil adenoma in the anterior lobe of the hypophysis, hyperplasia of the adrenal cortex. Which of the diagnoses listed below was the most probable?A. Cushing's disease B. Cushing's syndromeC. Hypertensive diseaseD. Addison's diseaseE. Adiposogenital dystrophyANSWER: A

A 50-year-old female took medical advice complaining of excretion of a large amount of urine and excessive thirst. On examination, her nourishment was reduced, the skin was dry, density of the urine ranged from 1001 to 1010, data of an ultrasound examination and computed tomography of the brain revealed a tumour in the posterior lobe of the hypophysis. Indicate the most probable disease.A. Acromegaly B. Babinsky-Frelich disease C. Diabetes insipidusD. Simmonds disease E. Cushing's diseaseANSWER: C

A 46-year-old male patient, who suffered from bulimia, polydipsia, polyuria with glucosuria and albuminuria, died of renal insufficiency. On autopsy, the kidneys were reduced in size, dense and had a fine-grained surface. The pancreas was reduced and partially substituted for a fatty tissue. Microscopically, islets of Langerhans were fine, in some places they were substituted for a connective tissue, solitary ones were hypertrophic. The kidneys reveal intracapillary glomerulosclerosis. Which of the diagnoses was the most probable? A. Subacute glomerulonephritis B. Diabetes mellitus C. Chronic indurative pancreatitis D. Diabetes insipidus E. Chronic glomerulonephritisANSWER: B

A 36-year-old female patient underwent resection of the both lobes of her thyroid gland; each of them was 5 x 6 cm in size, pink-yellow, moderately dense and had a tuberous surface. A microscopic examination revealed follicles of various size, some of them were dilated like cysts and filled with some colloid; the follicular walls were lined with the smoothed cuboidal epithelium; the stroma of the gland was redundantly developed owing to the connective tissue, there were foci of calcinosis. Which of the diseases listed below corresponded most to the changes found? A. Parenchymatous goiterB. Colloid goiterC. Toxic goiter D. Hashimoto's disease

E. Ligneous thyroiditisANSWER: B

An autopsy of a 24-year-old female (from her case history it is known that a year before the woman had given birth to a child) revealed a sharp decrease of the body weight down to 38 kg, the skin was dry and thin, the weight of the internal organs was lowered. Also, there was a sharp decrease in the weight of the adenohypophysis, the latter had cicatrices; there were foci of dystrophy, necrobiosis and hyalinosis in the diencephalon. The ovaries, thyroid and adrenal glands had phenomena of hypotrophy, the mucous membrane of the intestines was atrophied. Which of the diagnoses was the most probable?A. Nutritional dystrophy B. Suprarenal cachexia C. Cachexia associated with chronic amoebiasisD. Cachexia associated with pellagraE. Cerebrohypophysial cachexiaANSWER: E

An autopsy of a 45-year-old female patient, who suffered from obesity by the upper type, steroid diabetes mellitus, arterial hypertension and secondary ovarian dysfunction, revealed hypertrichosis, hirsutism, striae on the skin of the thighs and abdomen. The anterior lobe of the hypophysis contained a white-pink encapsulated tumour, 2.5 cm in diameter (microscopically, it was a pituitary basophil adenoma); the adrenal glands were characterized by bilateral hyperplasia of the fascicular layer. Which of the diagnoses was the most probable? A. Cushing's syndrome B. Adiposogenital dystrophyC. Cushing's diseaseD. Simmonds disease E. Pituitary dwarfismANSWER: C

A 52-year-old male died from renal insufficiency. On microscopic examination of his organs, the pancreas revealed lipomatosis and sclerosis with an atrophy of islets of Langerhans, the kidneys had hyalinosis of the mesangium and glomeruli (Kimrnelstiel-Wilson syndrome) and a glycogenic infiltration of the epithelium of the tubules, the liver was characterized by fatty degeneration. Which of the diagnoses listed below was the most probable? A. Diabetic glomerulosclerosisB. Arterial nephrosclerosis C. Amyloid shrunk kidneys D. Chronic glomerulonephritis E. Goodpasture's syndromeANSWER: A

An autopsy of a male revealed a tumour in the anterior lobe of the hypophysis, enlarged adrenal glands, a reduction of the gonads in size, a hypertrophy of the left cardiac ventricle, the pancreas was reduced in size and thickened. Histologically, there was a pituitary basophil adenoma and a hyperplasia of the cortical layer in the adrenal glands. The pancreas was characterized by a moderately expressed atrophy of the parenchyma, including islets of Langerhans. What disease did the patient suffer from?A. Diabetes mellitus B. Adiposogenital dystrophy C. Cushing's diseaseD. Cushing's syndrome

E. Simmonds diseaseANSWER: C

For a histological examination, a lobe and a part of the isthmus of the thyroid gland were received. The tissue of the gland was dense and tuberous, on section it was pale brown and had grey-whitish foci. Microscopically, against a background of an atrophy of the follicles of the gland, there was some diffuse lymphoplasmacytic infiltration of the stroma with formation of lymphoid follicles. What pathological process were these changes typical for? A. Toxic goiter B. Allergic thyroiditisC. Thyroid adenoma D. Colloid goiter E. Sporadic goiterANSWER: B

An autopsy of a male, who died from chronic renal insufficiency, revealed atherosclerosis of the aorta and large arteries, small and dense kidneys with a finegrained surface, an enlarged yellow-brown and flaccid liver, the pancreas was reduced in size. Microscopically, there was atherocalcinosis of the aorta and arteries, an atrophy of the parenchyma, sclerosis and lipomatosis of the pancreas; the kidneys were characterized by hyalinosis of the mesangium and glomeruli, a glycogenic infiltration of the epithelium of the tubules, with large-drop adiposis in the hepatocytes. What pathological process took place in the kidneys? A. Diabetic nephrosclerosis B. Arterial nephrosclerosis C. Chronic pancreatitisD. Chronic glomerulonephritis E. SteatosisANSWER: A

On autopsy of a male, who died from uraemia, it was found that the pancreas was reduced in size, his contracted kidneys had a fine-grained surface, the liver was enlarged, yellow and flaccid. Microscopically, the pancreatic tissue revealed an atrophy of the parenchyma, including islets of Langerhans, the atrophied parenchyma was substituted for hyperplastic connective and fatty tissues. The kidneys were characterized by sclerosis and hyalinosis of the glomeruli, as well as by a glycogenic infiltration of the tubules; there was a fatty degeneration in the liver and a fibrinous inflammation in the mucous coats of the trachea, bronchi and stomach. What disease did the died person suffer from?A. Chronic indurative pancreatitis B. Chronic glomerulonephritis C. Hypertensive disease D. Diabetes mellitus E. SteatosisANSWER: D

A histological examination of a thyroid gland revealed follicles of various size and shape which were lined with the columnar epithelium; the latter proliferated and formed papillae of various size. The follicular lumens contained some liquid and vacuolized colloid. The stroma of the gland was characterized by a lymphoplasmacytic infiltration, in some places with formation of lymphatic follicles having light centres. Which of the diagnoses was the most probable?A. Colloid goiter B. Nodular goiter C. Hashimoto's disease

D. Ligneous thyroiditis E. Toxic goiterANSWER: E

An autopsy of a 48-year-old male, who died from vascular collapse, revealed an increased pigmentation of the skin, the adrenal glands were reduced in size, the brown-yellow liver was enlarged. On histological examination, foci of necrosis with a tuberculous granulation tissue were found in the adrenal glands. The liver was characterized by phenomena of fatty degeneration. Which of the diagnoses was the most probable?A. Addison's disease B. Steatosis C. Primary aldosteronism D. Cushing's syndromeE. LipofuscinosisANSWER: A

An autopsy of a 45-year-old male, who suffered from numerous pathological fractures during his lifetime, revealed changes in his long tubular bones: the bones of the thigh and shin were bent, in some places they resembled spirals, their surface was tuberous, a section revealed an obliterated medullary channel and a change in the compact structure of the cortical layer by the spongy type. Microscopically, there was a mosaic type of the bone structures: against a background of a disordered thin-fibrous or lamellar structure of the bone fragments there were numerous cavities of sinusal resorption combined with signs of new formation of the osseous tissue. The arteries, which supplied the bone tissue, were dilated and convoluted. Name a diagnosis.A. Deforming osteodystrophy B. Osteopetrosis C. Parathyroid osteodystrophy D. Fibrous dysplasiaE. Chronic osteomyelitisANSWER: A

?Autopsy of a woman, who died from renal insufficiency, revealed in her spinal column, cranial bones and ribs some defects of the osseous tissue with tumour nodes on their margins. The kidneys were enlarged, dense and "greasy" on section. Microscopically, the tumour nodes and bone marrow were characterized by a proliferation of tumour cells of the plasmacytic line. Which of the diseases listed below corresponds to the description?A. Metastasis of pulmonary carcinoma into bonesB. Osteosarcoma C. Multiple myelomaD. Osteoporosis E. OsteomyelitisF. WER CG. istological examination of an enlarged lymph node revealed a proliferation of lymphocytes, histiocytes, reticular cells, acidophilic leukocytes, small and large Hodgkin's cells, multinuclear Reed-Sternberg cells. Which of the diseases listed below do the described morphological data correspond to?H. Lymphosarcoma I. Metastasis of carcinoma J. Chronic leukaemia K. Acute leukaemiaL. LymphogranulomatosisM. WER E

N. autopsy of a female, who suffered from some blood disease (in her clinical blood analysis leukocytosis achieved 100 x 109), revealed a pyoid bone marrow (microscopically, it had myelocytes, promyelocytes and blast cells), an enlarged spleen weighing up to 7 kg (microscopically, it had ischaemic infarcts and an infiltration of the pulp by myelocytes), an enlarged liver weighing up to 6 kg (microscopically, it had a diffuse leukaemic infiltration of the sinusoids). Name the diagnosis which was the most probable one of those listed below.O. Acute granulocytic leukaemiaP. Chronic granulocytic leukaemiaQ. Acute stem cell leukaemiaR. Acute lymphocytic leukaemia S. Multiple myelomaT. WER BU. istological examination of an enlarged cervical lymph node revealed the following microscopic signs: proliferation of the lymphoid cells with various degrees of maturity, presence of giant Hodgkin's and Reed-Sternberg cells, as well as eosinophils, plasma cells and neutrophilic leukocytes, among which there were foci of necrosis and fibrosis. Which of the variants of lymphogranulomatosis listed below was the most probable?V. With prevalence of the lymphoid tissueW. Nodular sclerosis X. Mixed-cell variant Y. With suppression of the lymphoid tissueZ. Hodgkin's sarcoma[. WER C\. racotomy in a 55-year-old male patient revealed a packet of lymph nodes in the anterior mediastinum; a biopsy was taken from one of them. Microscopically, there were infiltrates consisting of lymphocytes, histiocytes, eosinophils and Reed-Stemberg multinuclear cells which were surrounded by vegetations of a fibrous connective tissue. Name the clinical-morphological form of lymphogranulomatosis.]. A.Lymphogranulomatosis with suppression of the lymphoid tissue ^. Mixed-cell variant of lymphogranulomatosis _. Lymphogranulomatosis, nodular sclerosis `. Lymphogranulomatosis with prevalence of the lymphoid tissuea. -b. WER Cc. autopsy of a person, who died at a haematological department, revealed numerous haemorrhages in the skin, mucous and serous membranes, enlarged dark-grey tonsils. The lymph nodes of different localization were up to 1.5 cm in diameter, soft, and grey-pink on section. The bone marrow of the femur was dark red. Microscopically, the lymph nodes, liver, spleen and kidneys contained leukaemic infiltrates consisting of lymphoblasts. It was known that during the life-time the clinical blood analysis showed up to 3 x 10l2/l of erythrocytes, 100 x 109/l of leukocytes, a lot of lymphoblasts with presence of mature forms and absence of prolymphocytes. Which of the diagnoses listed below was the most probable?d. Lymphogranulomatosis e. Chronic lymphocytic leukaemiaf. Acute lymphocytic leukaemia g. Lymphosarcoma h. Chronic granulocytic leukaemiai. WER Cj. autopsy of a male, who suffered from frequent fractures of his bones and died from uraemia, revealed phenomena of osteoporosis and multiple smooth-walled defects (as if produced by punching) in the bones of the skull, ribs and spine. A microscopic examination of the bone

marrow revealed its diffuse infiltration by tumour cells of the lymphoplasmacytic line. Which of the diagnoses listed below was the most probable?k. Multiple myeloma l. Primary macroglobulinaemia m. Heavy-chain disease n. Paget's disease o. Recklinghausen's diseasep. WER Aq. autopsy of a male, who died from chronic renal insufficiency, revealed numerous nodes with soft elastic consistency in the ribs, bones of the vault of the skull and the breastbone. The osseous substance was decalcified according to the nodes. The kidneys were enlarged, light grey, dense, their section had some greasy lustre. What is your diagnosis?r. Primary amyloid nephropathy s. Parathyroid osteodystrophy t. Multiple myeloma u. Osteoma v. Osteosarcomaw. WER Cx. umour was found in the locus of a pathological fracture of a rib in a male patient. The case history contained information about persistent proteinuria with presence of abnormal proteins of Bence-Jones type, as well as presence of osteolytic foci in the bones of the spine, skull and pelvis. Histologically, the tumour cells were represented by plasmablasts and plasmacytes. What is your diagnosis?y. Primary macroglobulinaemia z. Heavy-chain disease {. Osteosarcoma |. Multiple myeloma }. Fibrosarcoma~. WER D. ale patient, who worked for a long period of time with benzene, develops progressing anaemia �and the haemorrhagic syndrome. A biopsy of his breastbone reveals prevalence of a fatty tissue, and there are some small islets of haemopoiesis with solitary cells of myelopoiesis. What is your diagnosis?Ђ. Chronic myeloleukaemia Ѓ. Pernicious anaemia ‚. Haemolytic anaemia ѓ. Hypoplastic anaemia „. Aplastic anaemiaANSWER: D

An X-ray examination of a male patient revealed numerous foci of osteoporosis and osteolysis in his flat bones. A high content of tumour plasma cells was found in a trepanobiopsy. What is your diagnosis?A. Acute monocytic leukaemiaB. Chronic myeloleukaemiaC. Osteosarcoma D. Multiple myeloma E. FibrosarcomaANSWER: D

A death of a 7-year-old boy resulted from acute posthaemorrhagic anaemia caused by a profuse bleeding from the gastrointestinal tract. A postmortem examination revealed: macroscopically -

an anaemia of the internal organs, an enlargement of lymph nodes in different groups, thymomegaly, a moderately manifested hepatosplenomegaly, a bright red bone marrow; microscopically - a hypercellular bone marrow with some monomorphous infiltrate of blast cells, diffuse-focal tumour infiltrates in the liver, spleen, lymph nodes, meninges and substance of the brain. Make a diagnosis for this form of leukaemia.A. Acute lymphoblastic B. Acute myeloblastic C. Acute stem cell D. Acute monoblastic E. Acute plasmablasticANSWER: A

A 70-year-old male patient with an expressed hepatosplenomegaly and cachexia underwent a diagnostic puncture biopsy of his liver. A histological examination revealed that along the portal tracts there were numerous infiltrates of monomorphous round cells verified as prolymphocytes and B-lymphocytes. What disease are the above changes characteristic of?A. Lymphosarcoma B. B.Acute lymphoplastic leukaemia C. Lymphogranulomatosis D. Chronic lymphatic leukaemiaE. Cesari's diseaseANSWER: D

The patient who long worked with benzene, progressing anemia and hemorrhagic syndrome. In the biopsy of sternum predominant adipose tissue, revealed a few small foci with isolated blood cells myelopoiesis. Your diagnosis. A. Chronic myeloid leukemia B. Pernicious anemia C. Hemolytic anemia D. Anemotrophy E. Iron deficiency anemia ANSWER: D

?Autopsy of a woman, who died from renal insufficiency, revealed in her spinal column, cranial bones and ribs some defects of the osseous tissue with tumour nodes on their margins. The kidneys were enlarged, dense and "greasy" on section. Microscopically, the tumour nodes and bone marrow were characterized by a proliferation of tumour cells of the plasmacytic line. Which of the diseases listed below corresponds to the description?A. Metastasis of pulmonary carcinoma into bonesB. Osteosarcoma C. Multiple myelomaD. Osteoporosis E. OsteomyelitisANSWER: C

A histological examination of an enlarged lymph node revealed a proliferation of lymphocytes, histiocytes, reticular cells, acidophilic leukocytes, small and large Hodgkin's cells, multinuclear Reed-Sternberg cells. Which of the diseases listed below do the described morphological data correspond to?A. Lymphosarcoma B. Metastasis of carcinoma C. Chronic leukaemia

D. Acute leukaemiaE. LymphogranulomatosisANSWER: E

An autopsy of a female, who suffered from some blood disease (in her clinical blood analysis leukocytosis achieved 100 x 109), revealed a pyoid bone marrow (microscopically, it had myelocytes, promyelocytes and blast cells), an enlarged spleen weighing up to 7 kg (microscopically, it had ischaemic infarcts and an infiltration of the pulp by myelocytes), an enlarged liver weighing up to 6 kg (microscopically, it had a diffuse leukaemic infiltration of the sinusoids). Name the diagnosis which was the most probable one of those listed below.A. Acute granulocytic leukaemiaB. Chronic granulocytic leukaemiaC. Acute stem cell leukaemiaD. Acute lymphocytic leukaemia E. Multiple myelomaANSWER: B

A histological examination of an enlarged cervical lymph node revealed the following microscopic signs: proliferation of the lymphoid cells with various degrees of maturity, presence of giant Hodgkin's and Reed-Sternberg cells, as well as eosinophils, plasma cells and neutrophilic leukocytes, among which there were foci of necrosis and fibrosis. Which of the variants of lymphogranulomatosis listed below was the most probable?A. With prevalence of the lymphoid tissueB. Nodular sclerosis C. Mixed-cell variant D. With suppression of the lymphoid tissueE. Hodgkin's sarcomaANSWER: C

Thoracotomy in a 55-year-old male patient revealed a packet of lymph nodes in the anterior mediastinum; a biopsy was taken from one of them. Microscopically, there were infiltrates consisting of lymphocytes, histiocytes, eosinophils and Reed-Stemberg multinuclear cells which were surrounded by vegetations of a fibrous connective tissue. Name the clinical-morphological form of lymphogranulomatosis.A. A.Lymphogranulomatosis with suppression of the lymphoid tissue B. Mixed-cell variant of lymphogranulomatosis C. Lymphogranulomatosis, nodular sclerosis D. Lymphogranulomatosis with prevalence of the lymphoid tissueE. -ANSWER: C

An autopsy of a person, who died at a haematological department, revealed numerous haemorrhages in the skin, mucous and serous membranes, enlarged dark-grey tonsils. The lymph nodes of different localization were up to 1.5 cm in diameter, soft, and grey-pink on section. The bone marrow of the femur was dark red. Microscopically, the lymph nodes, liver, spleen and kidneys contained leukaemic infiltrates consisting of lymphoblasts. It was known that during the life-time the clinical blood analysis showed up to 3 x 10l2/l of erythrocytes, 100 x 109/l of leukocytes, a lot of lymphoblasts with presence of mature forms and absence of prolymphocytes. Which of the diagnoses listed below was the most probable?A. Lymphogranulomatosis B. Chronic lymphocytic leukaemiaC. Acute lymphocytic leukaemia

D. Lymphosarcoma E. Chronic granulocytic leukaemiaANSWER: C

An autopsy of a male, who suffered from frequent fractures of his bones and died from uraemia, revealed phenomena of osteoporosis and multiple smooth-walled defects (as if produced by punching) in the bones of the skull, ribs and spine. A microscopic examination of the bone marrow revealed its diffuse infiltration by tumour cells of the lymphoplasmacytic line. Which of the diagnoses listed below was the most probable?A. Multiple myeloma B. Primary macroglobulinaemia C. Heavy-chain disease D. Paget's disease E. Recklinghausen's diseaseANSWER: A

An autopsy of a male, who died from chronic renal insufficiency, revealed numerous nodes with soft elastic consistency in the ribs, bones of the vault of the skull and the breastbone. The osseous substance was decalcified according to the nodes. The kidneys were enlarged, light grey, dense, their section had some greasy lustre. What is your diagnosis?A. Primary amyloid nephropathy B. Parathyroid osteodystrophy C. Multiple myeloma D. Osteoma E. OsteosarcomaANSWER: C

A tumour was found in the locus of a pathological fracture of a rib in a male patient. The case history contained information about persistent proteinuria with presence of abnormal proteins of Bence-Jones type, as well as presence of osteolytic foci in the bones of the spine, skull and pelvis. Histologically, the tumour cells were represented by plasmablasts and plasmacytes. What is your diagnosis?A. Primary macroglobulinaemia B. Heavy-chain disease C. Osteosarcoma D. Multiple myeloma E. FibrosarcomaANSWER: D

A male patient, who worked for a long period of time with benzene, develops progressing anaemia and the haemorrhagic syndrome. A biopsy of his breastbone reveals prevalence of a fatty tissue, and there are some small islets of haemopoiesis with solitary cells of myelopoiesis. What is your diagnosis?A. Chronic myeloleukaemia B. Pernicious anaemia C. Haemolytic anaemia D. Hypoplastic anaemia E. Aplastic anaemiaANSWER: D

An X-ray examination of a male patient revealed numerous foci of osteoporosis and osteolysis in his flat bones. A high content of tumour plasma cells was found in a trepanobiopsy. What is your diagnosis?A. Acute monocytic leukaemiaB. Chronic myeloleukaemiaC. Osteosarcoma D. Multiple myeloma E. FibrosarcomaANSWER: D

A death of a 7-year-old boy resulted from acute posthaemorrhagic anaemia caused by a profuse bleeding from the gastrointestinal tract. A postmortem examination revealed: macroscopically - an anaemia of the internal organs, an enlargement of lymph nodes in different groups, thymomegaly, a moderately manifested hepatosplenomegaly, a bright red bone marrow; microscopically - a hypercellular bone marrow with some monomorphous infiltrate of blast cells, diffuse-focal tumour infiltrates in the liver, spleen, lymph nodes, meninges and substance of the brain. Make a diagnosis for this form of leukaemia.A. Acute lymphoblastic B. Acute myeloblastic C. Acute stem cell D. Acute monoblastic E. Acute plasmablasticANSWER: A

A 70-year-old male patient with an expressed hepatosplenomegaly and cachexia underwent a diagnostic puncture biopsy of his liver. A histological examination revealed that along the portal tracts there were numerous infiltrates of monomorphous round cells verified as prolymphocytes and B-lymphocytes. What disease are the above changes characteristic of?A. Lymphosarcoma B. B.Acute lymphoplastic leukaemia C. Lymphogranulomatosis D. Chronic lymphatic leukaemiaE. Cesari's diseaseANSWER: D

The patient who long worked with benzene, progressing anemia and hemorrhagic syndrome. In the biopsy of sternum predominant adipose tissue, revealed a few small foci with isolated blood cells myelopoiesis. Your diagnosis. A. Chronic myeloid leukemia B. Pernicious anemia C. Hemolytic anemia D. Anemotrophy E. Iron deficiency anemia ANSWER: D

?An autopsy of 78-year-old male patient, who died from cardiopulmonary insufficiency, revealed an enlarged right lung with massive fibrinous superpositions on the pleura. Histologically, the alveolar lumen had accumulations of fibrin and neutrophils. The lymph nodes in the lung roots were pale pink and somewhat enlarged. Name the stage of croupous pneumonia. A. Red hepatizationB. B.InfluxC. Grey hepatization

D. ResolutionE. EdemaANSWER: C

An autopsy of a 45-year-old male patient, who had had double bronchopneumonia and died under the phenomena of intoxication, revealed in the lower lobe of the right lung some thick-walled cavity, 4 cm in diameter, filled with liquid yellowish masses. What pathological process complicated the course of pneumonia?A. A.TuberculomaB. GangreneC. AbscessD. Sequester E. EmpyemaANSWER: B

During past 3 years a 45-year-old male suffered from a dry cough, dyspnoea, loss of weight. His death occurred under the phenomena of growing pulmonary insufficiency. An autopsy revealed hypertrophy of the right ventricle, the lungs had sac-like ectasiae of the bronchi with formation of cavities having uneven edges and containing some purulent exudate against a background of expressed fibrosis. Histologically, the bronchial walls and pulmonary interstice had a chronic inflammation and fibrosis. What is your diagnosis?A. Bronchial asthma B. Chronic bronchitis C. Chronic bullous emphysemaD. Fibrosing alveolitisE. Bronchiectatic diseaseANSWER: E

A 38-year-old female patient died during an uncontrolled attack of bronchial asthma. A histological examination revealed accumulations of mucus in the lumen of the bronchi, their wall had a large number of mast cells (labrocytes) and most of them were in the state of degranulation, as well as there were a lot of eosinophils. What was the mechanism of development of these changes in the bronchi?A. Anaphylactic reactionB. Cytolytic effect of antibodies C. Immunocomplex reactionD. Cell-dependent cytolysis E. GranulomatosisANSWER: A

A severe form of influenza (a viral strain of AB influenza) in a 57-year-old male resulted in a lethal outcome. On autopsy, enlarged "motley" lungs were found. A microscopic examination revealed a sharp plethora of vessels, an oedema and haemorrhages in the pulmonary parenchyma, the lumens of the bronchi A. Haemorrhagic bronchopneumoniaB. Catarrhal bronchopneumonia C. Purulent bronchopneumonia D. Desquamative bronchopneumoniaE. Fibrinous pneumoniaANSWER: A

An autopsy of a male patient, who died from cancerous intoxication, revealed some grey-yellow dense irregular foci which were located in the posteroinferior portions of the lungs and rose above the surface of a section. Microscopically, the lumens and walls of small bronchi, as well as alveoli, revealed some exudate containing a lot of heterophilic leukocytes. What complication developed in the patient?A. Acute pulmonary bronchopneumoniaB. Acute pulmonary bronchitis C. Croupous pneumonia D. Acute alveolitis E. Acute serous bronchopneumoniaANSWER: A

A histological examination of a biopsy from a bronchial wall revealed in its mucous layer some focal squamous metaplasia of the columnar epithelium, an increased number of goblet cells, a vegetation of the granulation tissue which rose above the surface of the mucous layer and contained some diffuse inflammatory infiltrate consisting of lymphocytes, plasma cells and histiocytes. What is your diagnosis?A. Chronic mucopurulent bronchitisB. Chronic mucous bronchitis C. Chronic purulent bronchitisD. Chronic deforming bronchitisE. Chronic polypous bronchitisANSWER: D

An autopsy of a 55-year-old male, who died from cardiopulmonary insufficiency, revealed thickened deformed bronchi filled with some mucopurulent exudate. Microscopically, an inflammatory infiltration, foci of squamous metaplasia of the epithelium and an increased number of goblet cells were found in the bronchi. What is your diagnosis?A. Chronic bronchitisB. BronchopneumoniaC. Bronchiectatic disease D. Acute bronchitis E. Fibrinous bronchitisANSWER: A

On autopsy, 380 ml of some yellow fluid with an unpleasant odour were found in the right pleural cavity. Microscopically, the liquid contained a lot of neutrophilic granulocytes. What is your diagnosis?A. Pulmonary abscess B. PhlegmonC. Pleural empyema D. Pulmonary gangreneE. Purulent catarrh of pleuraANSWER: C

An autopsy of a 34-year-old male, who died from cardiopulmonary insufficiency, revealed an enlarged grey dense air-free lower lobe of the left lung, the vincernl pleura of this lobe had superpositions of fibrin. Microscopically, the lumens of alveoli revealed some fibrinous-leukocytic exudate. Name the stage of croupous pneumonia.A. InfluxB. Red hepatization C. Resolution

D. Grey hepatizationE. EdemaANSWER: D

Following a severe supercooling, a male patient developed a fever, a cough, pains in the right half of his chest that increased on breathing. The patient died on the 5th day of the disease under the phenomena of acute cardiac failure. On autopsy, the lower lobe of his right lung was thickened, the surface of a section was grey and fine-grained, the pleura was covered with some pale grey film. Histologically, the lumens of alveoli contained fibrin and a large amount of heterophilic leukocytes. Make a diagnosis.A. Bronchopneumonia B. Interlobular pneumoniaC. Croupous pneumoniaD. Interalveolar pneumonia E. PeribronchopneumoniaANSWER: B

An autopsy of a male patient, who died from cardiopulmonary insufficiency, revealed some dim liquid with grey flakes in the pleural cavities, the lungs were enlarged, air-free and had some grey-whitish films on the pleura. On section, the lungs were white-greyish and dense, their surface was granular. The peribronchial lymph nodes were enlarged and whitish-grey on section. Microscopically, the alveoli contained some fibrinous-leukocytic exudate. Name the stage of croupous pneumonia.A. Grey hepatizationB. Red hepatization C. Influx D. ResolutionE. Caseous pneumoniaANSWER: A

An autopsy of a male, who suffered from right-sided pneumonia, revealed in the right lung some cavity 3.5 cm in diameter, that had uneven edges, communicated with the bronchus and was filled with some purulent exudate. Microscopically, the wall of the cavity was formed by the granulation tissue diffusely infiltrated by leukocytes. Which of the diagnoses, listed below, was the most probable? A. Acute pneumogenic abscessB. Acute bronchiogenic abscess C. Chronic bronchiogenic abscessD. Chronic pneumogenic abscess E. Pulmonary echinococcosisANSWER: B

A 72-year-old male patient was complaining of an excruciating cough with production of some purulent sputum and dyspnoea. The death was caused by cardiopulmonary insufficiency. On autopsy, the lumens of the bronchi contained purulent sputum, the bronchial walls were thickened, there was hypertrophy of right chambers of the heart. Microscopically, the bronchial mucosa was atrophic and had a large number of goblet cells, the bronchial glands were dilated like cysts, the walls of the bronchi were diffusely infiltrated by leukocytes, there was some peribronchial and perivascular vegetation of the connective tissue. Which of the diagnoses was the most probable?A. Deforming bronchitis B. Chronic purulent bronchitis

C. Chronic polypous bronchitisD. Bronchiectatic diseaseE. –ANSWER: B

An autopsy of a male, who suffered from right-sided pneumonia in the lower lobe during his life-time and for a long period of time expectorated sputum of a purulent character, revealed some cavity with dense edges that was located in the 9th-10th segments of the lung and was filled with yellowish cream-like masses. There was some whitish path from the cavity to the root of the lung. Microscopically, the cavity was separated from the intact pulmonary tissue with a membrane which consisted of a fibrous connective tissue from the outside and a granulation one from inside. Which of the diagnoses was the most probable?A. Bronchiectatic disease B. Pulmonary gangrene C. Acute pulmonary abscess D. Chronic pneumoniaE. Chronic abscessANSWER: E

An autopsy of a male, who suffered from chronic bronchitis for a long period of time and several episodes of acute pneumoniae, revealed dense lungs which had a little of air and were "honeycomb" on section. The bronchial walls were thickened and with sac-like protrusions containing purulent masses. The weight of the heart was 520 g. The thickness of the right ventricular wall was 2 cm, that of the left one 1.2 cm, the cavity of the right ventricle was dilated. Which of the diagnoses, listed below, was the most probable?A. Bronchiectatic diseaseB. Chronic obstructive emphysemaC. Chronic pneumonia D. Chronic bronchitis E. Pulmonary gangreneANSWER: A

An autopsy of a 65-year-old female, who died from cardiopulmonary insufficiency, revealed that the lungs were enlarged, pale, oedematous, covered the mediastinum, did not collapse and were cut with a crunch. On pressure, the lumens of the bronchi discharged some mucus of a purulent character. Which of the diagnoses, listed below, was the most probable?A. Bronchiectatic diseaseB. Bronchial asthma C. Chronic diffuse obstructive emphysemaD. Chronic pneumonia E. Chronic bronchitisANSWER: C

?A 7 year old child had an acute onset of disease. Pediatrician stated that mucous membrane of fauces is hyperemic and covered with a lot of mucus. Mucous membrane of cheeks has whitish stains. Next day the child's skin of face, neck, body was covered with coarsely-papular rash. What disease may be presumed?A. Allergic dermatitis B. Diphteria C. Scarlet fever D. MeaslesE. Meningococcemia

ANSWER: D

Microscopic analysis of tissue sampling from patient's skin reveals granulomas that consist of epithelioid cells surrounded mostly by T-lymphocytes. Among epithelioid cells there are solitary giant multinuclear cells of Pirogov-Langhans typ. In the centre of some granulomas there are areas of caseous necrosis. Blood vessels are absent. What disease are the described granulomas typical for? A. TuberculosisB. Syphilis C. Leprosy D. Rhinoscleroma E. Glanders ANSWER: A

A patient consulted a doctor about the intensive skin itch, especially between fingers, in axillary creases, in the inferior part of belly. During the skin examination there were found twisting whitish tracts with speckles at the end of them. What disease are these clinical presentations typical for? A. Dermatotropic leishmaniosis B. ScabiesC. Demodicosis D. Pediculosis E. Miasis ANSWER: B

Autopsy of an 8 year old boy who was ill with pharyngeal and tonsillar diphtheria and died one week after illness begin revealed myocardial changes in form of small-focal myocardiocyte necroses, stroma edema with slight lymphocytic infiltration. What type of myocarditis is it: A. Septic B. AlternativeC. Granulomatous D. Focal-intermediate, exudative E. Interstitional ANSWER: B

A 7 year old child was taken to the infectious disease hospital with complaints of acute pain during swallowing, temperature rise up to 390С, neck edem. Objective signs: tonsills are enlarged, their mucous membrane is plethoric and covered with a big number of whitish-yellowish films that are closely adjacent to the mucous membran. After removal of these films the deep bleeding defect remains. What type of inflammation is it? A. Purulent B. Crupous C. Serous D. HemorrhagicE. DiphteriticANSWER: E

Autopsy of a man who died from ethylene glycol poisoning revealed that his kidneys are a little bit enlarged, edematic; their capsule can be easily remove. Cortical substance is broad and light-grey. Medullary substance is dark-re. What pathology had this man? A. Necrotic nephrosisB. Acute tubular-interstitial nephritis

C. Lipoid nephrosisD. Acute glomerulonephritisE. Acute pyelonephritisANSWER: A

Histologic examination revealed in all layers of appendix a big number of polymorthonuclear leukocytes; hyperemia, stases. What disease are these symptoms typical for? A. Superficial appendicitis B. Phlegmonous appendicitisC. Gangrenous appendicitisD. Chronic appendicitis E. Simple appendicitisANSWER: B

Dystrophic changes of heart are accompanied by dilatation of cardiac cavities, decrease of heart beat force, increased volume of blood that remains in cardiac cavity after systole; veins are overfille. What state is this presentation typical for? A. Cardiac tamponadeB. Tonogenic dilatation C. Cardiosclerosis stageD. Emergency phase of myocardial hypertrophy E. Myogenic dilatationANSWER: E

A 46 year old patient was admitted to the hematological department. It was found that he had disorder of granulocytopoesis and thrombocytogenesis processes. In what organ does this pathological process take pace? A. Lymphatic ganglion B. Thymus C. Spleen D. Palatine tonsil E. Red bone marrowANSWER: E

A 22 year woman has enlarged lymphatic ganglions. Histological analysis of a ganglion revealed lymphocytes, histiocytes, reticular cells, small and great Hodgkin's cells, multinuclear Reed-Sternberg cells, solitary foci of caseous necrosis. What disease are these changes typical for? A. Cancer metastasis B. Lymphosarcoma C. LymphogranulematosisD. Acute leukemia E. Chronic leukemiaANSWER: C

A 65 year old patient suddenly die. She suffered from thrombophlebitis of deep veins of shin. Autopsy revealed: trunk and bifurcation of pulmonary artery contain red loose masses with dull corrugated surfac. What pathological process did the morbid anatomist reveal in pulmonary artery?A. Foreign body embolism B. ThrombosisC. Fat embolismD. Thromboembolism

E. Tissue embolismANSWER: D

During the electronical microscopic analysis of salivary gland the cell fragmets were revealed which are surrounded by a membrane and contain condensed particles of nuclear substance and solitary organelles; the inflammatory reaction around these cells is absent. What process is meant? A. KaryopicnosisB. Apoptosis C. KaryorhexisD. Coagulation necrosis E. Karyolysis ANSWER: B

Autopsy of a woman who died of tumorous dissemination of mucinosous cystadenocarcinoma and before that had to stay in bed for a long time revealed big necrotic areas of skin and soft subjacent tissues in sacral region. What form of necrosis is the case? A. Sequester B. Infarction C. Pressure soreD. Zenker's necrosis E. Caseous necrosis ANSWER: C

During the histologic lung analysis of a man who died from cardiac insufficiency the inflammation focuses were reveale. Alveoles were full of light-pink fluid, here and there with pinkish fibers that formed a close-meshed reticulum with a small number of lymphocytes. What type of exudate is present in lungs? A. Fibrinous B. Hemorrhagic C. Serous D. SerofibrinousE. Purulent ANSWER: D

A 53 year old patient consulted a doctor about white patch on the mucous membrane of tongu. This patch sticks out from the mucous membrane, its surface is cracke. Microscopic analysis reveals thickening of multilayer epithelium, parakeratosis and acanthosis. What is the most probable diagnosis? A. Geographic tongue B. Papilloma C. Median rhomboid glossitis D. Epidermoid cancer E. LeukoplakiaANSWER: E

During morphologic analysis of pulp floor three zones can be distinctly differentiated: the one of softened dentin, transparent dentin and replacing dentin. What stage of caries are these changes typical for? A. Chronic cariesB. Deep caries C. Stain stage

D. Superficial caries E. Median caries ANSWER: E

During the histologic examination of thyroid gland of a man who died of cardiac insufficiency together with hypothyroidism there was found the diffusive infiltration of gland by lymphocytes and plasmocytes, parenchyma atrophy and growth of connective tissu. Formulate a diagnosis: A. Thyroid gland adenoma B. Thyrotoxic goiter C. Hashimoto's thyroiditisD. - E. Purulent thyroiditisANSWER: C

Microscopic analysis of brain base vessels of a patient who died of ischemic stroke revealed that intima of cerebral vessels is irregular, with moderate quantity of yellow stains and yellowish-whitish patches that narrow lumen. What is the most probable diagnosis? A. Nodular periarteritisB. Diabetes mellitus C. AtherosclerosisD. Rheumatism E. Primary hypertension ANSWER: C

A 10 year old child lives in the region where fluorine content in water is above the mark. A dentist examined the child and found teeth damage in form of chalky and also pigmentary stains and stripes. What is the most probable diagnosis? A. Wedge defects B. FluorosisC. Median cariesD. Tooth erosion E. Acidic necrosis of hard tooth tissues ANSWER: B

Histologic analysis of uterus mucous membrane revealed twisting glands, serrated and spinned, they were extended by stroma growth with proliferation of its cells. Formulate a diagnosis:A. Cystic moleB. Placental polypC. Leiomyoma D. Glandular hyperplasia of endometriumE. Acute endometritis ANSWER: D

The microscopic analysis of bronch biopsy revealed a tumor that consisted of circumscribed accumulations of atypical cells of multylayer plane epithelium, here and there with typical "pearls". What is the most likely diagnosis? A. Solid carcinoma B. Epidermoid cancer with keratinization C. Epidermoid cancer without keratinizationD. ScirrhusE. Mucous carcinomaANSWER: B

Autopsy of a man who died of typhoid fever revealed ulcers along the ileum. These ulcers have even sides, clean fundus formed by muscle layer or even by serous tunic of an intestin. What stage of disease does the described presentation correspond with?

A. Stage of medullary swellingB. Stage of "clean" ulcersC. Stage of necrosis D. Stage of "dirty" ulcersE. Stage of ulcer healingANSWER: B

Autopsy of a 5 year old child revealed that pia maters of brain are extremely plethoric, nebulous, have a look of yellowish-green "bonnet". Microscopic analysis: pia mater of brain is very thickened, plethoric, impregnated with purulent exudate containing fibrin. What disease is meant? A. Anthrax B. Measles C. Meningococcosis D. Influenza E. Tuberculosis ANSWER: C

The symptoms of regeneration process (callus) on the place of fracture were revealed in the histologic specimen of tubular bone. What tissue forms this structure? A. Loose connective tissue B. Reticular tissue C. Epithelial tissue D. Lamellar bone tissue E. Fibrous bone tissueANSWER: E

A 57 year old patient has periodic uterine bleedings. Diagnostic endometrectomy was performed. Biopsy material contains among the blood elements some glandular complexes of different sizes and forms that consist of atypic cells with hyperchromic nuclei and multiple mitoses (including pathological ones). What is the most probable diagnosis? A. Fibromyoma of uterus B. AdenocarcinomaC. Chorioepithelioma D. Glandular hyperplasia of endometrium E. Endometritis ANSWER: B

Histologic examination revealed a big number of polymorphonuclear leukocytes in all layers of appendix; hyperemia, stases. What disease are these symptoms typical for? A. Chronic appendicitis B. Simple appendicitis C. Superficial appendicitis D. Gangrenous appendicitis E. Phlegmonous appendicitisANSWER: E

In course of gastric endoscopy the biopsy material of mucous membrane was taken. Its histological examination revealed the following: mucous membrane is intact, thickened, edematic, hyperemic, with small droplike hemorrhages, coated with thick mucus. Name the form of acute gastritis: A. Erosive B. CatarrhalC. Fibrinous D. Necrotic E. Purulent ANSWER: B

During the histologic lung analysis of a man who died from cardiac insufficiency the inflammation focuses were revealed. Alveoles were full of light-pink fluid, here and there with pinkish fibers that formed a close-meshed reticulum with a small number of lymphocytes. What type of exudate is present in lungs? A. Purulent B. Serous C. SerofibrinousD. Hemorrhagic E. Fibrinous ANSWER: C

Autopsy of a man who suffered from essential hypertension revealed a cavity with rust-coloured walls in the cerebral substunce. What preceded the appearance of these changes? A. HematomaB. Plasmorrhagias C. Ischemic infarction D. Abscess E. Diapedetic hemorrhages ANSWER: A

Tissue sample of soft palate arches that was taken because a tumor was suspected (microscopic analysis revealed an ulcer with dense fundus) revealed mucous membrane necrosis, submucous layer was infiltrated by lymphocytes, epithelioid cells, plasmocytes, solitary neutrophils. There was also evident endovasculitis and perivasculitis. What desease are these changes typical for? A. Ulcerative stomatitis B. Primary syphilisC. Aphthous stomatitis D. Faucial diphteria E. Vensan's ulcerative-necrotic stomatitis ANSWER: B

Examination of a 16 year old boy revealed enlarged submandibular and cervical lymph nodes. The boy was subjected to biopsy. Microscopic examination of lymph nodes revealed: typical structure is obliterated, cell population is heterogenous, there are big cells with multilobe nuclei, multiple big mononuclear cells, eosinophilic and neutrophilic leukocytes, lymphocytes, besides that, there are necrotic areas and foci of sclerosis. What is the most probable diagnosis? A. lymphogranulomatosisB. Granulomatous lymphadenitis C. Suppurative lymphadenitis D. Lymph node hyperplasia E. Non-Hodgkin's lymphoma

ANSWER: A

Poisoning with mercuric dichloride caused acute renal insufficiency that included 4 stages: 1) the initial one, 2) the stage of oligoanuria, 4) the stage of recovery. What is the third stage of acute renal insufficiency? A. Pathochemic B. PolyureticC. Metabolic D. Ischemic E. Hemodynamic ANSWER: B

A 22 year woman has enlarged lymphatic ganglions. Histological analysis of a ganglion revealed lymphocytes, histiocytes, reticular cells, small and great Hodgkin's cells, multinuclear Reed-Sternberg cells, solitary foci of caseous necrosis. What disease are these changes typical for? A. Lymphosarcoma B. Acute leukemia C. LymphogranulematosisD. Chronic leukemia E. Cancer metastasis ANSWER: C

On the 5th day of illness a 12 year old child who was treated in the infectious department on account of influenza felt severe headache, sickness, dizziness, got meningeal signs. The child died 24 hours later from increasing brain edema. Dissection of cranial cavity revealed that pia maters of brain are edematic, plethoric, saturated diffusively with bright red liquid. Convolutions and sulci of brain are flattened. What influenza complication is in question? A. Hemorrhagic meningitisB. Venous hyperemia of brain membranes C. Cerebral hemorrhage D. Suppurative leptomeningitis E. Serous meningitis ANSWER: A

Histological examination of thyroid gland of a man who died from cardiac insufficiency accompanied by hypothyroidism revealed diffuse infiltration of the gland by lymphocytes and plasmocytes with formation of lymphoid follicles, as well as atrophy of parenchyma and growth of connective tissue. What is the most probable diagnosis? A. Adenoma of thyroid gland B. - C. Purulent thyroiditis D. Thyrotoxic goiter E. Autoimmune Hashimoto's thyroiditisANSWER: E

Examination of a child who has recently recovered from measles revealed in the soft tissues of cheeks and perineum some inaccurate, edematic, red-and-black, slightly fluctuating areas. What complication is it? A. Trophic ulcer B. Gas gangrene C. Pressure sore D. Humid gangrene

E. Dry gangrene ANSWER: D

Microscopic analysis of tissue sampling from patient's skin reveals granulomas that consist of epithelioid cells surrounded mostly by T-lymphocytes. Among epithelioid cells there are solitary giant multinuclear cells of Pirogov-Langhans type. In the centre of some granulomas there are areas of caseous necrosis. Blood vessels are absent. What disease are the described granulomas typical for? A. TuberculosisB. Syphilis C. Leprosy D. Rhinoscleroma E. Glanders ANSWER: A

A patient with suspected liver abscess was admitted to the surgical department. The patient had been staying for a long time on business in one of african countries and fell repeatedly ill with acute gastrointestinal disorders. What protozoal disease may the patient be now ill with? A. Malaria B. Toxoplasmosis C. Trypanosomosis D. Leishmaniasis E. AmebiasisANSWER: E

Examination of a patient who had been suffering from rheumatism for a long time revealed stenosis of mitral orifice, death was caused by cardiac and pulmonary insufficiency. Autopsy has shown brown induration of lungs. What type of circulation disturbance provokes such changes in lungs? A. Chronic left ventricular insufficiencyB. Chronic right ventricular insufficiency C. Acute right ventricular insufficiency D. Portal hypertension E. Acute left ventricular insufficiency ANSWER: A

Autopsy of a woman with cerebral atherosclerosis revealed in the left cerebral hemisphere a certain focus that is presented by flabby, anhistic, greyish and yellowish tissue with indistinct edges. What pathological process is the case? A. Ischemic strokeB. Focal encephalitis C. Multiple foci of fresh and old cerebral hemorrhage D. Senile encephalopathy E. Multifocal tumor growth with cystic degeneration ANSWER: A

Examination of coronary arteries revealed atherosclerotic plaques with calcification that close tle lumen by 1/3. The muscle contains multiple small whitish layers of connective tissue. What process was revealed in myocardium? A. Tiger heart B. Postinfarction cardiosclerosis C. Myocarditis

D. Myocardium infarction E. Diffuse cardiosclerosisANSWER: E

Post-mortem examination of a 5 year old boy who died from acute pulmonary and cardiac insufficiency revealed the following: serohemorrhagic tracheobronchitis with some necrotic areas of mucous membrane, multiple foci of hemorrhagic pneumonia in lungs. What disease is in question? A. Diphtheria B. Croupous pneumonia C. Scarlet fever D. InfluenzaE. Measles ANSWER: D

Opening of a patient's abdominal cavity revealed for about 2,0 L of purulent fluid. Peritoneum is dull, greyish, serous tunic of intestines has grayish layers that can be easily removed. It is most likely to be: A. Serous peritonitis B. Hemorrhagic peritonitis C. Tuberculous peritonitis D. - E. Fibrinopurulent peritonitisANSWER: E

A 42 y.o. man who had been suffering from chronic granulomatous periodontitis and chronic purulent osteomyelitis of his lower jaw for 8 years died under conditions of acute renal insufficiency. What complication of purulent osteomyelitis was developed in kidneys? A. Adipose degeneration B. Atrophy C. Necrosis of epithelium of convoluted tubules D. AmyloidosisE. Hyalinosis ANSWER: D

A patient with closed fracture of humeral bone was bandaged with plaster. The next day the injured hand became swollen, cyanotic and col D. What disorder of peripheral blood circulation are these symptoms typical for? A. Thrombosis B. Ischemia C. Arterial hyperemia D. Embolism E. Venous hyperemiaANSWER: E

Roentgenological examination of a patient revealed a cyst in the area of a premolar that contained a tooth in its cavity. Microscopical examination reveals that the cyst wall is represented by connective tissue and lined with multistratal squamous cell epithelium. Specify the diagnosis: A. Primordial cyst B. Eosinophilic granuloma C. Follicular cyst

D. Epulis E. Radicular cyst ANSWER: C

A 5 y.o. child had a temperature rise up to 400C, acute headache, vomiting, anxiety, chill. 4 days later there appeared hemorrhagic skin eruption, oliguria and adrenal insufficiency that caused death. Bacteriological examination of smears from the child's pharynx revealed meningococcus. What disease form was revealed? A. Meningococcal meningitis B. Meningicoccal nasopharyngitis C. MeningococcemiaD. Meningoencephalitis E. - ANSWER: C

Microscopic examination during autopsy of a 70 y.o. man who had been ill with atherosclerosis for a long time and died from cardiovascular insufficiency revealed in the abdominal area of aorta some dense oval fibrous plaques with lime deposition in form of dense brittle plates. What stage of atherosclerosis morphogenesis is it? A. Atheromatosis B. Liposclerosis C. Ulceration D. AtherocalcinosisE. Lipoidosis ANSWER: D

Examination of a 10 y.o. child revealed on the alveolar submandibular process a fixed tumourous mass 1,5 cm in diameter closing premolar crown on the vestibular sid E. Mucous membrane of its surface is reddish-brown, it bleeds as a reaction to a slight mechanical intervention. Biopsy results: the mass consists of small size vessels separated by thin layers of connective tissue and infiltrated by plasmocytes, mucous membrane is here and there ulcere D. What is the most probable diagnosis? A. Hypertrophic gingivitis B. Giant cell form of epulis C. Gingival fibromatosis D. Fibrous form of epulis E. Angiomatous form of epulisANSWER: E

A 5 y.o. girl has high temperature and sore throat. Objectively: soft palate edema, tonsills are covered with grey films that can be hardly removed and leave deep bleeding tissue injuries. What disease is the most probable? A. Lacunar angina B. Infectious mononucleosis C. Vincent's angina D. Necrotic angina E. Pharyngeal diphtheriaANSWER: E

There is a specimen of soft palate where both oral and nasal surfaces can be seen. It was revealed that oral cavity had damaged epithelium. What epithelium is damaged? A. Multirowed ciliated epithelium

B. Multistratal prismatic nonkeratinizing C. Multistratal squamous keratinizing D. Multistratal cubical nonkeratinizing E. Multistratal squamous nonkeratinizingANSWER: E

Examination of a 60 y.o. man's oral cavity revealed the following changes: the 26th and 27th tooth are covered with metallic crowns that plunge deep into the gums. There is a parodontal pouch 0,7 cm deep between them containing some pus. Gingival papillae of these teeth are hyperemic, edematic, cyanotic, bleed as a reaction to touching by a dental explorer. X-ray picture shows resorption of interdental septa of 1/2 of tooth root. What is the most probable diagnosis? A. Hypertrophic gingivitis B. Local parodontitisC. - D. Generalized parodontitis E. Chronic catarrhal gingivitis ANSWER: B

A patient who had been suffering from a renal disease for many years died from uremi A. Autopsy revealed that the kidneys were abnormally small, dense, fine-grained, light grey. What are the kidneys with such changes called? A. Sebaceous B. ArterioloscleroticC. Contracted D. - E. Mottled ANSWER: B

Macroscopic examination of lung tissue revealed areas of high airiness with small bubbles. histological examination revealed thinning and rupture of alveolar septa accompanied by formation of large diversiform cavities. What disease was revealed in a lung? A. Multiple bronchiectasis B. Cavernous tuberculosis C. Pulmonary emphysemaD. Chronic bronchitis E. Fibrosing alveolitis ANSWER: C

A patient has recently had staphylococcal infection that led to anasarca; laboratory urine analysis revealed massive proteinuri A. Results of blood analysis: hypoproteinemia, hyperlipemi A. What pathology can be suspected? A. Glomerulonephritis B. Chronic renal insufficiency C. Urolithiasis D. Nephrotic syndromeE. Pyelonephritis ANSWER: D

Soft palate arches were taken for bioptic examination because of suspected tumour (macroscopical examination revealed an ulcer with dense floor). Biopsy revealed necrosis of mucous membrane along with infiltration of submucous layer by lymphocytes, epithelioid cells,

plasmatic cells, single neutrophils. There is also evident endo- and perivasculitis. What disease are the described changes typical for? A. Ulcerative necrotic stomatitis (Vincent's stomatitis) B. Aphthous stomatitis C. Pharyngeal diphtheria D. Primary syphilisE. Ulcerative stomatitis ANSWER: D

Examination of a tooth revealed a large cavity in its crown. The floor of this cavity is formed by a thin layer of softened dentin that separates this cavity from the pulp. What is the most probable diagnosis? A. Deep cariesB. Pulpitis C. Periodontitis D. Median caries E. Superficial caries ANSWER: A

Microscopical examination of a surgical specimen (an ulcered part of a lip) revealed in the connective tissue of mucous membrane near the borders and under the floor of the ulcer some epithelial complexes consisting of atypic multistratal epithelium with accumulations of bright pink concentric formations. What pathology is it? A. Transitional cell carcinoma B. Papilloma C. Squamous cell keratinous carcinomaD. Squamous cell nonkeratinous carcinoma E. Basal cell carcinoma ANSWER: C

Mucous membrane of a patient's oral cavity has a greyish-white focus, the mass is dense and protrudes above the mucous membran E. Histological examination revealed hyperkeratosis, parakeratosis and acanthosis of epithelium in this are A. What pathological process was revealed in the mucous membrane? A. LeukoplakiaB. Hyalinosis C. Local tumourous amyloidosis D. Focal ichthyosis E. Leukoderm ANSWER: A

A 9 y.o. child has been taking antibiotics on account of bronchopneumonia for a long tim E. There appeared pain and burning in the area of mucous mebrane of his lips and tongu E. Objectively: mucous membrane of lips and tongue has caseous and grey plaques that can be easily removed by a spatula leaving hyperemia foci on their spot. Microscopical examination of the plaques revealed mycelium. What is the most probable diagnosis? A. Leukoplakia B. Manganotti's cheilitis C. Exfoliative cheilitis D. Candidous cheilitisE. Contactant allergic cheilitis ANSWER: D

Autopsy of a 35 y.o. woman revealed not only enlargement of many lymph nodes but also enlarged spleen weighting 600,0. Its incision showed that it was heterogeneous, dark red, dense with greyish-yellow necrotic areas up to 1 cm in diameter (porphyritic spleen). What disease can be assumed? A. Chronic lymphoid leukosis B. Chronic myeloid leukosis C. Cancer metastases D. LymphogranulomatosisE. Lymphosarcoma ANSWER: D

Histological examination of periapical tissue taken from a patient who has been suffering from chronic periodontitis for a long time revealed a granulation tissue interlaced by taeniae of squamous cell epithelium and encircled within a fibrous capsul E. What is the most probable diagnosis? A. Abscessing periodontitis B. Simple granuloma C. Granulating periodontitis D. Composite granulomaE. Cystic granuloma ANSWER: D

A 13-year-old patient complains of general weakness, dizziness, fatiguability. Mental retardation is also observed. Examination revealed high concentration of valine, isoleucine and

leucine in blood and urine. The patient's urine has a specific smell. What is the likely cause of such condition? A. Maple syrup urine diseaseB. Addison's disease C. Tyrosinosi D. Histidinemia E. Basedow's disease ANSWER: A

A patient has increased pyruvate concentration in blood. Large amount of it is excreted with urine. What vitamin deficiency is observed? A. B1B. EC. B3D. B6E. B2ANSWER: A

Pellagra may be caused by maize domination and low quantity of animal foodstuffs in the dietary intake. This pathology results from lack of the following amino acid:

A. TryptophaneB. Isoleucine C. Phenylalanine D. Methionine E. Histidine ANSWER: A

During the histologic examination of thyroid gland of a man who died of cardiac insufficiency together with hypothyroidism there was found the diffusive infiltration of gland by lymphocytes and plasmocytes, parenchyma atrophy and growth of connective tissue. Formulate a diagnosis: A. Hashimoto's thyroiditisB. Thyroid gland adenoma C. Purulent thyroiditis D. Thyrotoxic goiter E. - ANSWER: A

The lung hypertension and cardiac insufficiency of right ventricle with ascites and edemata developed at patient with pneumosclerosis. What is the principal pathogenetic mechanism of edemata development? A. Increase of hydrostatic blood pressure in veinsB. Increase of oncotic pressure of intercellular fluid C. Decrease of oncotic blood pressure D. Decrease of osmotic blood pressure E. Increase of vascular permeability ANSWER: A

A patient with pneumosclerosis has pulmonary hypertension and cardiac insuffisiency of right ventricle with ascites and edemata. What is the main pathogenetic mechanism of edemata development? A. Rise of hydrostatic blood pressure in veinsB. Rise of oncotic pressure of intracellular fluid C. Reduction of oncotic blood pressure D. Increased permeability of vessel walls E. - ANSWER: A

Dystrophic changes of heart are accompanied by dilatation of cardiac cavities, decrease of heart beat force, increased volume of blood that remains in cardiac cavity after systole; veins are overfilled. What state is this presentation typical for? A. Myogenic dilatationB. Tonogenic dilatation C. Emergency phase of myocardial hypertrophy D. Cardiosclerosis stage E. Cardiac tamponade ANSWER: A

A 7 year old child had an acute onset of disease. Pediatrician stated that mucous membrane of fauces is hyperemic and covered with a lot of mucus. Mucous membrane of cheeks has whitish stains. Next day the child's skin of face, neck, body was covered with coarsely-papular rash. What disease may be presumed? A. MeaslesB. Scarlet fever C. Diphteria D. Meningococcemia E. Allergic dermatitis ANSWER: A

Autopsy of a woman with cerebral atherosclerosis revealed in the left cerebral hemisphere a certain focus that is presented by flabby, anhistic, greyish and yellowish tissue with indistinct edges. What pathological process is the case? A. Ischemic strokeB. Multifocal tumor growth with cystic degeneration C. Multiple foci of fresh and old cerebral hemorrhage D. Focal encephalitis E. Senile encephalopathy ANSWER: A

Autopsy of a 52-year-old woman with a long history of chronic glomerulonephritis revealed significantly reduced in size, dense kidneys with a surface of fine granularity; fibrinous inflammation of serous and mucous membranes; dystrophic changes in parenchymatous organs; cerebral edema. The described changes of serous membranes and internal organs are caused by the following complication: A. UraemiaB. Anaemia C. Sepsis D. DIC syndrome E. Thrombocytopenia ANSWER: A

A 10 year old child lives in the region where fluorine content in water is above the mark. A dentist examined the child and found teeth damage in form of chalky and also pigmentary stains and stripes. What is the most probable diagnosis? A. FluorosisB. Median caries C. Wedge defects D. Tooth erosion E. Acidic necrosis of hard tooth tissues ANSWER: A

Histological study of the bronchial wall and adjacent lung segments revealed sheets and strands of squamous epithelium. The cells have moderately expressed symptoms of atypia: polymorphism, nuclear hyperchromatism, mitoses. In the center of the complex there are concentric pink formations. What is the most likely diagnosis? A. Keratinizing squamous cell carcinomaB. Non-keratinizing squamous cell carcinoma C. Adenocarcinoma D. Scirrhus E. Undifferentiated carcinoma ANSWER: A

Histologically, the internal wall of a cyst localized on the upper jaw is lined with stratified squamous epithelium with underlying granulation tissue infiltrated by lymphocytes. The external layer is represented by loose fibrous connective tissue surrounded by cicatrical fibrous tissue. What diagnosis can be made? A. Cystic granulomaB. Simple granuloma C. Epithelial granuloma D. Keratocyst E. Ameloblastoma

ANSWER: A

A 53 year old patient consulted a doctor about white patch on the mucous membrane of tongue. This patch sticks out from the mucous membrane, its surface is cracked. Microscopic analysis reveals thickening of multilayer epithelium, parakeratosis and acanthosis. What is the most probable diagnosis? A. LeukoplakiaB. Geographic tongue C. Epidermoid cancer D. Papilloma E. Median rhomboid glossitis ANSWER: A

The microscopic analysis of bronch biopsy revealed a tumor that consisted of circumscribed accumulations of atypical cells of multylayer plane epithelium, here and there with typical"pearls". What is the most likely diagnosis? A. Epidermoid cancer with keratinizationB. Epidermoid cancer without keratinization C. Solid carcinoma D. Mucous carcinoma E. Scirrhus ANSWER: A

Histologic examination revealed a big number of polymorphonuclear leukocytes in all layers of appendix; hyperemia, stases. What disease are these symptoms typical for? A. Phlegmonous appendicitisB. Gangrenous appendicitis C. Superficial appendicitis D. Simple appendicitis E. Chronic appendicitis ANSWER: A

A female patient suffering from secondary syphilis got foci of skin depigmentation in the upper parts of her back. What pathological process is it?

A. LeukodermaB. Metaplasia C. Leukoplasia D. Dysplasia E. Parakeratosis ANSWER: A

Opening of a patient's abdominal cavity revealed for about 2,0 L of purulent fluid. Peritoneum is dull, greyish, serous tunic of intestines has grayish layers that can be easily removed. It is most likely to be: A. Fibrinopurulent peritonitisB. Hemorrhagic peritonitis C. Serous peritonitis D. Tuberculous peritonitis E. - ANSWER: A

A 77-year-old patient with atherosclerosis got pain in his right foot. The foot is enlarged, the skin is black and macerated, the demarcation zone is not defined clearly. What pathological process arose in the foot? A. Wet gangreneB. Dry gangrene C. Noma D. Sequestrum E. Coagulation necrosis ANSWER: A

Microscopic analysis of brain base vessels of a patient who died of ischemic stroke revealed that intima of cerebral vessels is irregular, with moderate quantity of yellow stains and yellowish-whitish patches that narrow lumen. What is the most probable diagnosis?A. AtherosclerosisB. Primary hypertension C. Diabetes mellitus D. Rheumatism E. Nodular periarteritis ANSWER: A

A 42-year-old man died with symptoms of severe intoxication and respiratory failure. A slide of lung tissue was heterogenous, with multiple microfocal hemorrhages and foci of emphysema. Histological examination of lungs revealed hemorrhagic abscessing bronchopneumonia; eosinophilic and basophilic granules in the cytoplasm of epithelial cells of bronchi. What is the most likely diagnosis? A. InfluenzaB. Parainfluenza C. Adenovirus infection D. Respiratory syncytial virus infection E. Staphylococcal bronchopneumonia ANSWER: A

Autopsy of a 5 year old child revealed that pia maters of brain are extremely plethoric, nebulous, have a look of yellowish-green "bonnet". Microscopic analysis: pia mater of brain is very thickened, plethoric, impregnated with purulent exudate containing fibrin. What disease is meant? A. MeningococcosisB. Tuberculosis C. Anthrax D. Influenza E. Measles ANSWER: A

Histological examination of thyroid gland of a man who died from cardiac insufficiency accompanied by hypothyroidism revealed diffuse infiltration of the gland by lymphocytes and plasmocytes with formation of lymphoid follicles, as well as atrophy of parenchyma and growth of connective tissue. What is the most probable diagnosis? A. Autoimmune Hashimoto's thyroiditisB. Adenoma of thyroid gland C. Purulent thyroiditis D. Thyrotoxic goiter E. -

ANSWER: A

Post-mortem examination of a 5 year old boy who died from acute pulmonary and cardiac insufficiency revealed the following: serohemorrhagic tracheobronchitis with some necrotic areas of mucous membrane, multiple foci of hemorrhagic pneumonia in lungs. What disease is in question? A. InfluenzaB. Measles C. Scarlet fever D. Diphtheria E. Croupous pneumonia ANSWER: A

Autopsy of a 70-year-old man who died from cardiac insufficiency revealed deformed and constricted coronary arteries. The artery section shows that the intimal surface is stony hard and fragile. It is also whitish, with nodular appearance. What stage of atherosclerosis is it? A. AtherocalcinosisB. Liposclerosis C. Atheromatosis D. Lipoidosis E. Ulceration ANSWER: A

Autopsy of a man who died from intraintestinal hemorrhage revealed necrosis of grouped and solitary follicles, dead tissues imbibed with bile and blood in the ileum; sequestration and

rejection of necrotic masses with defect formation in the lower segment of the intestine. Which of the following diagnoses is most likely? A. Typhoid fever, ulcerative stageB. Typhoid fever, "clean ulcer" stage C. Typhoid fever, necrosis stage D. Abdominal typhoid salmonellosis E. Crohn's disease ANSWER: A

Autopsy of an aged man who had been suffering from acute intestinal upset for the last 2 weeks revealed the following changes in the rectum and sigmoid colon: mucous membrane surface was coated with brown-green film. The intestine wall was thickened, and its cavity was extremely constricted. Microscopical examination revealed variously deep penetrating necrosis of mucous membrane; necrotic masses contained fibrin fibers and bore signs of leukocytic infiltration. What is the most likely diagnosis? A. Fibrinous colitisB. Catarrhal colitis C. Ulcerative colitis D. Follicular collitis E. -ANSWER: A

Examination of coronary arteries revealed atherosclerotic plaques with calcification that close tle lumen by 1/3. The muscle contains multiple small whitish layers of connective tissue. What process was revealed in myocardium? A. Diffuse cardiosclerosisB. Tiger heart

C. Postinfarction cardiosclerosis D. Myocarditis E. Myocardium infarction ANSWER: A

Microscopic analysis of tissue sampling from patient's skin reveals granulomas that consist of epithelioid cells surrounded mostly by T-lymphocytes. Among epithelioid cells there are solitary giant multinuclear cells of Pirogov-Langhans type. In the centre of some granulomas there are areas of caseous necrosis. Blood vessels are absent. What disease are the described

granulomas typical for? A. TuberculosisB. Syphilis C. Leprosy D. Rhinoscleroma E. Glanders ANSWER: A

Examination of a child who has recently recovered from measles revealed in the soft tissues of cheeks and perineum some inaccurate, edematic, red-and-black, slightly fluctuating areas. What complication is it? A. Humid gangreneB. Dry gangrene C. Gas gangrene D. Pressure sore E. Trophic ulcer ANSWER: A

Autopsy of a woman who died of tumorous dissemination of mucinosous cystadenocarcinoma and before that had to stay in bed for a long time revealed big necrotic areas of skin and soft subjacent tissues in sacral region. What form of necrosis is the case? A. Pressure soreB. Infarction C. Sequester D. Caseous necrosis E. Zenker's necrosis ANSWER: A

Examination of a patient who had been suffering from rheumatism for a long time revealed stenosis of mitral orifice, death was caused by cardiac and pulmonary insufficiency. Autopsy has shown brown induration of lungs. What type of circulation disturbance provokes such changes in lungs? A. Chronic left ventricular insufficiencyB. Chronic right ventricular insufficiency C. Acute left ventricular insufficiency D. Acute right ventricular insufficiency E. Portal hypertension ANSWER: A

During the electronical microscopic analysis of salivary gland the cell fragmets were revealed which are surrounded by a membrane and contain condensed particles of nuclear substance and solitary organelles; the inflammatory reaction around these cells is absent. What process is meant?

A. ApoptosisB. Karyorhexis C. Coagulation necrosis D. Karyopicnosis E. Karyolysis ANSWER: A

On the 5th day of illness a 12 year old child who was treated in the infectious department on account of influenza felt severe headache, sickness, dizziness, got meningeal signs. The child died 24 hours later from increasing brain edema. Dissection of cranial cavity revealed that pia maters of brain are edematic, plethoric, saturated diffusively with bright red liquid. Convolutions and sulci of brain are flattened. What influenza complication is in question? A. Hemorrhagic meningitisB. Cerebral hemorrhage C. Venous hyperemia of brain membranes D. Suppurative leptomeningitis E. Serous meningitis ANSWER: A

Autopsy of a man who died of typhoid fever revealed ulcers along the ileum. These ulcers have even sides, clean fundus formed by muscle layer or even by serous tunic of an intestine. What stage of disease does the described presentation correspond with? A. Stage of "clean" ulcersB. Stage of medullary swelling C. Stage of necrosis D. Stage of "dirty" ulcers E. Stage of ulcer healing ANSWER: A

Morphological examination of carious cavity floor differenciated distinctly three zones: the one of softened dentin, transparent dentin and replacing dentin. What stage of caries are these changes typical for? A. Median cariesB. Spot stage C. Superficial caries D. Deep caries E. Chronic caries ANSWER: A

A man had an acute onset of disease, he complained of chill, temperature rise up to 40oС, headache, cough, dyspnea. On the fifth day of illness he died. Autopsy revealed:his lungs were enlarged, they had a look of "coal-miner's lungs". What illness is such postmortem diagnosis typical for?A. InfluenzaB. Adenovirus infection C. Croupous pneumonia D. Respiratory syncytial infection E. Multiple bronchiectasis ANSWER: A

Mucous membrane of the right palatine tonsil has a painless ulcer with smooth lacquer fundus and accurate edges of cartlaginous consistency. Microscopically: inflammatory infiltrate that

consists of lymphocytes, plasmocytes, a small number of neutrophils and epithelioid cells; endovasculistis and perivasculitis. What disease is in question? A. SyphilisB. Actinomycosis C. Tuberculosis D. Pharyngeal diphtheria E. Necrotic (Vincent's) tonsillitis ANSWER: A

A 75 year old male patient consulted a surgeon about a brown nonhealing ulcer of shin. Examination of biopsy material revealed diffuse growth of polymorphic atypic cells with brown pigment in their cytoplasm. Pearls reaction was negative. There were also a lot of pathological mitoses and foci of tissue necrosis. What is the most probable diagnosis? A. MelanomaB. Local hemosiderosis C. Intradermal nevus D. Trophic ulcer E. Skin cancer ANSWER: A

A 22 year woman has enlarged lymphatic ganglions. Histological analysis of a ganglion revealed lymphocytes, histiocytes, reticular cells, small and great Hodgkin's cells, multinuclear Reed-Sternberg cells, solitary foci of caseous necrosis. What disease are these changes typical for? A. LymphogranulematosisB. Lymphosarcoma C. Chronic leukemia D. Acute leukemia E. Cancer metastasis ANSWER: A

A 57 year old patient has periodic uterine bleedings. Diagnostic endometrectomy was performed. Biopsy material contains among the blood elements some glandular complexes of different sizes and forms that consist of atypic cells with hyperchromic nuclei and multiple mitoses (including pathological ones). What is the most probable diagnosis? A. AdenocarcinomaB. Fibromyoma of uterus C. Chorioepithelioma D. Glandular hyperplasia of endometrium E. Endometritis ANSWER: A

Autopsy of a man who died from ethylene glycol poisoning revealed that his kidneys are a little bit enlarged, edematic; their capsule can be easily removed. Cortical substance is broad and light-grey. Medullary substance is dark-red. What pathology had this man? A. Necrotic nephrosisB. Acute pyelonephritis C. Acute glomerulonephritis D. Acute tubular-interstitial nephritis E. Lipoid nephrosis ANSWER: A

During the histologic lung analysis of a man who died from cardiac insufficiency the inflammation focuses were revealed. Alveoles were full of light-pink fluid, here and there with pinkish fibers that formed a close-meshed reticulum with a small number of lymphocytes. What type of exudate is present in lungs? A. SerofibrinousB. Hemorrhagic C. Serous D. Purulent E. Fibrinous ANSWER: A

A 38 year old patient died during intractable attack of bronchial asthma. Histological examination revealed mucus accumulations in bronchi's lumen, a lot of mast cells (labrocytes) in bronchi's wall, some of these cells are degranulated, there are also many eosinophils. Name pathogenesis of these changes in bronchi: A. Atopy, anaphylaxisB. Cytotoxic, cytolytic effect of antibodies C. Immune complex mechanism D. Cell-mediated cytolysis E. Granulematosis ANSWER: A

Having recovered from angina a 23-year-old patient developed urinary syndrome (hematuria, proteinuria, leukocyturia). Study of the puncture biopsy of a kidney revealed manifestations of intracapillary proliferative glomerulonephritis, and electron microscopy revealed large subepithelial deposits. What is the pathogenesis of this disease? A. Immunocomplex mechanismB. Atopy, anaphylaxis with production of IgE and their fixation to the mast cells C. Cytotoxic, cytolytic action of antibodies D. Cell-mediated cytolysis E. Granulomatosis ANSWER: A

48 hours after tuberculine test (Mantoux test) a child had a papule up to 10 mm in diameter on the spot of tuberculine introduction. What hypersensitivity mechanism underlies the mentioned changes? A. Cellular cytotoxicityB. Anaphylaxis C. Antibody-dependent cytotoxicity D. Immunocomplex cytotoxicity E. Granulomatosis ANSWER: A

A 7 year old child was taken to the infectious disease hospital with complaints of acute pain during swallowing, temperature rise up to 39oС, neck edema. Objective signs: tonsills are enlarged, their mucous membrane is plethoric and covered with a big number of whitish-yellowish films that are closely adjacent to the mucous membrane. After removal of these films the deep bleeding defect remains. What type of inflammation is it?A. DiphteriticB. Purulent C. Serous D. Crupous

E. Hemorrhagic ANSWER: A

In course of gastric endoscopy the biopsy material of mucous membrane was taken. Its histological examination revealed the following: mucous membrane is intact, thickened, edematic, hyperemic, with small droplike hemorrhages, coated with thick mucus. Name the form of acute gastritis: A. CatarrhalB. Erosive C. Fibrinous D. Purulent E. Necrotic ANSWER: A

A 4 year old child had Mantoux test. 60 hours after tuberculin introduction a focal skin hardening and redness 15 mm in diameter appeared. It was regarded as positive test. What type of hypersensitivity reaction is this test based upon? A. Delayed-type hypersensitivityB. Immune complex-mediated hypersensitivity C. Complement-mediated cytotoxic hypersensitivity D. Immediate hypersensitivity E. - ANSWER: A

Autopsy of an 8 year old boy who was ill with pharyngeal and tonsillar diphtheria and died one week after illness begin revealed myocardial changes in form of small-focal myocardiocyte necroses, stroma edema with slight lymphocytic infiltration. What type of myocarditis is it: A. AlternativeB. Septic C. Granulomatous D. Interstitional E. Focal-intermediate, exudative ANSWER: A

Histologic analysis of uterus mucous membrane revealed twisting glands, serrated and spinned, they were extended by stroma growth with proliferation of its cells. Formulate a diagnosis: A. Glandular hyperplasia of endometriumB. Acute endometritis C. Leiomyoma D. Cystic mole E. Placental polyp ANSWER: A

Medical examination of the first-year pupils included Mantoux test. 15 pupils out of 35 had negative reaction. What actions should be taken against children with negative reaction? A. BCG vaccinationB. Antitoxin vaccination C. Rabies vaccination D. Repeat Mantoux test E. Examination of blood serum ANSWER: A

Vaccination is done by means of a toxin that has been neutralized by a formaldehyde (0,4%) at a temperature 37-40oC for four weeks. Ramond was the first to apply this preparation for diphtheria prophylaxis. What preparation is it?A. AnatoxinB. Immunoglobulin C. Antitoxic serum D. Adjuvant E. Inactivated vaccine ANSWER: A

Examination of a 16 year old boy revealed enlarged submandibular and cervical lymph nodes. The boy was subjected to biopsy. Microscopic examination of lymph nodes revealed: typical structure is obliterated, cell population is heterogenous, there are big cells with multilobe nuclei, multiple big mononuclear cells, eosinophilic and neutrophilic leukocytes, lymphocytes, besides that, there are necrotic areas and foci of sclerosis. What is the most probable diagnosis? A. lymphogranulomatosisB. Lymph node hyperplasia C. Granulomatous lymphadenitis D. Suppurative lymphadenitis E. Non-Hodgkin's lymphoma ANSWER: A

Microscopical examination of an enlarged cervical lymph node revealed blurring of its structure, absence of lymphoid follicles; all the microscopic fields showed cells with roundish nuclei and thin limbus of basophil cytoplasm. It is known from the clinical data that other groups of lymph nodes are also enlarged as well as spleen and liver. What disease might be suspected? A. Lymphoid leukosisB. Lymphogranulomatosis C. Lymphosarcoma D. Myeloid leukosis E. Multiple myeloma ANSWER: A

A hospital admitted a 9 y.o. boy with mental and physical retardation. Biochemical blood analysis revealed high content of phenylalanine. Such condition may be caused by blocking of the following enzyme: A. Phenylalanine-4-monooxigenaseB. Oxidase of homogentisic acid C. Glutamine transaminase D. Aspartate aminotransferase E. Glutamate decarboxylase ANSWER: A

A worker of a cattle farm fell acutely ill and then died from the progressing intoxication. Autopsy revealed enlarged, hyposthenic spleen of dark-cherry colour when dissected; excessive pulp scraping. At the base and fornix of brain pia maters are edematous, soaked with blood, dark-red ("scarlet hat"). Microscopic examination revealed serous haemorrhagic inflammation of brain tissues and tunics along with destruction of small vessel walls. What is the most likely diagnosis? A. AnthraxB. Tularemia C. Brucellosis

D. Plaque E. Cholera ANSWER: A

An electronic microphotography represents a cell without nucleoli and nuclear membrane. Chromosomes are loosely scattered, centrioles migrate to the poles. What phase of cell cycle is it? A. ProphaseB. Anaphase C. Metaphase D. Telophase E. Interphase ANSWER: A

An 18 y.o. boy applied to a geneticist. The boy has asthenic constitution: narrow shoulders, broad pelvis, nearly hairless face. Evident mental deficiency. The provisional diagnosis was Klinefelter's syndrome. What method of clinical genetics will enable the doctor to confirm this diagnosis? A. CytogeneticB. Genealogical C. Twin study D. Dermatoglyphics E. Population-and-statistical ANSWER: A

Abnormal chromosome disjunction during meiosis resulted in formation of: an ovum with 22 autosomes and polar body with 24 chromosomes. If such an ovum would be fertilized with a normal spermatozoon (22+X) the child might have the following syndrome:A. Turner's syndromeB. Klinefelter's syndrome C. Trisomy X D. Down's syndrome E. Edwards' syndrome ANSWER: A

A patient with android-type obesity had been suffering from arterial hypertension, hyperglycemia, glycosuria for a long time and died from the cerebral haemorrhage. Pathologic examination revealed pituitary basophil adenoma, adrenal cortex hyperplasia. What is the most likely diagnosis? A. Itsenko-Cushing's syndromeB. Diabetes mellitus C. Acromegalia D. Pituitary nanism E. Adiposogenital dystrophy ANSWER: A

Autopsy of a man with a malignant stomach tumour who had died from cancer intoxication revealed in the posteroinferior lung fields some dense, grayish-red irregular foci protruding above the section surface. Microscopic examination revealed exudate containing a large amount of neutrophils in the lumen and walls of small bronchi and alveoles. Such pulmonary alterations indicate the following disease: A. Acute purulent bronchopneumonia

B. Acute bronchitis C. Croupous pneumonia D. Intermittent pneumonia E. Acute serous bronchopneumonia ANSWER: A

A patient is ill with dermatitis, diarrhea, dementia. During history taking it was revealed that the main foodstuff of the patient was maize. These disturbances are caused by deficiency of the following vitamin: A. PPB. B1C. B2D. B9E. B8ANSWER: A

Autopsy of a 1,5-year-old child revealed haemorrhagic skin rash, moderate hyperaemia and edema of nasopharyngeal mucous membrane, small haemorrhages in the mucous membranes and internal organs; dramatic dystrophic alterations in liver and myocardium; acute necrotic nephrosis; massive haemorrhages in the adrenal glands. What disease are these alterations the most typical for? A. Meningococcal infectionB. Scarlet fever C. Diphtheria D. Measles E. Epidemic typhus ANSWER: A

A 20 y.o. patient complains of general weakness, dizziness, rapid fatigability. Examination results: Hb- 80 g/l; microscopical analysis results: erythrocytes are deformed. These symptoms might be caused by: A. Sickle-cell anemiaB. Parenchymatous jaundice C. Acute intermittent porphyria D. Obturative jaundice E. Addison's disease ANSWER: A

Autopsy of a 50-year-old man revealed the following changes: his right lung was moderately compact in all parts, the dissected tissue was found to be airless, fine-grained, dryish. Visceral pleura had greyish-brown layers of fibrin. What is the most likely diagnosis? A. Croupous pneumoniaB. Tuberculosis C. Bronchopneumonia D. Interstitial pneumonia E. Pneumofibrosis ANSWER: A

A pregnant woman lost for about 800 ml of blood during labour. There is also tachycardia, arterial pressure is 100/70 mm Hg, tachypnea up to 28/min. What hypoxia type is primary in such clinical situation? A. Blood

B. Cardiovascular C. Mixed D. Tissue E. Respiratory ANSWER: A

Enzymatic jaundices are characterized by disbalanced activity of UDP-glucuronyl transferase. What compound is accumulated in the blood serum in case of these pathologies? A. Indirect bilirubinB. Direct bilirubin C. Biliverdin D. Mesobilirubin E. Verdoglobin ANSWER: A

Autopsy of a man, who had been suffering from the multiple bronchiectasis for 5 years and died from chronic renal insufficiency, revealed that kidneys were dense and enlarged, with thickened cortical layer of white colour with greasy lustre. What renal disease might be suspected? A. Secondary amyloidosisB. Glomerulonephritis C. Chronic pyelonephritis D. Necrotic nephrosis E. -ANSWER: A

A patient with apparent icteritiousness of skin, sclera and mucous membranes was admitted to the hospital. The patient's urine was of brown ale colour, analysis revealed presence of direct bilirubin. Feces had low concentration of bile pigments. What type of jaundice is it? A. ObturativeB. Parenchymatous C. Haemolytic D. Conjugated E. Absorbtion ANSWER: A

Microscopic examination during autopsy of a 70 y.o. man who had been ill with atherosclerosis for a long time and died from cardiovascular insufficiency revealed in the abdominal area of aorta some dense oval fibrous plaques with lime deposition in form of dense brittle plates. What stage of atherosclerosis morphogenesis is it? A. AtherocalcinosisB. Liposclerosis C. Atheromatosis D. Ulceration E. Lipoidosis ANSWER: A

A 71-year-old man had been presenting with diarrhea for 10 days. The feces had admixtures of blood and mucus. He was delivered to a hospital in grave condition and died 2 days later. Autopsy of the body revealed the following: diphtheritic colitis with multiple irregularly-shaped ulcers of different depth in both sigmoid colon and rectus. Bacteriological analysis revealed Shigella. What was the main disease? A. Dysentery

B. Typhoid fever C. Salmonellosis D. Nonspecific ulcerous colitis E. Yersiniosis ANSWER: A

Microscopy of colonic biopsy material revealed a tumour made up of prismatic epithelium and forming atypical glandular structures of various shapes and sizes. The basal membrane of glands was destroyed. Tumour cells were polymorphic, with hyperchromatic nuclei and a large number of pathological mitoses. What is the most likely diagnosis? A. AdenocarcinomaB. Basal cell carcinoma C. Solid carcinoma D. Mucosal carcinoma E. Undifferentiated carcinoma ANSWER: A

During examination a dentist revealed cervical caries of right inferior incisors as well as enlargement of a certain group of lymph nodes. What lymph nodes are enlarged? A. SubmentalB. Occipital C. Superficial cervical D. Deep cervical E. Facial ANSWER: A

Various cells of the oral mucous membrane and antimicrobial substances synthesized by these cells play an important part in the local immunity of the oral cavity. Specify the key factors for the local immunity: A. Secretory IgAB. B-lymphocytes C. IgG D. Macrophages E. Eosinophils ANSWER: A

Gynecological examination of the uterine cervix in a 30-year-old woman revealed some bright-red lustrous spots that easily bleed when touched. Biopsy showed that a part of the uterine cervix was covered with cylindrical epithelium with papillary outgrowths; in the depth of tissue the growth of glands was present. What pathology of the uterine cervix was revealed? A. PseudoerosionB. True erosion C. Endocervicitis D. Glandular hyperplasia E. Leukoplakia ANSWER: A

A stillborn child was found to have thickened skin resembling of the tortoise shell, underdeveloped auricles. Histological examination of skin revealed hyperkeratosis, atrophy of the granular epidermis layer; inflammatory changes were not present. What is the most likely diagnosis? A. Ichthyosis

B. Leukoplakia C. Xerodermia D. Erythroplakia E. Dermatomyositis ANSWER: A

A pathology-histology laboratory received a vermiform appendix up to 2,0 cm thick. Its serous membrane was pale, thick and covered with yellowish-green films. The wall was flaccid, of grayish-red colour. The appendix lumen was dilated and filled with yellowish-green substance. Histological examination revealed that the appendix wall was infiltrated with neutrophils. Specify the appendix disease: A. Acute phlegmonous appendicitisB. Acute gangrenous appendicitis C. Acute superficial appendicitis D. Acute simple appendicitis E. Chronic appendicitis ANSWER: A

A 35-year-old patient with chronic periodontitis underwent excision of a cyst 3 cm in diameter found at a root of the 15th tooth. Histological examination revealed that it had thin wall formed by mature connective tissue infiltrated by lymphocytes and plasmatic cells. Its internal surface was lined with multilayer pavement epithelium with no signs of keratinization; the cavity contained serous exudate. What is the most likely diagnosis? A. Radicular cystB. Follicular cyst C. Primordial cyst D. Cherubism E. Follicular ameloblastoma ANSWER: A

What preventive medications should be injected to a patient with open maxillofacial trauma provided that he has never got prophylactic vaccination before? A. Antitetanus immunoglobulin and anatoxinB. Anticonvulsive drugs and anatoxin C. Antitetanus serum and antibiotics D. Diphtheria, tetanus toxoids and pertussis vaccine and antibiotics E. Tetanus anatoxin and antibiotics ANSWER: A

Vestibular surface of the left lower incisor has a pink fungoid formation up to 2 cm large which is fixed to the supra-alveolar tissue by a wide pedicle. Histological examination revealed branched capillary vessels with multiple hemorrhages and foci of hemosiderosis. What is the most likely diagnosis? A. Angiomatous epulisB. Fibrous epulis C. Giant cell epulis D. Gingival fibromatosis E. Cavernous hemangioma ANSWER: A

The surgically excised connective tissue of the deformed mitral valve gives a basophilic reaction when stained with hematoxylin and eosin. When stained with toluidine blue it turns purple (metachromasia). What changes of the connective tissue can be detected by such reactions? A. Mucoid edemaB. Fibrinoid necrosis of connective tissue C. Connective tissue edema D. Petrification E. Hyalinosis ANSWER: A

A 69-year-old patient got a small plaque with subsequent ulceration on the skin of the lower eyelid. The formation was removed. Microscopic examination of dermis revealed complexes of atypical epitelial cells arranged perpendicularly to the basal membrane on the periphery. The cells were dark, of polygonal prismatic shape with hyperchromic nuclei with frequent mitoses. What is the histological form of carcinoma in this patient? A. Basal cell carcinomaB. Keratinizing squamous cell carcinoma C. Nonkeratinizing squamous cell carcinoma D. Adenocarcinoma E. Undifferentiated ANSWER: A

42 - year old man died 1 hour after the heart attack. At autopsy revealed irregular blood supply of myocardium, histologically - venous plethora. What histochemical method of research is necessary to confirm to ischemic myocardial injury?A. Stained with carminB. Stained with Sudan IIIC. Stained with trichrome by MassonD. Stained with hematoxylin and eosinE. Stained by van GiesonANSWER: A

42 year-old man suspected endomyocardial fibrosis died of congestive heart failure. At the autopsy revealed a sharp endocardial thickening of thrombotic formations and a significant decrease in ventricular cavities, fibrosis atrio-ventricular valves and papillary muscles. Specify the cardiomyopathy.A. RestrictiveB. HypertrophicC. CongestiveD. SymmetricE. DilatedANSWER: A

During postmortem study of brain 81-year old patient psychiatric clinic, which in life suffered from senile psychosis and died after a stroke, in vessels of the basis revealed multiple ulcerated yellow spots and stony density stenosing their lumen, blood clots. Specify pathological process that caused the thrombosis.A. AtherosclerosisB. Obliterating endarteritis C. Nodular periarteritisD. ThromboembolismE. Aneurysm

ANSWER: A

At ultrasonic examination the heart of a man who had a transmural myocardial infarction anterior wall of the left ventricle myocardial infarction 5 months ago revealed an aneurysm. Specify the type of aneurysm depending course of the disease.A. ChronicB. RecurrentC. SubacuteD. AcuteE. DissectionANSWER: A

At the autopsy 45 years old woman suffered from rheumatism and died of congestive heart failure revealed complete obliteration of the pericardial cavity with deposition of calcium in the fibrous tissue. Name the pathological process.A. Armored heartB. Hairy heartC. Tiger heartD. Stone heartE. Fibrotic heartANSWER: A

Tests for figures

1. Specify the form of tuberculosis (fig. 82).A. TuberculomaB. Focal tuberculosisC. Miliary tuberculosisD. Cirrhotic tuberculosisE. Fibrous cavernous tuberculosis*

2. Which form of tuberculosis is presented in this specimen (fig. 82)?A. PrimaryB. Hematogenically disseminatedC. InfiltrativeD. Primary tubercular complexE. Secondary*

3. What structure is marked by number 1 on the specimen of lungs at tuberculosis (fig. 82)?A. Lymph nodB. Lymphatic vesselsC. TuberculomaD. InfiltrateE. Cavern*

4. What structure is marked by number 2 on the specimen of lungs at tuberculosis (fig. 82)?A. CavernB. Lymphatic nodC. Lymphatic vesselsD. InfiltrateE. Draining bronchus*

5. What the internal layer of wall of cavern on a photomicrograph (fig. 85) marked by number 1 is presented by?A. Connective tissueB. Limphohystiocytic infiltrateC. Fibrous capsuleD. Pyogenic membraneE. Caseous мasses*

6. What the middle layer of wall of cavern on the photomicrograph (fig. 85) marked by number 2 is presented by?A. Connective tissueB. Caseous massesC. Fibrous capsuleD. Pyogenic membraneE. Epithelioid and giant cells*

7. What the external layer of wall of cavern on the photomicrograph (fig. 85) marked by number 3 is presented by?A. Limfohystiocitic іnfiltrateB. Caseous massesC. Epithelioid and giant cellsD. Pyogenic membraneE. Fibrous capsule*

8. Cells that are similar to the owl eye (marked by number 1) were discovered at microscopic research of lungs (fig. 86) of person who died from pneumonia. Specify the possible etiology of inflammation.A. Influenza virus pneumonia

B. Adenovirus infectionC. Pneumococcus infectionD. Staphylococcus pneumoniaE. Cytomegalovirus*

9. Specify the constituent of primary tubercular complex marked by number 2 (fig. 87).A. Focus of caseous necrosisB. LymphangitisC. LymphangoitisD. Simons’ focusE. Lymphadenitis*

10. Specify the constituent of primary tubercular complex marked by number 1(fig. 87).A. LymphangitisB. LymfangoitisC. LymphadenitisD. Simons’ focusE. Focus of caseous necrosis*

11. Specify the name of the subpleurally located and calcificated focus, marked by number 1 (fig. 88), in the person who died from pulmonary embolism.A. Aschoff”s-Pull’sB. Talalaev’sC. Simons’D. Koch’s focusE. Ghon’s focus*

12. Name the component part of primary tubercular complex marked by number 1 on specimen of lungs (fig. 88).A. LymphangitisB. TuberculomaC. LymphadenitisD. Site of invasionE. Primary аffect*

13. What form of tuberculosis is presented in this specimen (fig. 89)?A. TuberculomaB. CirrhoticC. Fibrous-cavernousD. Primary tubercular complexE. Miliary*

14. At hystological research of pulmonic tissue granulomas (fig. 90), that contained giant multi-nucleous cells marked by number 1 on the photomicrograph, were found. How is such granuloma named?A. TyphoidalB. Syphilitic C. Giant-cell granulomaD. EpithelioidE. Tuberculous*

15. At histological research of lung tissue tubercular granulomas were found (fig. 90). What component part of this granuloma is marked by number 1 on the photomicrograph?A. Sternberg cellB. Mikulich’s cellC. Koch’s cellD. Valdeer’s cellE. Langhans cell *

16. What component of tubercular granulosum is marked bynumber 1 on the microphotograph (fig. 91)?A. Epithelioid cellsB. Langhans’ cellsC. Ghon’s focusD. Connective tissueE. Focus of caseous necrosis*

17. What component part of tubercular granuloma is marked by number 2 on the microphotograph (fig. 91)?A. Epithelioid cellsB. Focus of caseous necrosisC. Ghon’s focusD. Connective tissueE. Langhans’ cells*

18. Specify the form of meningococcal infection in the patient (fig. 83).A. SepticopyemiaB. MeningopyemiyaC. Meningococcal meningitisD. Waterhouse-Friderichsen syndromeE. Meningococcemia*

19. Specify the form of мeningococcal infection (fig. 84).A. SepticopyemiaB. MeningopyemiyaC. MeningococcemiaD. Waterhouse-Friderichsen syndromeE. Meningococcal meningitis*

20. The following morphological picture was found at histological research of kidney biopsy material in the patient who had a scarlet fever 2 weeks ago (fig. 21). Diagnose.A. NormB. PyelonephritisC. Carbuncle of kidneyD. HydronephrosisE. Glomerulonephritis*

21. The morphological picture of glomerulonephritis was found at histological research of kidney tissue (fig. 21). What infection preceded this disease? A. MeaslesB. German measlesC. VaricellaD. MeningococcalE. Scarlet fever*

22. At endoscopical research a doctor found the changes of mucus membrane of stomach (fig. 20.B). What previous diagnosis did a doctor put?A. Acute gastritisB. Chronic gastritis with acuteeningC. Chronic hyperplastycal gastritis (Menetrie)D. There are no visible changes mucusE. Chronic atrophy gastritis*

23. At pathoanatomical research a doctor noticed a pied picture of stomach mucus membrane (fig. 20.C). What pathological process did a doctor see?A. Pigmental spots of stomach mucus.B. Catarrhal inflammation of stomach mucus.C. Perforation of stomach mucus.

D. Abscesses of stomach.E. Haemorrhagic erosion.*

24. Pathoanatomical macroscopic research found pathology of stomach (fig. 20.C). This pathology was confirmed by a microscopic method (fig. 20.D). What pathology do you see?A. Pigmental spots of stomach mucus.B. Catarrhal inflammation of stomach mucus.C. Perforation of stomach mucus.D. Abscesses of stomach.E. Haemorrhagic erosion of stomach mucus.*

25. At posthumous research of 40-years-old man a pathologist found pathology of mucus membrane of stomach (fig. 48.A). What did a doctor see?A. Acute fibrinous gastritisB. Chronic atrophic gastritisC. Chronic polypus gastritisD. Chronic gastric ulcerE. Acute gastric ulcer*

26. Pathoanatomical research of stomach found the pathology of mucus membrane in dying man (fig. 48.B). Your diagnosis is...A. Acute fibrinous gastritisB. Chronic atrophic gastritisC. Chronic polypus gastritisD. Acute gastric ulcerE. Chronic gastric ulcer*

27. At macroscopic research a pathologist found the pathology of stomach mucus (fig. 48.B). What pathology do you see?A. Fibrinous inflammation of mucus membraneB. The acid burn of mucus membrane on a small curvatureC. Hyperplastycal gastritis (Menetrie)D. Metaplasy of mucus membraneE. Atrophy and deformation of mucus membrane*

28. Pathoanatomical research of gastroenteric tract found the pathology ... (fig. 47.A)A. Acute gastric ulcerB. Chronic gastric ulcerC. Acute duodenitisD. Acute ulcer of duodenumE. chronic ulcer of duodenum *

29. In the pathoanatomical department was delivered a man which died in the cardiologic department. In the history of diseases was written that the man complained on pain in a stomach a long time. At research of internalss a pathologist discovered (fig. 47.B) ...A. Acute bleeding from a gastric ulcerB. Erosion of mucus stomachC. Acute bleeding from the ulcer of duodenumD. Perforated ulcer of duodenumE. erosive vessels on the bottom of duodenum chronic ulcer*

30. The pathoanatomical macroscopic research of gastroenteric tract discovered (fig. 47)A. Acute ulcer of duodenumB. Signs of repair of acute ulcer of duodenumC. Gastric ulcer with complication by bleedingD. Chronic gastric ulcer with arrosive vessels on a bottomE. chronic ulcer of duodenum with arrosive vessels on a bottom*

31. Macroscopic research of gastroenteric tract found pathology of mucus stomach (fig. 16.A). It is ...

A. Acute gastritisB. Chronic gastritis in the acute conditionC. Gastric ulcerD. Haemorragic gastritisE. Scar of mucus*

32. Macroscopic research of gastroenteric tract of man, which died from acute peritonitis, discovered ... (fig. 16.B)A. Perforated gastric ulcerB. PancreanecrosisC. Fatty hepatosis of liver («ancerine» liver)D. Muscat liverE. Perforated ulcer of duodenum*

33. Pathoanatomical research of stomach mucus discovered cicatrical tissue in the pylorus area. It became reason of ... (fig. 16.B)A. Gastric ulcersB. Ulcers of a duodenumC. Perforations of stomachD. Perforations of a duodenumE. Dilatations of stomach*

34. Pulmonary pathology became was the reason of death of 45-years-old woman. At macroscopic research of abdominal cavity found increased light color liver with speckle on a cut (fig. 17). Your diagnosis ...A. Liver steatosisB. Liver hemosiderosisC. Viral hepatitisD. Fatty necrosisE. Fatty hepatosis, muscat liver*

35. The acquired heart trouble was the reason of death of 50-years-old woman. Pathoanatomical research found increased flabby liver with an antypical light color and speckle on a cut (fig. 17). Your diagnosis ...A. HepatitisB. CholestasisC. Extramedullar areas of haemopoesisD. HemosiderosisE. Muscat liver*

36. The congenital pathology of heart was a reason of death of 25-years-old man. At pathoanatomical research of internalss a doctor discovered (fig. 17)... A. Hyaline-dropped dystrophy of liverB. Hemosiderosis of liverC. Cholestasis of liverD. Liver without pathological signsE. Muscat liver*

37. After surgical operation in the abdominal cavity the ectomied organ was presented to the pathologist on research (fig. 18.A.). Your diagnosis ...A. Acute phlegmonous appendicitisB. Acute fibrinous-festering appendicitisC. Acute destructively-necrotizing appendicitisD. Chronic appendicitisE. Acute catarrhal appendicitis*

38. After appendectomy the appendix was given to the pathologist on research (fig. 18.B.). Your diagnosis ...A. Abscess of appendix

B. Fibrinous appendicitisC. Phlegmonous appendicitisD. Chronic appendicitisE. Mucocele (mucocyst)*

39. The removed at the operation appendix became the object of pathoanatomical research. Result of investigation using the original view of organ (fig. 18.A.).A. Acute fibrinous-festering appendicitisB. Acute phlegmonous appendicitisC. Acute destructively-necrotizing appendicitisD. Chronic appendicitisE. Acute catarrhal fibrinous appendicitis*

40. Acute festering-destructive appendicitis was the reason of death of man. At pathoanatomical research (fig. 19.A.) a doctor sees ...A. Acute serous peritonitisB. Necrosis of colonC. Necrosis of intestineD. Chronic peritonitis with connections formationE. Acute fibrinous-festering peritonitis*

41. Pathologist explored the abdominal cavity of dying where perforated ulcer was the reason of death (fig. 19.A.). Name the direct reason of death of this man.A. Acute hemorragic anaemiaB. Chronic hemorragic anaemiaC. Necrosis of intestineD. Necrosis of colonE. Acute fibrinous-festering peritonitis*

42. The pathologist investigated the died man of 42 age, which suddenly died from a stroke. At the study of abdominal cavity organs (fig. 19.B.) a doctor saw ...A. Neoplasm of intestineB. Fibrinous-festering peritonitisC. simple catarrhal peritonitisD. unchanged intestineE. Adhesions which were the result of past peritonitis*

43. Specify the histological structure marked by number 1 on the microphotograph of kidney (fig 23).A. Bowman's capsuleB. Henle's loopC. Ascending ductuleD. Renal pyramidE. Glomerule of capillars*

44. Diagnose the pathological process on the microphotograph of kidney (fig 23).A. Extracapillar glomerulonephritisB. Serosal glomerulonephritisC. PyelonephritisD. Renal cell carcinoma (hypernephroma)E. Intracapillar glomerulonephritis*

45. Specify changes in the epithelium of tubules of kidneys marked by number 3 (fig 24).A. Necrotic nephrosisB. AmyloidosisC. HyalinosisD. HemosiderosisE. Hydropic [vacuolar] degeneration*

46. Specify the histological structure marked by number 3 on the microphotograph of kidney (fig 24).A. Bowman's capsuleB. Glomerule of capillarsC. Renal pelvisD. Renal pyramidE. Renal tubule*

47. What character of clinical course has a glomerulonephritis, morphological picture of which is presented on the microphotograph (fig 24)?A. ChronicB. AcuteC. RecurrentD. Permanent-paroxismalE. Subacute*

48. Diagnose the pathological process on the microphotograph of kidney (fig 24)A. Serosal glomerulonephritisB. Hemorrhagic glomerulonephritisC. PyelonephritisD. Renal cell carcinoma (hypernephroma)E. Extracapillar glomerulonephritis*

49. The following histological picture of kidneys is observed in the person who died from pulmonary bleeding caused by cavernous fibrous tuberculosis (fig 56). Diagnose changes.A. NephrosclerosisB. HyalinosisC. GlomerulonephritisD. PyelonephritisE. Amyloidosis*

50. What histological structure is marked by number 2 on microslide (fig 56)?A. Kidney glomeruleB. Bowmen’s capsuleC. Henle’s loopD. Renal pyramidE. Kidney tubule*

51. What dye is used for the exposure of amyloid in this microslide (fig 56)?A. Sudan IIIB. After HeidenhignC. After MaloraD. After Van Hison E. Congo red*

52. The histological picture of amyloidosis of kidneys is observed in the person who died from pulmonary bleeding caused by cavernous fibrous tuberculosis (fig 56). Specify the form of amyloidosis.A. IdiopathicB. SenileC. InheritedD. LocalE. Secondary*

53. What is the most frequent reason of these histological changes in kidneys (fig 46)?A. Embolia of kidney arteryB. Atherosclerosis of kidney arteriesC. Calculous pyelonephritisD. Streptococcus infection

E. Mushrooms poisoning*54. What syndrome will predominate in the clinical presentation of patient with the following

histological changes in kidneys (fig 46)?A. Chronic kidney insufficiencyB. Symptomatic arterial hypertensionC. OedematousD. Septic shockE. Acute kidney insufficiency*

55. Specify the histological structure marked by number 1 on the microphotograph of kidney (fig 46).A. Bowman's capsuleB. Henle's loopC. Ascending ductuleD. Renal pyramidE. Glomerule of capillars*

56. Specify a histological structure marked by number 2 on the microphotograph of kidney (fig 46).A. Bowman's capsuleB. Glomerule of capillarsC. Hosh's loopD. Renal pyramidE. Kidney ductule*

57. Biopsy material of kidney was delivered to laboratory (fig 21). Diagnose.A. NephrosclerosisB. AmyloidosisC. HyalinosisD. PyelonephritisE. Glomerulonephritis*

58. Biopsy material of kidney was delivered to laboratory (fig 21). Diagnose.A. Intracapillary glomerulonephritisB. Amyloidosis of kidneyC. Subacute glomerulonephritisD. PyelonephritisE. Serosal extracapillary glomerulonephritis*

59. After what infection do the following pathological changes of kidneys develop more frequently (fig 21)?A. StaphylococcalB. Viral (flu)C. TubercularD. MycoticE. Streptococcic *

60. What changes has developed in the wall of urinary bladder (fig 22)?A. AtrophyB. Physiological hypertrophyC. HiperplasiaD. Vicarious [substitutional] hypertrophyE. Work hypertrophy*

61. What pathological process did cause changes in the wall of urinary bladder (fig 22)?A. CystitisB. Calculous pyelonephritisC. GlomerulonephritisD. Amyloidosis of kidneys

E. Benign adenoma of prostate*62. What complications can develop in kidneys in presence of such changes from the prostate

gland (fig 22)?A. GlomerulonephritisB. Polycystic kidneyC. Contracted [granular] kidneyD. Arteriolosclerotic kidneyE. Pyelonephritis*

63. Histological examination of lung tissue (Fig.75) revealed dense protein masses, which are colored intensely with eosin, and are on the respiratory regions of the lungs. Hyaline membranes composed of fibrin. They block the exchange of gases in the lungs. What kind of pathology: A. Pathology of the placenta B. Fetopathy C. Blastopathy D. Antenatal PathologyE. Perinathal pathology * *

64. Histological examination of lung tissue (Fig.75) revealed dense protein masses, which are colored intensely with eosin, and are on the respiratory regions of the lungs. Hyaline membranes composed of fibrin. They block the exchange of gases in the lungs. What kind of pathology:A. Pathology of the placenta B. Fetopathy C. Blastopathy D. Antenatal PathologyE. Pneumopathy * *

65. What is the pathology of lung tissue is on Fig.75. Histology revealed dense protein masses, which are colored intensely with eosin, and firmly stick to the walls of respiratory regions of the lungs. Hyaline membranes are composed of fibrin and blocks gas exchange in the lungs.A. Pathology of the placenta B. Fetopathy C. Blastopathy D. Antenatal PathologyE. Pneumopathy*

66. What is the pathology of lung tissue in Fig.75. Histology revealed dense protein masses, which are colored intensely with eosin, and firmly stick to the walls of respiratory regions of the lungs. Hyaline membranes are composed of fibrin and blocks gas exchange in the lungs.A. Pathology of the placentaB. FetopathyC. BlastopathyD. Perinatal pathologyE. Congenital malformations of the respiratory system **

67. Grossly the brain tissue preterm neonates revealed a hematoma in the cavity of the ventricles with the development of tamponade (Fig.76). What are the causes of birth injury, embodied in the state of the fetus:A. Embriopathy - malformations, which are accompanied by venous congestion in the

tissues of the fetusB. Fetopathy, which are accompanied by haemorrhagic syndromeC. Rigidity tissue birth canalD. All of the optionsE. Premature birth or fetal postmaturity **

68. Grossly the brain tissue preterm neonates revealed a hematoma in the cavity of the ventricles with the development of tamponade (Fig.76). What are the causes of birth trauma rooted in the mother's birth canal:A. Distortion of the pelvisB. Tumors of the birth canalC. oligohydramnios and premature rupture of membranesD. Rigidity tissues birth canalE. All of these options correct*

69. What are the causes of birth injury, embodied in the mother's birth canal, which lead to the development of pathologic changes, shown in Fig. 76. Grossly the brain tissue in a premature newborn revealed a hematoma in the cavity of the ventricles with the development of tamponade.A. Distortion of the pelvisB. Tumors of the birth canalC. Oligohydramnios and premature rupture of membranesD. Rigidity tissues birth canalE. All of these options correct*

70. Grossly the brain tissue preterm neonates revealed a hematoma in the cavity of the ventricles with the development of tamponade (Fig.76). What are the possible causes of birth trauma rooted in the dynamics of birth:A. fetal hypoxiaB. oligohydramnios and premature rupture of membranesC. Rigidity tissues birth canalD. All of these options correctE. accelerated labor**

71. Grossly tissue of the abdominal cavity in a child who died at 2 days was found additional spleen (Fig.77) in the figure indicated by the arrow. What it is:A. AplasiaB. HypoxiaC. EctopiaD. All of the optionsE. Congenital hypertrophy **

72. Grossly tissue of the abdominal cavity in a child who died at 2 days was found additional spleen (Fig.77) in the figure indicated by the arrow. Specify in what time period are formed such changesA. Period blastogenesisB. Period fetogenezaC. Early fetal periodD. postural fetal periodE. Period embryogenesis*

73. Grossly tissue of the abdominal cavity of the child, who died at 2 days, found additional spleen (Fig.77) in the figure indicated by the arrow. Indicate to what pathology include these changes.A. BlastopathyB. FetopathyC. Early fetal periodD. Postural fetal periodE. Embriopathy*

74. At Fig.78 shows kefalogematoma newborn baby. What the reason for this pathological process:A. PolyhydramniosB. Blastopathy

C. EmbriopathyD. FetopathyE. Overlay forceps **

75. At Fig.78 shows kefalogematoma newborn baby. What the reason for this pathological process:A. PolyhydramniosB. BlastopathyC. EmbriopathyD. FetopathyE. Accelerated labor **

76. At Fig.78 shows kefalogematoma newborn baby. What the reason for this pathological process:A. PolyhydramniosB. BlastopathyC. EmbriopathyD. FetopathyE. Prolonged labor*

77. At Fig.78 shows kefalogematoma newborn baby. What the reason for this pathological process:A. PolyhydramniosB. BlastopathyC. EmbriopathyD. FetopathyE. State of prematurity or newborn postmaturity**

78. At Fig.78 shows kefalogematoma newborn baby. What the reason for this pathological process:A. PolyhydramniosB. BlastopathyC. EmbriopathyD. FetopathyE. Oligohydramnios*

79. Women 32 years removed uterine tube. Histologically, it revealed chorionic villi, which are depicted on Fig.80. Name the cause of intervention:A. Abdominal PregnancyB. Ovarian PregnancyC. Uterine pregnancyD. Vesicular moleE. Tubal pregnancy*

80. After removal of uterine tubes of women 32 years old, histologically revealed chorionic villi, which are depicted on Fig.80. Name the cause of intervention:A. Abdominal PregnancyB. Ovarian PregnancyC. Uterine pregnancyD. Vesicular moleE. Tubal pregnancy*

81. The presence of decidual reaction and chorionic villi shown in Fig.80 shows:A. Specific granulomatous inflammationB. Nonspecific granulomatous inflammationC. Аcute inflammationD. Chronic inflammationE. Pregnancy **

82. A young woman was operated on urgently rupture uterine tube . Gistologicheski reaction revealed decidua and chorionic villi (fig.80).This is typical:A. Specific granulomatous inflammationB. Nonspecific granulomatous inflammationC. Acute inflammationD. Chronic inflammationE. Pregnancy **

83. What pathology is shown in Fig.55 the number 3:A. uterine pregnancyB. tubal pregnancyC. teratomaD. Corpus luteumE. Ovarian Pregnancy **

84. Name the structure shown in Fig.79.A. embryoblastB. trophoblastC. EmbryonD. ChorionE. Fetus **

85. In what period of developing pathology shown on Fig.81.A. GametopathyB. EmbriopathyC. CongenitalD. bicuspid aorticE. Blastopathy*

86. Aentsefaly - agenesis of the brain, shown in Fig. 52 refers to:A. GametopathyB. BlastopathyC. Afterbirth pathologyD. HypoplasiaE. Embriopathy **

87. What pathology is shown in Fig.51A. Hypoplasia of muscle tissueB. DysplasiaC. chondrodysplasiaD. Perverted OsteogenesisE. Congenital face-heyloshiz*

88. At fig. 36 shows epicardium covered with fibrin. Indicate the type of inflammation.A. HemorrhagicB. PurulentC. SerousD. MixedE. Fibrinous*

89. Indicate the type of inflammation in Fig. 36 which shows the epicardium covered with fibrin.A. HemorrhagicB. PurulentC. SerousD. MixedE. Fibrinous*

90. At the heart of dermatomyositis (fig.45) observed dystrophy, cardiomyocytes mediate myocarditis, and the pathological process is completed:A. The partial regeneration of cardiomyocytes

B. Complete regeneration of cardiomyocytesC. Pathological cardiomyocyte regenerationD. Focal elastofibrosisE. Diffuse cardiosclerosis atrophy of cardiomyocytes*

91. A characteristic feature of lupus glomerulonephritis (fig.35) is the deposition: A. Non-immune complexes and thickening of the capillaries in the form of " wire sticks" B. Immune complexes and thickening of the capillaries in the form of " wire Rings C. Non-immune complexes and thickening of the capillaries in the form of " wire Rings D. Non-immune complexes and utonsheniem capillaries in the form of " wire loops" E. Immune complexes and thickening of capillaries in the form of " wire loops" *

92. What are the characteristic feature of lupus glomerulonephritis (fig.35):A. Non-immune complexes and thickening of the capillaries in the form of " wire sticks" B. Immune complexes and thickening of the capillaries in the form of " wire Rings C. Non-immune complexes and thickening of the capillaries in the form of " wire Rings D. Non-immune complexes and utonsheniem capillaries in the form of " wire loops" E. Immune complexes and thickening of capillaries in the form of " wire loops" *

93. What is a characteristic feature of lupus glomerulonephritis (fig.35):A. Non-immune complexes and thickening of the capillaries in the form of " wire sticks" B. Immune complexes and thickening of the capillaries in the form of " wire Rings C. Non-immune complexes and thickening of the capillaries in the form of " wire Rings D. Non-immune complexes and utonsheniem capillaries in the form of " wire loops" E. Immune complexes and thickening of capillaries in the form of " wire loops" *

94. The consequence of lupus glomerulonephritis (fig.35) is the development of:A. Kidney decreasingB. Wrinkled kidneysC. Cystic expansion of renalD. Hydronephrosis KidneyE. Motley kidney*

95. What are the characteristic feature of lupus glomerulonephritis (fig.35): A. Non-immune complexes and thickening of the capillaries in the form of " wire sticks" B. Immune complexes and thickening of the capillaries in the form of " wire Rings C. Non-immune complexes and thickening of the capillaries in the form of " wire Rings D. Non-immune complexes and utonsheniem capillaries in the form of " wire loops" E. Immune complexes and thickening of capillaries in the form of " wire loops" *

96. Focal exudative interstitial myocarditis manifested (fig.45):A. To a large focal interstitial infiltration by lymphocytesB. To a large focal interstitial infiltration eosinophilsC. To a large focal interstitial infiltration of plasma cellsD. Ashof- bodyE. The small focal interstitial infiltration of lymphocytes, histiocytes and neutrophils*

97. What is manifested focal interstitial myocarditis, exudative (fig.45):A. To a large focal interstitial infiltration by lymphocytesB. To a large focal interstitial infiltration eosinophilsC. To a large focal interstitial infiltration of plasma cellsD. Ashof- bodyE. The small focal interstitial infiltration of lymphocytes, histiocytes and neutrophils*

98. The development of sclerosis myocarditis completed (fig.44) at:A. Not a favorable flow of rheumatismB. The occurrence of rheumatic disease with frequent relapsesC. The occurrence of rheumatic fever with high activity indices of bloodD. Predominance of endocarditisE. Favorable course of rheumatism*

99. What are the nature of inflammation in the pericardium (fig.36):A. Mixed inflammationB. Purulent exudative inflammationC. Hemorrhagic exudative inflammationD. Catarrhal exudative inflammationE. Serous exudative inflammation*

100. What is the nature of inflammation in the pericardium on fig.36:A. Mixed inflammationB. Purulent exudative inflammationC. Hemorrhagic exudative inflammationD. Catarrhal exudative inflammationE. Serous exudative inflammation*

101. Pericarditis rheumatism has character (Fig.36):A. Mixed inflammationB. Purulent exudative inflammationC. Hemorrhagic exudative inflammationD. Catarrhal exudative inflammationE. Serous exudative inflammation*

102. Frequently rheumatic pericarditis completed education (Fig.36):A. Suppuration pericardial cavityB. Papillary proliferation of the pericardiumC. InflammationD. Рapillary transformation of the epicardiumE. Adhesions pericardial cavity*

103. The result of pericarditis (Fig.36) is education:A. Suppuration pericardial cavityB. Papillary proliferation of the pericardiumC. InflammationD. Рapillary transformation of the epicardiumE. Adhesions pericardial cavity*

104. Most rheumatic pericarditis (Fig.36) concludes obliteration of the cavity of the heart bags with calcification, formation of connective tissue in this case leads to the development:A. Cor PulmonaleB. Tiger HeartC. Hairy heartD. Bovine heartE. Stone heart*

105. Histological examination of kidney tissue revealed focal and diffuse thickening of the basal membrane with the formation of " wire loops " Fig.37. Occurs appearance gematoksiliny body. What clinical and anatomical form of the disease.A. CardiacB. ArthriticC. MixedD. CerebralE. Visceral*

106. Histological examination of kidney tissue revealed focal and diffuse thickening of the basal membrane with the formation of " wire loops " Fig.37. Occurs appearance gematoksiliny body. What clinical and anatomical form of the disease.A. CardiacB. ArthriticC. MixedD. Cerebral

E. Visceral*107. What clinical and anatomical form of the disease on Pic.37. Histologically in the study of

kidney tissue revealed focal and diffuse thickening of the basal membrane with the formation of " wire loops”. Appear gematoksilinovye calfA. CardiacB. ArthriticC. MixedD. CerebralE. Visceral*

108. Specify (Fig.1) fibrinous exudate (lobar pneumonia):A. 2B. 3C. 4D. 5E. 1*

109. Specify leukocyte infiltration in lobar pneumonia (Fig.1):A. 1B. 3C. 4D. 5E. 2*

110. Specify (Fig.1) the thickened walls of the alveoli (lobar pneumonia):A. 1B. 2C. 4D. 5E. 3*

111. Specify leukocyte infiltration in lobar pneumonia (Fig.1):A. 1B. 3C. 4D. 5E. 2*

112. Name the pathological process (Fig.1):A. bronchopneumoniaB. bronchitisC. pneumoniaD. larengitE. lobar pneumonia*

113. Select the pus in the abscess cavity (Fig.8):A. 2B. 3C. 4D. 5E. 1*

114. Select the wall abscess (Fig.8):A. 1B. 3C. 4D. 5E. 2*

115. Select the dilated vessels in the lung abscess (Fig.8):

A. 1B. 2C. 4D. 5E. 3*

116. Select the neutrophilic infiltration in the lung abscess (Fig.8):A. 1B. 2C. 3D. 5E. 4*

117. What are the pathological process (Fig.8):A. lobar pneumoniaB. bronchitisC. cancerD. larengitE. abscess*

118. Specify (Fig.9) the leucocyte infiltration (bronchopneumonia):A. 2B. 3C. 4D. 5E. 1*

119. Specify (Fig.9) the exudate in the bronchial lumen (bronchopneumonia):A. 1B. 3C. 4D. 5E. 2*

120. Specify (Fig.9) the advanced vessels (bronchopneumonia):A. 1B. 2C. 4D. 5E. 3*

121. Specify (Fig.9) the thickened walls of the alveoli (bronchopneumonia):A. 1B. 2C. 3D. 5E. 4*

122. What is the pathological process in this figure (Fig.9):A. lobar pneumoniaB. bronchitisC. pneumoniaD. larengitE. bronchopneumonia*

123. What are the pathological process (Fig.10):A. bronchopneumoniaB. bronchitisC. pneumoniaD. larengit

E. carnification*124. Specify (Fig.11) the leukocyte infiltration (intermediate pneumonia):

A. 2B. 3C. 4D. 5E. 1*

125. Specify (Fig.11) the extended alveolar walls (intermediate pneumonia):A. 1B. 3C. 4D. 5E. 2*

126. Specify (Fig.11) the advanced vessels (intermediate pneumonia):A. 1B. 2C. 4D. 5E. 3*

127. What are the pathological process (Fig.11):A. pneumonia in influenzaB. viral bronchitisC. pneumoniaD. Viral larengitE. intermediate pneumonia*

128. Specify (Fig.12) the leukocyte infiltration (chronic bronchitis):A. 2B. 3C. 4D. 5E. 1*

129. Specify (Fig.12) the advanced vessels (chronic bronchitis):A. 1B. 2C. 4D. 5E. 3*

130. Specify (Fig.12) the exudate in the lumen of the bronchus (chronic bronchitis):A. 1B. 3C. 4D. 5E. 2*

131. Name (Fig.12) the pathological process:A. bronchopneumoniaB. intermediate pneumoniaC. pneumoniaD. larengitE. bronchitis*

132. Specify (Fig.13) the leukocyte infiltration (chronic bronchitis):A. 2B. 3

C. 4D. 5E. 1*

133. Specify (Fig.13) the advanced vessels (chronic bronchitis):A. 1B. 2C. 4D. 5E. 3*

134. Specify (Fig.13) the exudate in the lumen of the bronchus (chronic bronchitis):A. 1B. 3C. 4D. 5E. 2*

135. Name (Fig.13) the pathological process:A. bronchopneumoniaB. intermediate pneumoniaC. pneumoniaD. larengitE. chronic bronchitis*

136. Specify (Fig.14) advanced lumen alveoli (emphysema):A. 2B. 3C. 4D. 2 i 4E. 1*

137. Specify (Fig.14) thinning alveolar septum:A. 1B. 3C. 1 i 4D. 1 i 2E. 2 i 4*

138. Name the pathological process (Fig.14):A. bronchopneumoniaB. intermediate pneumoniaC. pneumoniaD. larengitE. emphysema*

139. Specify hyperchromic nuclei (Fig.15):A. 2 and 3B. 3 and 1C. 3D. 2E. 1*

140. Specify the cancer cells (Fig.15):A. 1B. 2 and 3C. 3D. 3 and 1E. 2*

141. What type of cell atypizm can be seen in the picture (Fig.15):

A. biochemicalB. fabricC. intercellularD. intersticesE. cell and tissue*

142. Name the pathological process (Fig.15):A. bronchopneumoniaB. intermediate pneumoniaC. pneumoniaD. larengitE. lung*

143. What type of cell atypizm is numbered 1 (Fig.15):A. hipohromii nucleiB. different size fibersC. atypical location of glandsD. irregular shape of glandsE. hyperchromic nuclei*

144. What type of cell atypizm can be seen (Fig.15):A. cancer than in normalB. different size fibersC. Atypical location of glandsD. irregular shape of glandsE. polymorphism nuclei*

145. What type of cell atypizm can be seen (Fig.15):A. cancer than in normalB. different size fibersC. Atypical location of glandsD. irregular shape of glandsE. different size nuclei*

146. Specify the hyperchromicity nuclei (Fig.2):A. 2 and 3B. 3 and 1C. 3D. 2E. 1*

147. Specify (Fig.2) the cancer cells:A. 1B. 2 and 3C. 3D. 3 and 1E. 2*

148. What types of atypizm can be seen (Fig.2):A. biochemicalB. fabricC. intercellularD. intersticesE. cell and tissue*

149. Name the pathological process (Fig.2):A. bronchopneumoniaB. intermediate pneumoniaC. pneumoniaD. larengit

E. lung*150. What kind of cell atypical numbered 2 (Fig.2):

A. hyperchromic nucleiB. different size fibersC. atypical location of glandsD. irregular shape of glandsE. hiporhromiya nuclei*

151. What type of cell atypizm can be seen (Fig.2):A. cancer than in normalB. different size fibersC. Atypical location of glandsD. irregular shape of glandsE. polymorphism nuclei*

152. What type of cell atypizm can be seen (Fig.2):A. cancer than in normalB. different size fibersC. Atypical location of glandsD. irregular shape of glandsE. different size nuclei*

153. Name the pathological process (Fig.3):A. lobar pneumoniaB. tracheitisC. larengitD. lungE. bronchopneumonia*

154. What is a pathological process (Fig.4)?A. lobar pneumoniaB. tracheitisC. larengitD. lungE. bronchoalveolitis*

155. Name the pathological process (Fig.5):A. lobar pneumoniaB. tracheitisC. larengitD. lungE. Bullous emphysema*

156. Specify (Fig.7) the diagnosis:A. bronchitisB. tracheitisC. laryngitisD. pericarditisE. cancer of the lung*

157. ?What pathological processes caused the name of spleen ("porphyry") in lymphogranulomatosis (Fig. 26)?A. AmyloidosisB. HyalinosisC. Necrosis and amyloidosisD. Sclerosis and hyalinosisE. Necrosis and sclerosis*

158. The specific pathognomonic sign of this type of anaemia is atrophy of fundal mucus cells in stomach. Name disease. (Fig.49)

A. Acute posthemorrhagicB. Chronic posthemorrhagicC. HemolyticD. AplasticE. Pernicious (vit. B 12-deficiency)*

159. The sign of what pathological process is decrease of glands in the mucus layer of stomach (a number 1 on Fig. 49) at pernicious (vit. B 12-deficiency) anemia?A. HyperplasiaB. HypertrophyC. AplasiaD. SclerosisE. Atrophy*

160. At histological research of stomach (Fig. 49) patient with anaemia founded atrophy of glands and sclerosis (1), focal lymphocytes infiltration (2). Name the type of anaemia.A. Acute posthemorrhagicB. Chronic posthemorrhagicC. HemolyticD. AplasticE. Pernicious (vit. B 12-deficiency)*

161. At histological examination of lymph node revealed of Hodgkin's giant single - nucleus cells (Fig. 25). What pathological process shown in the lymph node slide? A. Acute leukemia B. Ekstramedullar hemopoiesisC. Chronic erythromyelosisD. Chronic lympholeukosis E. Lymphogranulomatosis*

162. At histological examination of lymph node diagnosed lymphogranulomatosis (fig.25). Determine group of hemopoietic and lymphatic tissue diseases? A. Pretumor disease B. Immune inflammation C. Systemic disease hemopoietic tissue D. Malignant tumor of mesenhyma E. Regional tumor - lymphomas*

163. At histological examination of lymph node in patient with lymphogranulomatosis detected giant single - nucleus cells (fig.25). Name the cells marked by arrow on the lymph node slide.A. Pirogov-Langhans' giant cells B. Mykulich' cells C. Eosinophil D. Berezovsky-Sternberg' cells E. Hodgkin' cells *

164. In patients diagnosed lymphogranulomatosis. What specific cells founded in the lymph nodes (Fig. 25)? A. Pirogov-Langhans' giant cells B. Mykulich' cells C. Anichkov' cells D. Virchow' cells E. Hodgkin' cells *

165. In patients diagnosed lymphogranulomatosis. What specific cells founded in the lymph nodes (Fig. 25)? A. Pirogov-Langhans' giant cells B. Mykulich' cells

C. Anichkov ' cells D. Virchow' cells E. Berezovsky-Sternberg' cells *

166. Autopsy of a woman, who died from lymphogranulomatosis revealed enlarged spleen. On the section - multiple white-yellow areas of necrosis and sclerosis (Fig. 26). What is named of such spleen?. A. SebaceousB. Sago-like spleenC. Glazed D. Hyperplastic E. Porphyry*

167. Autopsy of a woman, who died from lymphogranulomatosis revealed enlarged spleen. On the section - multiple white-yellow areas of necrosis and sclerosis (porphyry) (Fig. 26). Name the diseases which characterizing such spleen. A. Acute leukemia B. Ekstramedullar hemopoiesisC. Chronic miyeloleykosis D. Chronic lymphocytic leukemia E. Lymphogranulomatosis*

168. At histological examination of the stomach (Fig. 49) patients with anemia founded glands atrophy and sclerosis (1), focal lymphocytic infiltration (2). Name the cause of this phenomen. A. Iron deficiency B. Excess vit. B12 C. Deficiency of folic acid D. Bone marrow aplasia E. Deficiency of vit. B12 *

169. ?Histologically in the tissue of the pancreas revealed B-insuloma (Figure 74). Specify the clinical manifestation of this pathology. A. Hyperglycemic syndrome B. Obstructive jaundice C. Ellison - Zolinger syndrome D. Kimmelstil-Wilson syndrome E. Hypoglycemic syndrome *

170. Clinically and histologically of a patient diagnosed insuloma (Fig. 74). Name the rise of this tumor. A. plasmocytesB. endocrinic cells C. gland cellsD. ductal epithelium E. B-cells of the pancreas *

171. Histologically the tumor tissue removed from the pancreas composed from B-cells (Fig. 74). Clinically characterized by hypoglycemic syndrome. Whichof the listed diagnoses is most probable? A. Regenerative hyperplasia B. Embrioblastoma C. Adenoma D. Adenoacanthoma E. Insuloma *

172. Histological examination of biopsy of the thyroid gland revealed granulomas which contain from giant multinuclear cells (Fig. 66) and "bricked" among the fibrous connective tissue. Which of the listed form of the thyroiditis is most probable?

A. Hashimoto B. Riedel C. Graves' D. Basedow E. De Quervain *

173. The removed thyroid gland enlarged whith uneven nodular surface (Fig. 67). Histologically - colloid goiter. Which of the listed macroscopic form of goiter is most probable? A. Diffuse B. Mixed C. Parenchymal D. Polypous E. Nodular *

174. In the patient aged 52 removed nodular goiter (Fig. 67). Which of the listed microscopic form of goiter is most probable? A. Parenchymal B. SporadicC. Autoimunny D. Fibrotic E. Colloid*

175. In the patient aged 52 removed nodular goiter (Fig. 67). After some time she had symptoms of hypopituitarism - hypotension, edema, dry skin, bradycardia. Which of the listed diagnoses is most probable? A. Graves' disease B. Basedow' goiter C. Goitre Hashimoto D. Goiter Riedel E. Myxedema *

176. On a fig.33 pointer is indicate the wall of sharp abscess of lung. What tissue is it presented?A. by young granulationB. by mature granulationC. unchanged lung tissueD. fibroticE. changed lung tissue by necrosis and inflamation*

177. An autopsy of a 34-year-old male, who died from croupous (lobar) pneumonia (Fig. 33), revealed defect with irregular scalloped edges and yellow-green contents. Name the type of pulmonary complication, marked a pointer:A. chronic abscessB. carnificationC. colliquationD. caseous necrosisE. acute abscess*

178. Name a pathogenetic mechanism which is underlaid of abscess formation at croupous (lobar) pneumonia (Fig.33).A. hypoxiaB. sclerosingC. decreasing of proteolytic properties of leucocytesD. inflammatory hyperemiaE. increasing of proteolytic properties of leucocytes*

179. At histological research of lung tissue (Fig.28) revealed a hemorragic exsudate in alveoli lumen with infiltration by leucocytes (1). Name the most credible etiologic factor of pneumonia.A. clamudiaB. staphylococcusC. pneumococcusD. fungusE. virus of influenza*

180. At histological research of lung tissue (Fig.28) revealed a leucocytes infiltration of bronchial walls and alveolar septa, hemorragic exsudate in alveoli lumen with leucocytes (1). Name a pathological process in lung.A. hemorrhageB. acute bronchitisC. red hepatizationD. infarction of lungE. hemorragic bronchopneumonia*

181. At histological research of lung tissue (Fig.28) revealed a leucocytes infiltration of bronchial walls and alveolar septa, hemorragic exsudate in alveoli lumen with leucocytes (1). Diagnosing the morphological form of pneumonia.A. focal pneumoniaB. infarction-pneumoniaC. traumatic pneumoniaD. congested pneumonia E. hemorragic bronchopneumonia*

182. At histological investigation of lung tissue (Fig.34) revealed exsudate with large maintenance of fibrin in alveoli lumen (1). At a bacteriologic examination founded pneumococcus I and the II types. Name the morphological form of pneumonia.A. focal fibrinousB. draining fibrinousC. lobularD. pneumocysticE. croupous (lobar) *

183. At histological investigation of lung tissue (Fig.34) revealed exsudate with large maintenance of fibrin in alveoli lumen (1).The diagnos of croupous pneumonia was proposed. Name the etiologic factor of pneumonia.A. collibacillusB. staphylococcusC. Lefler's bacillaD. fungiE. pneumococcus*

184. At histological investigation of lung tissue (Fig.34) dying from croupous pneumonia revealed exsudate with large maintenance of fibrin (1) and leucocytes in alveoli lumen (2). Name the stage of pneumonia.A. red hepatizationB. initial stageC. microbal edemaD. resolutionE. yellow hepatization*

185. Name the morphological component of focal pneumonia marked a number 1 (Fig. 27).A. atelectasisB. inflammatory exsudateC. bronchiolitis

D. alveolitisE. emphysema*

186. Name the morphological component of focal pneumonia marked a number 2 (Fig. 27).A. atelectasisB. inflammatory exsudateC. emphysemaD. alveolitisE. bronchitis*

187. At histological investigation (Fig.27) revealed a bronchitis (2) and area of exsudate inflammation of pulmonary parenchima round a bronchi. Name the morphological form of pneumonia.A. alveolar-bronchialB. acinarC. croupousD. congestedE. focal*

188. In the autopsy the lobe of the lung marked by the arrow in Fig. 29 is gray with a superimposition of fibrin on the visceral pleura and has liver density. Name the type of pneumonia. A. bronchopneumonia B. influenza C. indurational D. interstitial E. lobar *

189. In the autopsy the lobe of the lung, marked by the arrow in Fig. 29, dense, gray, with several layers of fibrin on the visceral pleura. Diagnosed lobar pneumonia. Name stage of pneumonia. A. red hepatizationB. initial stageC. microbal edema D. resolutionE. gray hepatization*

190. In the autopsy the lobe of the lung, marked by the arrow in Fig. 29, is gray with several layers fibrin on the visceral pleura and has liver density. Was diagnosed lobar pneumonia. Name the causative factor of pneumonia. A. colibacillus B. staphylococcus C. Leffler's bacillusD. fungi E. pneumococcus *

191. At histological examination of lung tissue (Fig. 30) revealed multiple, continues, enlarged airspaces from less then 0,5 mm to more then 2.0 cm in diameter (2), sometimes forming cyst-like structures (1). Name the pathological process. A. Interstitial emphysema B. Idiopathic emphysema C. Centriacinar emphysema D. Senile emphysema E. Unilateral emphysema*

192. At histological examination of lung tissue (Fig. 30) revealed multiple, continues, enlarged airspaces from less then 0,5 mm to more then 2.0 cm in diameter (2), sometimes forming cyst-like structures (1). Diagnosed as chronic obstructive pulmonary emphysema. What is the reason the development of such emphysema?

A. Lobar pneumonia B. Removing part of the lung C. Senile involution of the lungs D. Croupous pneumonia E. Chronic bronchitis, bronchiolitis *

193. At histological examination of lung tissue (Fig. 30) revealed thinning and discontinuities of the interalveolar septa (2) and dilatation of acini lumen(1). Diagnosed as chronic obstructive pulmonary emphysema. Name the cause of emphysema. A. penetration of air in the intermediate tissue B. removal of the lung C. unknown D. removal of part of one lung E. deficiency of protease inhibitors – elastase and collagenase *

194. Indicate the form of tumor growth, as shown by the arrow in Fig. 31, in relation to the bronchus. A. Exophytic B. Exophytic-endophytic C. Apposition D. Expansive E. Endophytic *

195. Specify the most frequent histological form of central lung cancer, shown by arrows in Fig. 31. A. Adenocarcinoma B. Solid carcinoma C. Scirrhous D. Polypous E. Squamous-cells*

196. At histological examination of the lung tumor (Fig. 31) revealed squamous cell carcinoma without keratinization. Name the pathological process in the bronchi, which precedes cancer. A. normal structure of the surface epithelium B. desquamation of lining epithelium C. necrosis of the lining epithelium D. "Intestinal" metaplasia of columnar epithelium E. metaplasia of columnar epithelium into stratified squamous *

197. Microscopic examination of tumor of the bronchus (Fig. 32) revealed layers of atypical epithelial cells (2) with the formation of "cancer pearls" (1). Enter the histological form of cancer. A. Squamous cell without keratinization B. Adenocarcinoma C. Small cell carcinomaD. Bronchioloalveolar carcinomaE. Squamous cell with keratinization *

198. At microscopic examination of tumor of the bronchus (Fig. 32) revealed layers of atypical cells of squamous epithelium (2), which formed a circular structure with keratin deposits in centre (1). Was diagnosed squamous cell carcinoma with keratinization. Name of these structural components? A. psammoma bodies B. dystrophic calcificationC. keratin pearls D. plasmoma bodies E. cancer pearls *

199. At microscopic examination of tumor of the bronchus (Fig. 32) revealed layers of atypical cells of squamous epithelium (2), which formed a circular structure with keratin deposits in centre (1). It was diagnosed squamous cell carcinoma with keratinization. Name the pathological process in the bronchi, which preceded the cancer. A. normal structure of the surface epithelium B. desquamation of lining epithelium C. necrosis of the lining epithelium D. 'Intestinal' metaplasia of columnar epithelium E. metaplasia of columnar epithelium into stratified squamous *

200. The organized blood clot with the recanalization signs was found in the road clearance of coronal artery at histological research (fig. 57). What changes are detected in the myocardium?A. Dystrophy cardiomyocytesB. CardiosclerosisC. Ischemic necrosisD. Necrobiotic changes in cardiomyocytesE. Possible options are all listed*

201. The organized blood clot with the recanalization signs was found in the road clearance of coronal artery at histological research (fig. 57). Which clinical pathology may initiate these changes?A. Stenocardia of tension (Angina)B. Stenocardia of calmness (Angina calm)C. Myocardial infarctionD. Chronic heart failureE. It is possible to all of these options*

202. The organized blood clot with the recanalization signs was found in the road clearance of coronal artery at histological research (fig. 57). Explain the meaning of the term "recanalized thrombus”.A. Purulent meltingB. Aseptic autolysisC. Accumulation of calcium saltsD. The separation of the fragments with the formation of thrombus embolismE. The cracks formation in the thrombus with a partial resumption of blood flow*

203. Fig. 57 presented the histological slice of coronary artery. What is in the aperture of the vessel?A. Tumor embolusB. Bacterial embolusC. AtheromaD. The aperture of the vessel is emptyE. Occlusive thrombus*

204. Explore the macropreparation shown in Fig. 58, and determine the most probable diagnosis.A. AortarctiaB. Traumatic rupture of the aorta wallC. Syphilitic mezaortitD. Periarteritis nodosaE. Atherosclerotic abdominal aortic aneurysm*

205. Autopsy revealed abdominal aortic aneurysm (Fig. 58). Which of the following diseases is the most common cause of this pathology?A. Hypertonic diseaseB. RheumatismC. Typhoid

D. Myocardial infarctionE. Atherosclerosis*

206. Autopsy revealed abdominal aortic aneurysm (Fig. 58). What are the possible complications of this pathology.A. Aneurysm ruptureB. Atrophy of the adjacent organsC. Thrombosis of the aortaD. ThromboembolismE. All enumerated*

207. The generalize manifestations of atherosclerosis were found at autopsy of the deceased (Fig. 58). Describe the pathological changes of the kidneys.A. Primary shriveledB. Secondary shriveledC. Shield (Thyroid)D. ShockE. Atherosclerotic shriveled*

208. At autopsy the deceased (Fig. 58) were found to generalize the manifestations of atherosclerosis. The metabolism violation of which substances are in the base of this disease?A. Simple carbohydratesB. MucopolysaccharidesC. Phosphorus and calciumsD. Iron-containing pigmentE. Imbalance of HDL and low density lipoproteides*

209. The structureless masses which resemble the contents of atheroma, the damage of fibrous structures, the proliferation of connective tissue, hemorrhage, delaying calcium salts are founded in the aorta wall at the histological examination (Fig. 59). Put the diagnosis.A. Hypertonic diseaseB. Syphilitic mezaortitC. Diabetic microangiopathyD. Wegener's granulomaE. Atherosclerosis*

210. The structureless masses which resemble the contents of atheroma, the damage of fibrous structures, the proliferation of connective tissue, hemorrhage, delaying calcium salts are founded in the aorta wall at the histological examination (Fig. 59). Determine the stage of atherosclerosis morphogenesis.A. PrelipidB. LipoidosisC. LiposclerosisD. AtherosclerosisE. Atherocalcinosis*

211. The structureless masses which resemble the contents of atheroma, the damage of fibrous structures, the proliferation of connective tissue, hemorrhage, delaying calcium salts are founded in the aorta wall at the histological examination (Fig. 59). In which stage of atherosclerosis can display such changes?A. I (initial)B. IIIC. IVD. VE. VI (finishing)*

212. The structureless masses which resemble the contents of atheroma, the damage of fibrous structures, the proliferation of connective tissue, hemorrhage, delaying calcium salts are

founded in the aorta wall at the histological examination (Fig. 59). Such microscopic changes are correspond next morphogenesis stage of atherosclerosis …A. Lipid spots and stripesB. Fibrous plaquesC. Complicated lesionsD. These changes are not characteristic for atherosclerosisE. Aterocalcinosis*

213. The changes which are typical for atherosclerosis were founded at histological examination of aorta wall (Fig. 59). Specify another name for this disease.A. Wilson's diseaseB. Cushing's syndromeC. Alzheimer diseaseD. Cat scratch diseaseE. Disease Marchand*

214. At the exploring of aorta during the autopsy the dissector found dense oval white-yellow color plates-formation, which protrude above the surface of the aorta and merge with each other (Fig. 60). Signs of ulceration and thrombotic overlay are situated on the surfaces of many plaques. Put the diagnosis.A. Hypertonic diseaseB. Syphilitic mezaortitisC. Diabetic microangiopathyD. Wegener's granulomaE. Atherosclerosis*

215. At the exploring of aorta during the autopsy the dissector found dense oval white-yellow color plates-formation, which protrude above the surface of the aorta and merge with each other (Fig. 60). Signs of ulceration and thrombotic overlay are situated on the surfaces of many plaques. The metabolism violation of which substances did initiate the pathological changes of the vascular wall?A. Simple carbohydratesB. MucopolysaccharidesC. Phosphorus and calciumD. Iron-containing pigmentE. Imbalance of HDL and low density lipoproteides*

216. At the exploring of aorta during the autopsy the dissector found dense oval white-yellow color plates-formation, which protrude above the surface of the aorta and merge with each other (Fig. 60). Signs of ulceration and thrombotic overlay are situated on the surfaces of many plaques. Such microscopic changes are correspond next morphogenesis stage of atherosclerosis …A. Grease spots and stripesB. Fibrous plaquesC. These changes are not characteristic for atherosclerosisD. AterokaltsinozaE. Complicated lesions*

217. An autopsy of the left coronary artery was made. The blood clot that completely blocks the vessel’s clearance revealed (Fig. 61). What reason did lead to such pathology in heart?A. Productive myocarditisB. Concentric hypertrophyC. Mesenchymal lipophanerosisD. Structure and function of the myocardium will not be violatedE. Myocardial infarction*

218. An autopsy of the left coronary artery was made. The blood clot that completely blocks the vessel’s clearance revealed (Fig. 61). What do you call such a blood clot?

A. HyalineB. Clot-riderC. AnkylosingD. SepticE. Occlusive*

219. An autopsy of the left coronary artery was made. The blood clot that completely blocks the vessel’s clearance revealed (Fig. 61). Morphological substrate of which disease are those changes?A. MyocarditisB. Acquired heart valvular diseaseC. Alcoholic cardiomyopathyD. Systemic lupus erythematosusE. Coronary disease*

220. In the depth of the left ventricle heart wall the pathologist found a hotbed of whitish color with a red rim, which extends to the interventricular septum (Fig. 62). These changes are situated in all layers of the heart wall. Call the diagnosis.A. Atelocardia (congenital defect of heart)B. Endocardial fibroelastosisC. Rheumatic heart diseaseD. Diffuse bacterial endocarditisE. Myocardial infarction*

221. In the depth of the left ventricle heart wall the pathologist found a hotbed of whitish color with a red rim, which extends to the interventricular septum (Fig. 62). These changes are situated in all layers of the heart wall. What are the causes of these changes?A. Productive InflammationB. Exudative inflammationC. AtrophyD. Tumor growthE. Inadequate blood supply to the myocardium*

222. In the depth of the left ventricle heart wall the pathologist found a hotbed of whitish color with a red rim, which extends to the interventricular septum (Fig. 62). These changes are situated in all layers of the heart wall. What form of myocardial infarction is this?A. IntramuralB. FocalC. SubendothelialD. SubepicardialE. Transmural*

223. In the depth of the left ventricle heart wall the pathologist found a hotbed of whitish color with a red rim, which extends to the interventricular septum (Fig. 62). These changes are situated in all layers of the heart wall. What could be the cause of death in this case?A. Ventricular fibrillationB. AsystoleC. Cardiogenic shockD. Pulmonary edemaE. Any of these enumerated reasons*

224. At the histological examination of heart tissue pathologist found extensive ischemic zone with the disappearance of transverse striation in myocardium, the many cardiomyocytes haven’t the nuclei (Fig. 63). Call the diagnosis.A. Atelocardia (congenital defect of heart)B. Endocardial fibroelastosisC. Rheumatic heart defectD. Diffuse bacterial endocarditis

E. Myocardial infarction*225. At the histological examination of heart tissue pathologist found extensive ischemic zone

with the disappearance of transverse striation in myocardium, the many cardiomyocytes haven’t the nuclei (Fig. 63).What could be the cause of death in this case? A. Ventricular fibrillationB. AsystoleC. Cardiogenic shockD. Pulmonary edemaE. Any of these enumerated reasons*

226. According to results of autopsy and pathohistological investigation (Fig. 64) myocardial infarction was diagnosed. What was the cause of the patient death?A. Thromboembolic complicationsB. Miocardiosoftening and rupture of the heart wallC. Cardiogenic shockD. Pulmonary edemaE. Any of these enumerated reasons*

Situational tasks "KROK"

Multiple oval ulcers along the intestine were revealed on autopsy of the person, who died from diffuse of peritonitis in the distant part of the small intestine. Bottom parts of the ulcers are clear, smooth, formed with muscular or serous covering, edges of ulcers are flat, rounded. There are perforations up to 0,5 cm in diameter in two ulcers. What diseases can be diagnosed?A. Typhoid feverB. DysenteryC. CholeraD. TuberculosisE. TyphusANSWER: A

A woman suffering from dysfunctional metrorrhagia was made a diagnostic abortion. Histologically in the scrape there were a lot of small stamped glandulars covered with multirowed epithelium. The lumens of some glandulars were cystically extended. Choose the variant of general pathologic process in the endometrium.A. Glandular-cystic hyperplasia of endometriumB. Atrophy of endometriumC. Metaplasia of endometriumD. Neoplasm of endometriumE. Hypertrophic growthANSWER: A

A 46 year-old man complains of difficult nose breathing. Mikulich cells, storage of epithelioid cells, plasmocytes, lymphocytes, hyaline balls are discovered in the biopsy material of the nose thickening. What is the most likely diagnosis?A. ScleromaB. Virus rhinitisC. Allergic rhinitisD. Rhinovirus infectionE. Meningococcal nasopharyngitisANSWER: A

Extensive thromboembolic infarction of the left cerebral hemispheres, large septic spleen, immunocomplex glomerulonephritis, ulcers on the edges of the aortic valves, covered with polypous thrombus with colonies of staphylococcus were revealed on autopsy of the young man who died in coma. What disease caused cerebral thromboemboly?A. Septic bacterial endocarditisB. SepticemiaC. Acute rheumatic valvulitisD. SepticopyemiaE. Rheumatic thromboendocarditisANSWER: A

A denaturation of proteins can be found in some substances. Specify the substance that is used for the incomplete denaturation of hemoglobin:A. UreaB. TolueneC. Sulfuric acidD. Nitric acid

E. Sodium hydroxideANSWER: A

A sick man with high temperature and a lot of tiny wounds on the body has been admitted to the hospital. Lice have been found in the folds of his clothing. What disease can be suspected in the patient?A. Epidemic typhusB. TularemiaC. ScabiesD. MalariaE. PlagueANSWER: A

On autopsy it is revealed that kidneys are enlarged, surface is large-granular because of multiple cavities with smooth wall, which are filled with clear fluid. What kidney disease did the patient have?A. Polycystic kidneyB. Necrotic nephrosisC. PyelonephritisD. GlomerulonephritisE. InfarctionANSWER: A

On autopsy it is revealed enlarged dense right lung, fibrin layers on the pleura. Lung tissue is light green color on incision with muddy liqued exudates. What lung disease are these symptoms typical for?A. Lung-feverB. BronchopneumoniaC. Interstitial pneumoniaD. Pulmonary gangreneіE. Fibrosing alveolitisANSWER: A

On autopsy it is revealed: soft arachnoid membrane of the upper parts of cerebral hemisphere is plethoric, it is of yellowish-green color, soaked with purulent and fibrose exudate, it lookes like cap. For what disease is it characteristical pictureA. Meningococcal meningitisB. Tuberculous meningitisC. Influenza meningitisD. Meningitis at anthraxE. Meningitis at typhusANSWER: A

For a long time a 49-year-old woman was suffering from glomerulonephritis which caused death.On autopsy it was revealed that kidneys size was 7х3х2.5 sm, weight is 65,0 g, they are dence and small-grained. Microscopically: fibrinogenous inflammation of serous and mucous capsules, dystrophic changes of parenchymatous organs, brain edema. What complication can cause such changes of serous capsules and inner organs?A. UraemiaB. AnemiaC. SepsisD. DIC-syndrome

E. ThrombopeniaANSWER: A

A 59-year-old man has signs of the parenchymatous jaundice and portal hypertension. On histological examination of the puncture of the liver bioptate, it was revealed: beam-lobule structure is affected, part of hepatocytes has signs of fat dystrophy, port-portal connective tissue septa with formation of pseudo-lobules,with periportal lympho-macrophage infiltrations. What is the most probable diagnosis?A. Liver cirrhosisB. Alcohol hepatitisC. Chronic hepatosisD. Viral hepatitisE. Toxic dystrophyANSWER: A

On microscopic examination of the enlarged neck gland of a 14-year-old girl it was revealed destruction of the tissue structure of the node, absence of the lymph follicles, sclerotic and necrosis parts, cell constitution of the node is polymorphous, lymphocites, eosinophiles, atypical cells of the large size with multiple-lobule nuclei (Beresovsky-Shternberg cells) and onenucleus cells of the large size are present. What is the most likely diagnosis?A. LymphogranulomatousB. Acute lympholeucosisC. Chronic lympholeucosisD. Berkitt's lymphomaE. Fungous mycosisANSWER: A

Local lymphonodules enlarged near the infected wound. Increased amount of macrophages, lymphocytes, lymphatic follicles in the cortical layer and large amount of plasma cells were revealed on histological examination. What process in the lymphatic nodules represent these histological changes?A. Antigen stimulationB. Acquired insufficiency of the lymphoid tissueC. Innate insufficiency of the lymphoid tissueD. Tumour transformationE. Hypersensibility reactionANSWER: A

On autopsy of the man with alcohol abuse for a long time it was revealed: dense, small-knobby, small size liver. Microscopically: small pseudo-lobules, divided with thin layers of connective tissue with lymphomacrophagial infiltrates; hepatocytes in the state of globular fatty dystrophy. What is the most likely diagnosis?A. Alcohol cirrhosisB. Chronic active alcohol hepatitisC. Chronic persistent alcohol hepatitisD. Toxic liver dystrophyE. Fatty hepatosisANSWER: A

A 30-year-old patient with bacteriologically proved dysentery developed the signs of paraproctitis. What is the stage of local changes in this patient?A. Ulceration stage

B. Fibrinous colitisC. Follicular colitisD. Catarrhal colitisE. Healing of the ulcers stageANSWER: A

The intraoperational biopsy of mammal gland has revealed the signs of atypical tissue with disorder of parenchyma stroma proportion with domination of the last, gland structures of the different size and shape, lined with single-layer proliferative epithelium. What is the most appropriate diagnosis?A. FibroadenomaB. PapillomaC. Noninfiltrative cancerD. Infiltrative cancerE. MastitisANSWER: A

Arterial hypertension, hyperglycemia, glucosuria were observed clinically for a long time in the patient with upper type of obesity. Death was due to the cerebral haemorrhage. Basophilic hypophysis adenoma, hyperplasia of adrenal gland cortex were revealed on pathomorphological examination. What is the likely diagnosis?A. Cushing diseaseB. Diabetes mellitusC. AcromegalyD. Hypophysis nanismE. Adiposogenitalis dystrophyANSWER: A

On autopsy it was revealed: large (1-2 cm) brownish-red, easy crumbling formations covering ulcerative defects on the external surface of the aortic valve. What is the most likely diagnosis?A. Polypus-ulcerative endocarditisB. Recurrent warty endocarditisC. Acute warty endocarditisD. Fibroplastic endocarditisE. Diffusive endocarditisANSWER: A

Purulent endometritis with fatal outcome was progressing in the woman after abortion performed not at the hospital. On autopsy multiple lung abscesses, subcapsule ulcers in the kidneys, spleen hyperplasia were revealed. What form of sepsis developed in the patient?A. SeptopyemiaB. SepticemiaC. ChroniosepsisD. Lung sepsisE. UrosepsisANSWER: A

Autopsy of a woman with cerebral atherosclerosis revealed in the left cerebral hemisphere a certain focus that is presented by flabby, anhistic, greyish and yellowish tissue with indistinct edges. What pathological process is the case?A. Ischemic strokeB. Multifocal tumor growth with cystic degeneration

C. Multiple foci of fresh and old cerebral hemorrhageD. Focal encephalitisE. Senile encephalopathyANSWER: A

Autopsy of a 73-year-old man who had been suffering from the coronary heart disease along with cardiac insufficiency for a long time revealed: nutmeg liver, brown induration of lungs, cyanotic induration of kidneys and spleen. What kind of circulation disorder was the cause of such effects?A. General chronic venous congestionB. Arterial hyperaemiaC. General acute venous congestionD. Acute anaemiaE. Chronic anaemiaANSWER: A

Autopsy of a man who had tuberculosis revealed a 3x2 cm large cavity in the superior lobe of the right lung. The cavity was interconnected with a bronchus, its wall was dense and consisted of three layers: the internal layer was pyogenic, the middle layer was made by tuberculous granulation tissue and the external one was made by connective tissue. What is the most likely diagnosis?A. Fibrous cavernous tuberculosisB. Fibrous focal tuberculosisC. TuberculomaD. Acute focal tuberculosisE. Acute cavernous tuberculosisANSWER: A

Autopsy of a man who died from ethylene glycol poisoning revealed that his kidneys are a little bit enlarged, edematic; their capsule can be easily removed. Cortical substance is broad and light-grey. Medullary substance is dark-red. What pathology had this man?A. Necrotic nephrosisB. Acute pyelonephritisC. Acute glomerulonephritisD. Acute tubular-interstitial nephritisE. Lipoid nephrosisANSWER: A

Histologic analysis of uterus mucous membrane revealed twisting glands, serrated and spinned, they were extended by stroma growth with proliferation of its cells. Formulate a diagnosis:A. Glandular hyperplasia of endometriumB. Acute endometritisC. LeiomyomaD. Cystic moleE. Placental polypANSWER: A

A 22 year old patient from the West Ukraine complains of laboured nasal breathing. Morphological examination of biopsy material of nasal mucous membrane revealed lymphoid, epithelioid, plasma cells as well as Mikulicz's cells. What is the most probable diagnosis?A. RhinoscleromaB. Glanders

C. TuberculosisD. LeprosyE. SyphilisANSWER: A

Autopsy of a man who had been working as a miner for many years and died from cardiopulmonary decompensation revealed that his lungs were airless, sclerosed, their apexex had emphysematous changes, the lung surface was greyish-black, the incised lung tissue was coal-black. What disease caused death?A. AnthracosisB. SilicosisC. TalcosisD. AsbestosisE. AluminosisANSWER: A

A 63 year old male patient who had been suffering from chronic diffuse obstructive disease, pulmonary emphysema, for 15 years died from cardiac insufficiency. Autopsy revealed nutmeg liver cirrhosis, cyanotic induration of kidneys and spleen, ascites, edemata of lower limbs. These changes of internal organs are typical for the following disease:A. Chronic right-ventricular insufficiencyB. Acute right-ventricular insufficiencyC. Chronic left-ventricular insufficiencyD. Acute left-ventricular insufficiencyE. General cardiac insufficiencyANSWER: A

Microscopical examination of an enlarged cervical lymph node revealed blurring of its structure, absence of lymphoid follicles; all the microscopic fields showed cells with roundish nuclei and thin limbus of basophil cytoplasm. It is known from the clinical data that other groups of lymph nodes are also enlarged as well as spleen and liver. What disease might be suspected?A. Lymphoid leukosisB. LymphogranulomatosisC. LymphosarcomaD. Myeloid leukosisE. Multiple myelomaANSWER: A

A worker of a cattle farm fell acutely ill and then died from the progressing intoxication. Autopsy revealed enlarged, hyposthenic spleen of dark-cherry colour when dissected; excessive pulp scraping. At the base and fornix of brain pia maters are edematous, soaked with blood, dark-red ("scarlet hat"). Microscopic examination revealed serous haemorrhagic inflammation of brain tissues and tunics along with destruction of small vessel walls. What is the most likely diagnosis?A. AnthraxB. TularemiaC. BrucellosisD. PlaqueE. CholeraANSWER: A

Histological examination of a skin tissue sampling revealed granulomas consisting of macrophagal nodules with lymphocytes and plasmatic cells. There are also some big macrophages with fatty vacuoles containing causative agents of a disease packed up in form of spheres (Virchow's cells). Granulation tissue is well vascularized. What disease is this granuloma typical for?A. LepraB. TuberculosisC. SyphilisD. RhinoscleromaE. GlandersANSWER: A

A 40 year old man noticed a reddening and an edema of skin in the area of his neck that later developed into a small abscess. The incised focus is dense, yellowish-green. The pus contains white granules. Histological examination revealed drusen of a fungus, plasmatic and xanthome cells, macrophages. What type of mycosis is the most probable?A. ActinomycosisB. AspergillosisC. CandidosisD. SporotrichosisE. CoccidioidomycosisANSWER: A

A physician examined a patient and found inguinal hernia. Through what anatomic formation does it penetrate into the skin?A. Hiatus saphenusB. Anulus femoralisC. Canalis adductoriusD. Lacuna musculorumE. Anulus inguinalis superficialisANSWER: A

Autopsy of a man who died from burn disease revealed brain edema, liver enlargement as well as enlargement of kidneys with wide light-grey cortical layer and plethoric medullary area. Microscopic examination revealed necrosis of tubules of main segments along with destruction of basal membranes, intersticium edema with leukocytic infiltration and haemorrhages. What is the most probable postmortem diagnosis?A. Necrotic nephrosisB. Tubulointerstitial nephritisC. PyelonephritisD. Gouty kidneyE. Myeloma kidneyANSWER: A

A 30 year old man had been suffering from acute respiratory disease and died from cardiopulmonary decompensation. Autopsy revealed fibrinous-haemorrhagic inflammation in the mucous membrane of larynx and trachea, destructive panbronchitis, enlarged lungs that look black due to the multiple abcesses, haemorrhages, necrosis. What is the most probable postmortem diagnosis?A. InfluenzaB. ParainfluenzaC. Respiratory syncytial infection

D. MeaslesE. Adenoviral infectionANSWER: A

A man with a wound of his limb that had been suppurating for a long time died from intioxication. Autopsy revealed extreme emaciation, dehydration, brown atrophy of liver, myocardium, spleen and cross-striated muscles as well as renal amyloidosis. What diagnosis corresponds with the described picture?A. ChroniosepsisB. SepticopyemiaC. SepticemiaD. Chernogubov's syndromeE. BrucellosisANSWER: A

A patient with android-type obesity had been suffering from arterial hypertension, hyperglycemia, glycosuria for a long time and died from the cerebral haemorrhage. Pathologic examination revealed pituitary basophil adenoma, adrenal cortex hyperplasia. What is the most likely diagnosis?A. Itsenko-Cushing's syndromeB. Diabetes mellitusC. AcromegaliaD. Pituitary nanismE. Adiposogenital dystrophyANSWER: A

Mucous membrane of the right palatine tonsil has a painless ulcer with smooth lacquer fundus and regular cartilagenous edges. Microscopically: inflammatory infiltration that consists of lymphocytes, plasmocytes, a small number of neutrophils and epithelioid cells; endovasculitis and perivasculitis. What disease is it?A. SyphilisB. ActinomycosisC. TuberculosisD. Pharyngeal diphtheriaE. Ulcerous necrotic Vincent's anginaANSWER: A

Autopsy of a man with a malignant stomach tumour who had died from cancer intoxication revealed in the posteroinferior lung fields some dense, grayish-red irregular foci protruding above the section surface. Microscopic examination revealed exudate containing a large amount of neutrophils in the lumen and walls of small bronchi and alveoles. Such pulmonary alterations indicate the following disease:A. Acute purulent bronchopneumoniaB. Acute bronchitisC. Croupous pneumoniaD. Intermittent pneumoniaE. Acute serous bronchopneumoniaANSWER: A

Microscopical examination of a removed appendix revealed an edema, diffuse neutrophilic infiltration of appendix wall along with necrosis and defect of mucous membrane with affection of its muscle plate. What appendicitis form was developed?

A. UlcerophlegmonousB. PhlegmonousC. GangrenousD. SuperficialE. ApostematousANSWER: A

A 39 y.o. woman went through an operation in course of which surgeons removed her uterine tube that was enlarged and a part of an ovary with a big cyst. Histological examination of a tube wall revealed decidual cells, chorion villi. What was the most probable diagnosis made after examination of the uterine tube?A. Tubal pregnancyB. Placental polypC. ChoriocarcinomaD. Papyraceous fetusE. LithopedionANSWER: A

Autopsy of a 1,5-year-old child revealed haemorrhagic skin rash, moderate hyperaemia and edema of nasopharyngeal mucous membrane, small haemorrhages in the mucous membranes and internal organs; dramatic dystrophic alterations in liver and myocardium; acute necrotic nephrosis; massive haemorrhages in the adrenal glands. What disease are these alterations the most typical for?A. Meningococcal infectionB. Scarlet feverC. DiphtheriaD. MeaslesE. Epidemic typhusANSWER: A

48 hours after performing tuberculin test (Mantoux test) to a child a 10 mm papule appeared on the spot of tuberculin introduction. What hypersensitivity mechanism underlies these changes?A. Cellular cytotoxicityB. AnaphylaxisC. Antibody-dependent cytotoxicityD. Immune complex cytotoxicityE. GranulomatosisANSWER: A

Colonoscopy of a patient ill with dysentery revealed that mucous membrane of his large intestine is hyperemic, edematic, its surface was covered with grey-and-green coats. Name the morphological form of dysenteric collitis:A. FibrinousB. CatarrhalC. UlcerousD. PurulentE. NecroticANSWER: A

A patient has been syffering from diarrhea for 5 day. On the fith day colonoscopy revealed that membrane of rectum was inflamed, there were greyish-green films closely adhering to the subjacent tissue. What is the most probable diagnosis?

A. DysenteryB. Typhoid feverC. Nonspecific ulcerous colitisD. SalmonellosisE. Crohn's diseaseANSWER: A

Autopsy of a 48 y.o. man revealed a round formation 5 cm in diameter with clear-cut outlines in the region of the 1st segment of his right lung. This formation was encircled with a thin layer of connective tissue full of white brittle masses. Make a diagnosis of the secondary tuberculosis form:A. TuberculomaB. Caseous pneumoniaC. Acute cavernous tuberculosisD. Acute focal tuberculosisE. Fibrous cavernous tuberculosisANSWER: A

A man had worked in a coal mine for over 20 years. After his death autopsy revealed that his lungs were dense, grayish-black and had large areas of neogenic connective tissue containing a lot of microphages with black pigment in the cytoplasm. What is the most likely diagnosis?A. AnthracosisB. AnthracosilicosisC. SilicoanthracosisD. TalcosisE. SiderosisANSWER: A

Skin of a man who died from cardiac insufficiency has an eruption in form of spots and specks. There are also bedsores in the area of sacrum and spinous vertebral processes. Microscopical examination of CNS, skin, adrenal glands revealed in the vessels of microcirculatory bed and in small arteries destructive-proliferative endothrombovasculitis with Popov's granulomas; interstitial myocarditis. What diagnosis corresponds with the described picture?A. Spotted feverB. Q feverC. Enteric feverD. Nodular periarteritisE. HIVANSWER: A

Autopsy of a 17 year old girl who died from pulmonary failure revealed a small area of caseous necrosis in the inferior lobe of the right lung, and occurences of caseous necrosis in the bronchopulmonary, bronchial and bifurcational lymph nodes. What is the most probable postmortem diagnosis?A. Primary tuberculosisB. Hematogenous progression of primary tuberculosisC. Hematogenous tuberculosis with predominant lung affectionD. TuberculomaE. Caseous pneumonia under secondary tuberculosisANSWER: A

Autopsy of a man who died from the sepsis in his femoral bone revealed phlegmonous inflammation that affected the marrow, haversian canals and periosteum. Under the periosteum there are multiple abscesses, adjoining soft tissues of thigh also have signs of phlegmonous inflammation. What pathological process was described?A. Acute hematogenous osteomyelitisB. OsteoporosisC. Chronic hematogenous osteomielitisD. OsteopetrosisE. -ANSWER: A

An experimental animal was first sensibilized whereupon an antigen dose was introduced subcutaneously. This injection resulted in the development of a fibrinous inflammation with alteration of vessel walls, basal substance and fibrous structures of connective tissue in form of mucoid and fibrinoid swelling and necrosis. What immunological reaction took place?A. Immediate hypersensitivityB. Delayed-type hypersensitivityC. Reaction of transplantation immunityD. Normergic reactionE. GranulomatosisANSWER: A

A 4 year old child complained of pain during deglutition, indisposition. Objectively: palatine arches and tonsils are moderately edematic and hyperemic, there are greyish-white films up to 1 mm thick closely adhering to the subjacent tissues. What pathological process are these changes typical for?A. InflammationB. DystrophyC. NecrosisD. MetaplasiaE. OrganizationANSWER: A

A 9 m.o. child has delayed dentition, it is also out of order. Upper jaw configuration is horizontal ("high" palate); microscopically - irregular mineralization of tooth enamel, wrinkled enamel prisms, some of them are vacuolized. Predentin zone is extended; there are solitary denticles. What disease is it?A. Early ricketsB. Late ricketsC. OsteomalaciaD. GoutE. Hypervitaminosis DANSWER: A

Microscopical renal examination of a 36 y.o. woman who died from renal insufficiency revealed in the glomerules proliferation of capsule nephrothelium as well as of podocytes and phagocytes accompanied by formation of "crescents", capillary loop necrosis, fibrinous thrombs in their lumens; sclerosis and hyalinosis of glomerules, atrophy of tubules and fibrosis of renal stroma. What is the most probable diagnosis?A. Subacute glomerulonephritisB. Acute glomerulonephritisC. Chronic glomerulonephritis

D. Focal segmentary sclerosisE. Membranous nephropathyANSWER: A

A forensic medical expert examines the body of a 58 y.o. man who had been consuming large amounts of alcochol for a long time and died at home. Microscopicaly: the right lung is dense and enlarged, its incision revealed that the tissue is greyish and homogenous, pleura is covered with greyish layers. Microscopically - alveolar cavities contain fibrin, hemolyzed erythrocytes. Make a diagnosis:A. Croupous pneumoniaB. Focal pneumoniaC. Interstitial pneumoniaD. Primary pulmonary tuberculosisE. Caseous pneumoniaANSWER: A

Autopsy of a 50-year-old man revealed the following changes: his right lung was moderately compact in all parts, the dissected tissue was found to be airless, fine-grained, dryish. Visceral pleura had greyish-brown layers of fibrin. What is the most likely diagnosis?A. Croupous pneumoniaB. TuberculosisC. BronchopneumoniaD. Interstitial pneumoniaE. PneumofibrosisANSWER: A

A 22 y.o. woman has enlarged lymph nodes. Histologically: a lymph node contains lymphocytes, histiocytes, reticular cells, small and big Hodgkin's cells, multinucleated Sternberg cells, isolated foci of caseous necrosis. What disease are these changes typical for?A. LymphogranulomatosisB. LymphosarcomaC. Chronic leukosisD. Acute leukosisE. Lung cancer metastasisANSWER: A

Analysis of a punction biopsy material of liver revealed hepatocyte dystrophy with necroses as well as sclerosis with disorder of beam and lobulous structure, with formation of pseudolobules and regenerative nodes. What is the most probable diagnosis:A. Liver cirrhosisB. Chronic hepatosisC. Chronic hepatitisD. Progressive massive liver necrosisE. Acute hepatitisANSWER: A

Autopsy of a man, who had been suffering from the multiple bronchiectasis for 5 years and died from chronic renal insufficiency, revealed that kidneys were dense and enlarged, with thickened cortical layer of white colour with greasy lustre. What renal disease might be suspected?A. Secondary amyloidosisB. GlomerulonephritisC. Chronic pyelonephritis

D. Necrotic nephrosisE. -ANSWER: A

Autopsy of a 49-year-old woman who died from chronic renal insufficiency, revealed: kidneys were dense, reduced, multicoloured, with haemorrhagic areas. Microscopic examination revealed some hematoxylin bodies in the nuclei of the renal tubule epithelium; "wire-loop" thickening of the glomerular capillary basement membrane; here and there in the capillaries some hyaline thrombi and foci of fibrinoid necrosis were present. What is the most likely diagnosis?A. Systemic lupus erythematosusB. RheumatismC. Arteriosclerotic pneumosclerosisD. AmyloidosisE. Atherosclerotic nephrosclerosisANSWER: A

Unpainfull formation without marked borders appeared in the soft tissues of the thigh in the young man. On the tissue bioptate the formation lookes like a meat of a fish, consisting of the immature fibroblast-like cells with multiple mitosis, which grow through the muscles. What is the most likely diagnosis?A. FibrosarcomaB. MyosarcomaC. FibromaD. CancerE. MyomaANSWER: A

A 39 year old man who had been operated for the stomach ulcer died 7 days after the surgery. Autopsy revealed that peritoneal leaves were dull, plephoric, covered with massive yellow-greenish films, the peritoneal cavity contained for about 300 ml of thick yellow-greenish liquid. What pathologic process was revealed in the peritoneal cavity?A. Fibrinous suppurative peritonitisB. Serous peritonitisC. Fibrinous serous peritonitisD. Peritoneal commissuresE. Fibrinous haemorrhagic peritonitisANSWER: A

A 45 y.o. patient consulted a doctor about plaque-shaped formation on his neck. Histological examination of biopsy skin material revealed tumourous cells of round and oval form with thin ring of basophilic cytoplasma that resemble of cells of basal epidermal layer. What tumour is it?A. BasaliomaB. Epidermal cancerC. HydradenomaD. TrichoepitheliomaE. SyringoadenomaANSWER: A

A 71 year old man had been presenting with diarrhea for 10 days. The feces had admixtures of blood and mucus. He was delivered to a hospital in grave condition and died 2 days later. Autopsy of the body revealed the following: diphtheritic colitis with multiple irregularly-shaped

ulcers of different depth in both sigmoid colon and rectus. Bacteriological analysis revealed Shigella. What was the main disease?A. DysenteryB. Typhoid feverC. SalmonellosisD. Nonspecific ulcerous colitisE. YersiniosisANSWER: A

A 63 y.o. man fell ill with acute tracheitis and bronchitis accompanied by bronchial pneumonia. On the 10th day the patient died from cardiopulmonary insufficiency. Autopsy revealed fibrinous hemorrhagic laryngotracheobronchitis; lungs were enlarged, their incision revealed the "coal-miner's" effect caused by interlacing of sections of bronchial pneumonia, hemorrhages into the pulmonary parenchyma, acute abscesses and atelectases. Internal organs have discirculatory and dystrophic changes. What is the most probable diagnosis?A. Influenza, severe formB. Moderately severe influenzaC. ParainfluenzaD. Respiratory syncytial infectionE. Adenoviral infectionANSWER: A

A boy is 7 y.o. Objectively: against the background of hyperemic skin there is knobby bright-pink rash on his forehead, neck, at the bottom of abdomen, in the popliteal spaces; nasolabial triangle is pale. Examination of oropharyngeal surface revealed localized bright-red hyperemia; tonsils are swollen, soft, lacunas contain pus, tongue is crimson. Cervical lymph nodes are enlarged, dense and painful. What is the most probable diagnosis?A. Scarlet feverB. RubellaC. Whooping coughD. DiphtheriaE. Infectious mononucleosisANSWER: A

Autopsy of a 75 year old patient who had been suffering from disseminated atherosclerosis and died under chronic cardiac failure revealed constriction and deformation of coronary arteries, tuberous intima whose section appeared to be white and petrosal. Specify the stage of atherosclerosis morphogenesis:A. AtherocalcinosisB. LipoidosisC. LiposclerosisD. BilipidE. AtheromatosisANSWER: A

Examination of a bronchial tissue sample revealed atrophy of mucous membrane, cystic degeneration of glands, focal metaplastic changes of lining prismatic epithelial cells into multilayer squamous cells; increase in goblet cell number; in some parts of bronchial wall and especially in the mucous membrane there was marked cellular inflammatory infiltration and growth of granulation tissue bulging into the bronchial lumen in form of a polyp. What is the most likely diagnosis?A. Chronic bronchitis

B. Lobar pneumoniaC. Acute bronchitisD. BronchopneumoniaE. Interstitial pneumoniaANSWER: A

Gynecological examination of the uterine cervix in a 30-year-old woman revealed some bright-red lustrous spots that easily bleed when touched. Biopsy showed that a part of the uterine cervix was covered with cylindrical epithelium with papillary outgrowths; in the depth of tissue the growth of glands was present. What pathology of the uterine cervix was revealed?A. PseudoerosionB. True erosionC. EndocervicitisD. Glandular hyperplasiaE. LeukoplakiaANSWER: A

A stillborn child was found to have thickened skin resembling of the tortoise shell, underdeveloped auricles. Histological examination of skin revealed hyperkeratosis, atrophy of the granular epidermis layer; inflammatory changes were not present. What is the most likely diagnosis?A. IchthyosisB. LeukoplakiaC. XerodermiaD. ErythroplakiaE. DermatomyositisANSWER: A

Acute renal impairment caused death of a bleeding patient. Autopsy revealed enlarged kidneys with a broad pale pink cortical layer expressively demarcated from dark red renal pyramids. Macroscopic examination revealed lack of epithelial nuclei of convoluted tubules, tubulorrhexis, phlebostasis. The cell nuclei of choroid glomus and straight tubules were present. What pathology is it?A. NecronephrosisB. InfarctionC. GlomerulonephritisD. PyelonephritisE. NephrosisANSWER: A

A pathology-histology laboratory received a vermiform appendix up to 2,0 cm thick. Its serous membrane was pale, thick and covered with yellowish-green films. The wall was flaccid, of grayish-red colour. The appendix lumen was dilated and filled with yellowish-green substance. Histological examination revealed that the appendix wall was infiltrated with neutrophils. Specify the appendix disease:A. Acute phlegmonous appendicitisB. Acute gangrenous appendicitisC. Acute superficial appendicitisD. Acute simple appendicitisE. Chronic appendicitisANSWER: A

A 46 year old patient who had been suffering from tuberculosis for 6 years died from massive pulmonary haemorrhage. Autopsy revealed different-sixed foci of sclerosis and caseous necrosis in lungs, in the upper part of the right lung there was a cavity 5 cm in diameter with dense grey walls, the cavity contained liquid blood and blood clots. What type of tuberculosis is it?A. FibrocavernousB. Acute cavernousC. InfiltrativeD. Fibrous focalE. Acute focalANSWER: A

A patient died from cardiopulmonary decompensation. Histological examination revealed diffused pulmonary lesion together with interstitial edema, infiltration of tissue by limphocytes, macrophages, plasmocytes; pulmonary fibrosis, panacinar emphysema. What disease corresponds with the described picture?A. Fibrosing alveolitisB. Chronic bronchitisC. BronchopneumoniaD. Pulmonary atelectasisE. Bronchial asthmaANSWER: A

A 50 year old patient has been taking treatment thrice for the last 6 months because of fractures caused by domestic accidents. Microscopical examination of bony tissue revealed foci of lacunar resolution, giant-cell granulomas in the tumour-like formations, cysts. Bony tissue was substituted by fibrous connective tissue. Examination revealed also adenoma of parathyroid gland and hypercalcemia. What is the most probable diagnosis?A. Parathyroid osteodystrophyB. MyelomatosisC. OsteomyelitisD. OsteopetrosisE. Paget's diseaseANSWER: A

2 days after labour a woman developed shock along with DIC syndrome that caused her death. Autopsy revealed purulent endomyometritis, regional purulent lymphangitis, lymphadenitis and purulent thrombophlebitis. There were also dystrophic alterations and interstitial inflammation of parenchymal organs. What is the most likely diagnosis?A. SepticemiaB. SyphilisC. Tuberculosis of genital organsD. Chorioadenoma destruensE. Hydatid moleANSWER: A

A patient who abuses smoking has chronic bronchitis. Biopsy of his primary bronchus revealed multilayer pavement epithelium. What pathological process was revealed in the bronchus?A. MetaplasiaB. Physiological regenerationC. Reparative regenerationD. HyperplasiaE. Dysplasia

ANSWER: A

A 50 year old patient underwent resection of tumour of large intestine wall. Microscopically it presents itself as fascicles of divergent collagen fibers of different thickness and form and some monomorphous fusiform cells that are irregularly distributed among the fibers. Cellular atypia is not evident. What tumour is it?A. Hard fibromaB. FibromyomaC. Soft fibromaD. DesmomaE. FibrosarcomaANSWER: A

Autopsy of a 5 year old child revealed in the area of vermis of cerebellum a soft greyish-pink node 2 cm in diameter with areas of haemorrhage. Histologically this tumour consisted of atypical monomorphous small roundish cells with big polymorphous nuclei. What tumour is it?A. MedulloblastomaB. MeningiomaC. GlioblastomaD. AstrocytomaE. OligodendrogliomaANSWER: A

In course of severe respiratory viral infection there appeared clinical signs of progressing cardiac insufficiency that caused death of a patient in the 2nd week of disease. Autopsy revealed that the heart was sluggish, with significant cavity dilatation. Histological examination of myocardium revealed plephora of microvessels and diffuse infiltration of stroma by lymphocytes and histiocytes. What disease corresponds with the described picture?A. MyocarditisB. StenocardiaC. Acute coronary insufficiencyD. Myocardium infarctionE. CardiomyopathyANSWER: A

A patient with high-titer antinuclear antibodies died from progressing renal impairment. Autopsy revealed mesangioproliferative glomerulonephritis and abacterial polypous endocarditis. There was periarterial bulbar sclerosis in spleen and productive proliferative vasculitis in skin. What is the most likely diagnosis?A. Systemic lupus erythematosusB. Nephrotic syndromeC. RheumatismD. DermatomyositisE. Periarteritis nodosaANSWER: A

A 38 year old patient with full-blown jaundice, small cutaneous hemorrhages, general weakness and loss of appetite underwent puncture biopsy of liver. Histological examination revealed disseminated dystrophy, hepatocyte necrosis, Councilman's bodies. Lobule periphery has signs of significant infiltration by lymphocytes, there are also individual multinuclear hepatocytes. What is the most probable diagnosis?A. Acute viral hepatitis

B. Acute alcoholic hepatitisC. Miliary hepatic cirrhosisD. Toxic degeneration of liverE. Chronic hepatitisANSWER: A

A 20 year old patient died from intoxication 8 days after artificial illegal abortion performed in her 14-15th week of pregnancy. Autopsy of the corpse revealed yellowish colour of eye sclera and of skin, necrotic suppurative endometritis, multiple pulmonary abscesses, spleen hyperplasia with a big number of neutrophils in its sinuses. What complication after abortion was developed?A. SepticopyemiaB. SepticemiaC. Hemorrhagic shockD. ChroniosepsisE. Viral hepatitis type AANSWER: A

A section of the left lung was found to have an area of dense red tissue. The area was cone-shaped, stood out distinctly from the healthy tissue, with its base directed to the pleura. The dissected tissue was granular, dark-red. What is the most likely diagnosis?A. Haemorrhagic infarctionB. Lung abscessC. Lung gangreneD. Primary tuberculous affectionE. Croupous pneumoniaANSWER: A

A patient has a cluster of matted together dense lymph nodes on his neck. Histological examination of a removed lymph node revealed proliferation of reticular cells, presense of Reed-Sternberg cells. What disease is meant? A. LymphogranulomatosisB. Lymphoblastic leukosisC. Myeloblastic leukosisD. Myelocytic leukosisE. Lymphocytic leukosisANSWER: A

A patient had been suffering from profuse diarrhea and vomiting for 2 days. He died from acute dehydration. Autopsy revealed that the intestinal wall was edematic and hyperemic, with multiple haemorrhages in the mucous membrane. Intestine lumen contains whitish fluid resembling of rice water. What disease caused death?A. CholeraB. DysenteryC. SalmonellosisD. Typhoid feverE. EnterocolitisANSWER: A

Examination of a 66 year old patient revealed a lytic tumour in the locus of pathological rib fracture. Histologically this tumour consists of atypical plasmoblasts. Further examination revealed osteoporosis in the bones of vertebral column and pelvis. These changes are typical for:A. Myelomatosis

B. Tuberculous osteomyelitisC. Ewing's osteosarcomaD. NeuroblastomaE. Metastatic lung cancerANSWER: A

A patient died from acute cardiac insufficiency, among clinical presentations there was gastrointestinal haemorrhage. Examination of mucous membrane of sromach revealed some defects reaching myenteron; their edges and bottom were mostly even and loose, some of them contained dark-red blood. What pathological process was revealed?A. Acute ulcersB. Chronic ulcersC. ErosionsD. ThrombosisE. InflammationANSWER: A

A 33 year old man died from uraemia. Autopsy revealed enlarged kidneys weighing 500,0 each and consisting of multiple cavities 0,5-2 cm in diameter. The cavities were full of light-yellow transparent liquid. Renal pelvis and ureters had no pecularities. What renal disease caused uraemia?A. Bilateral polycystic renal diseaseB. Chronic pyelonephritisC. Renal tumourD. Renal tuberculosisE. Rapidly progressing glomerulonephritisANSWER: A

A patient ill with tuberculosis died from progressing cardiopulmonary decompensation. Autopsy in the area of the right lung apex revealed a cavity 5 cm in diameter communicating with lumen of a segmental bronchus. On the inside cavity walls are covered with caseous masses with epithelioid and Langhans cells beneath them. What morphological form of tuberculosis is it?A. Acute cavernous tuberculosisB. TuberculomaC. Caseous pneumoniaD. Infiltrative tuberculosisE. Acute focal tuberculosisANSWER: A

A 2 year old child had acute respiratory viral infection and died from cardiopulmonary decompensation. Autopsy revealed that his right lung was hyperemic; in the 2nd, 6th and 10th segments and on the incision there were airless yellowish foci of irregular form, from several mm up to 1 cm large. Microscopical examination revealed exudate consisting mainly of neutrophils in the given areas of pulmonary tissue in the alveoles, bronchioles and bronchial tubes. What is the most probable diagnosis?A. Focal pneumoniaB. Interstitial pneumoniaC. Croupous pneumoniaD. Acute bronchitisE. Pulmonary abscessANSWER: A

The upper lobe of the right lung is enlarged, grey and airless, the inscision surface is dripping with turbid liquid, the pleura has many fibrinogenous films; microscopical examination of alveoles revealed exudate containing neutrophils, desquamated alveolocytes and fibrin fibers. The bronchus wall is intact. What is the most probable diagnosis?A. Croupous pneumoniaB. Interstitial pneumoniaC. Pulmonary abscessD. Focal pneumoniaE. Influenzal pneumoniaANSWER: A

A 28 year old patient had high arterial pressure, hematuria and facial edemata. In spite of treatment renal insufficiency was progressing. 6 months later the patient died from uremia. Microscopic examination of his kidneys and their glomerules revealed proliferation of capsule nephrothelium and of podocytes with "demilune" formation, sclerosis and hyalinosis of glomerules. What disease corresponds with the described picture?A. Subacute glomerulonephritisB. Acute pyelonephritisC. Nephrotic syndromeD. Chronic glomerulonephritisE. Acute glomerulonephritisANSWER: A

Autopsy of a man ill with severe hypothyroidism revealed that connective tissue, organ stroma, adipose and cartilaginous tissues were swollen, semitransparent, mucus-like. Microscopic examination of tissues revealed stellate cells having processes with mucus between them. What type of dystrophy is it?A. Stromal-vascular carbohydrateB. Stromal-vascular adiposeC. Stromal-vascular proteinaceousD. Parenchymatous proteinaceousE. Parenchymatous adiposeANSWER: A